diff --git "a/Rationale/task52_mmedbench_explanation_english_test_only.jsonl" "b/Rationale/task52_mmedbench_explanation_english_test_only.jsonl" new file mode 100644--- /dev/null +++ "b/Rationale/task52_mmedbench_explanation_english_test_only.jsonl" @@ -0,0 +1,808 @@ +{ + "Contributors": "MAGIC", + "Source": "MMedBench", + "URL": "https://huggingface.co/datasets/Henrychur/MMedBench", + "Categories": [ + "Explanation" + ], + "Definition": [ + "Given a question and its answer, generate an explanation statement." + ], + "Reasoning": [], + "Input_language": [ + "English" + ], + "Output_language": [ + "English" + ], + "Instruction_language": [ + "English" + ], + "Domains": [ + "Public Health" + ], + "Positive Examples": [], + "Negative Examples": [], + "Instances": [ + { + "input": "Question: Two weeks after undergoing an emergency cardiac catherization with stenting for unstable angina pectoris, a 61-year-old man has decreased urinary output and malaise. He has type 2 diabetes mellitus and osteoarthritis of the hips. Prior to admission, his medications were insulin and naproxen. He was also started on aspirin, clopidogrel, and metoprolol after the coronary intervention. His temperature is 38°C (100.4°F), pulse is 93/min, and blood pressure is 125/85 mm Hg. Examination shows mottled, reticulated purplish discoloration of the feet. Laboratory studies show:\nHemoglobin count 14 g/dL\nLeukocyte count 16,400/mm3\nSegmented neutrophils 56%\nEosinophils 11%\nLymphocytes 31%\nMonocytes 2%\nPlatelet count 260,000/mm3\nErythrocyte sedimentation rate 68 mm/h\nSerum\nUrea nitrogen 25 mg/dL\nCreatinine 4.2 mg/dL\nRenal biopsy shows intravascular spindle-shaped vacuoles. Which of the following is the most likely cause of this patient's symptoms?\"\nOptions: A: Renal papillary necrosis\tB: Cholesterol embolization\tC: Eosinophilic granulomatosis with polyangiitis\tD: Polyarteritis nodosa\t\nThe right answer is B: Cholesterol embolization.", + "output": "The most likely cause of this patient's symptoms, considering the clinical presentation and laboratory findings, is B. Cholesterol embolization (also known as atheroembolism). The patient's recent history of cardiac catheterization with stenting raises the possibility of cholesterol crystal embolization, which can occur after vascular procedures. The mottled, reticulated purplish discoloration of the feet (livedo reticularis) and the renal biopsy showing intravascular spindle-shaped vacuoles are characteristic findings of cholesterol crystal embolization. The elevated ESR and eosinophilia could also be associated with this condition. While renal papillary necrosis, polyarteritis nodosa, and eosinophilic granulomatosis with polyangiitis may have similar presentations, the recent cardiac intervention and specific biopsy findings make cholesterol embolization the most likely diagnosis." + }, + { + "input": "Question: A 72-year-old man comes to the physician because of a 2-month history of fatigue and worsening abdominal pain. During this period, he also has excessive night sweats and shortness of breath on exertion. Over the past 3 months, he has had a 5.6-kg (12-lb) weight loss. He had a myocardial infarction 3 years ago. He has hypertension, diabetes mellitus, and chronic bronchitis. His medications include insulin, aspirin, lisinopril, and an albuterol inhaler. He has smoked half a pack of cigarettes for the past 45 years. Vital signs are within normal limits. The spleen is palpated 6 cm below the left costal margin. Laboratory studies show:\nHemoglobin 6.4 g/dL\nMean corpuscular volume 85 μm3\nLeukocyte count 5,200/mm3\nPlatelet count 96,000/mm3\nA blood smear is shown. Bone marrow aspiration shows extensive fibrosis and a few scattered plasma cells. A JAK 2 assay is positive. Which of the following is the most appropriate next step in management?\"\nOptions: A: Cladribine\tB: Prednisone\tC: Imatinib\tD: Ruxolitinib\t\nThe right answer is D: Ruxolitinib.", + "output": "The clinical picture and laboratory findings are suggestive of myelofibrosis, a myeloproliferative disorder characterized by bone marrow fibrosis, cytopenias, extramedullary hematopoiesis, and commonly, splenomegaly. This is supported by the findings of anemia, thrombocytopenia, splenomegaly, and the presence of a JAK 2 mutation. The appropriate treatment for symptomatic myelofibrosis, particularly in the presence of a JAK 2 mutation, is Ruxolitinib, a JAK 1/2 inhibitor. Ruxolitinib can help reduce spleen size and alleviate constitutional symptoms. Other options, such as Cladribine and Prednisone, are not standard treatments for myelofibrosis. Imatinib is used for chronic myelogenous leukemia, not myelofibrosis. Therefore, D. Ruxolitinib is the most appropriate next step in management." + }, + { + "input": "Question: A 59-year-old overweight woman presents to the urgent care clinic with the complaint of severe abdominal pain for the past 2 hours. She also complains of a dull pain in her back with nausea and vomiting several times. Her pain has no relation with food. Her past medical history is significant for recurrent abdominal pain due to cholelithiasis. Her father died at the age of 60 with some form of abdominal cancer. Her temperature is 37°C (98.6°F), respirations are 15/min, pulse is 67/min, and blood pressure is 122/98 mm Hg. Physical exam is unremarkable. However, a CT scan of the abdomen shows a calcified mass near her gallbladder. Which of the following diagnoses should be excluded first in this patient?\nOptions: A: Acute cholecystitis\tB: Gallbladder cancer\tC: Choledocholithiasis\tD: Pancreatitis\t\nThe right answer is B: Gallbladder cancer.", + "output": "Given the patient’s history of recurrent abdominal pain due to cholelithiasis, a family history of abdominal cancer, and the finding of a calcified mass near the gallbladder on CT, Gallbladder cancer (Option B) should be excluded first. Her age, overweight status, and the calcified mass are risk factors for gallbladder cancer. While her symptoms could also suggest acute cholecystitis, choledocholithiasis, or pancreatitis, the combination of a calcified mass and familial cancer history increases the index of suspicion for a malignant process. Rapid exclusion of cancer is essential due to its potentially aggressive nature and the need for timely intervention." + }, + { + "input": "Question: A 3-month-old boy is brought the emergency department by his parents after an episode of cyanosis and muscle hypotonia that resolved after 2 minutes. Diagnostic evaluation fails to discover an exact etiology of the boy's symptoms and the episode is classified as a brief resolved unexplained event (BRUE). The risk profile for BRUE in infants remains largely unknown. The pediatrician who saw the boy in the emergency department is trying to identify risk factors for BRUE. She is aware of several confounders, including age, socioeconomic background, and family history of medical illness. She recruits 75 infants under 1 year of age with BRUE and 75 infants without BRUE of the same age, socioeconomic background, and family history of medical illness. She then compares the two groups with regard to history of feeding problems and history of recent upper respiratory infection. Which of the following methods was conducted to control confounding bias in the study?\nOptions: A: Blinding\tB: Restriction\tC: Randomization\tD: Matching\t\nThe right answer is D: Matching.", + "output": "Matching is the method used to control confounding bias in this pediatrician's study. She selected infants with BRUE and a comparison group without BRUE but matched for age, socioeconomic background, and family history of medical illness. Matching ensures that both the study and comparison groups are similar in regards to specific confounding variables. This helps isolate the variables of interest – history of feeding problems and recent upper respiratory infections – by reducing the likelihood that confounding factors will distort the association between the exposure and the outcome. Randomization would involve randomly assigning subjects to different groups, which isn't applicable here since the groups are already determined by their BRUE-status. Blinding is a method to prevent bias in the assessment of outcomes, not to control for confounding. Restriction would involve only including subjects with certain characteristics, which was not the specific approach described here." + }, + { + "input": "Question: A 3-month-old infant is brought to her pediatrician because she coughs and seems to have difficulty breathing while feeding. In addition, she seems to have less energy compared to other babies and appears listless throughout the day. She was born by cesarean section to a G1P1 woman with no prior medical history and had a normal APGAR score at birth. Her parents say that she has never been observed to turn blue. Physical exam reveals a high-pitched holosystolic murmur that is best heard at the lower left sternal border. The most likely cause of this patient's symptoms is associated with which of the following abnormalities?\nOptions: A: 22q11 deletion\tB: Deletion of genes on chromosome 7\tC: Lithium exposure in utero\tD: Maternal alcohol consumption\t\nThe right answer is A: 22q11 deletion.", + "output": "The presenting symptoms of coughing, difficulty breathing while feeding, lethargy, and a high-pitched holosystolic murmur are suggestive of a congenital heart defect. A common condition associated with congenital heart defects is 22q11 deletion, also known as DiGeorge syndrome, which can cause abnormal development of the heart and results in cardiac murmurs and feeding difficulties due to poor coordination of sucking and swallowing. Deletion of genes on chromosome 7 is related to Williams syndrome, which doesn't typically present with congenital heart disease of this nature. Lithium exposure in utero and maternal alcohol consumption have known teratogenic effects but are less likely to cause the cardiac abnormalities indicated by the murmur described in the question. Hence, option A, a 22q11 deletion, is the most likely cause of this infant's symptoms." + }, + { + "input": "Question: A 30-year-old African American woman comes to the physician for the evaluation of a dry cough and chest discomfort for the past 3 days. During this period, the patient has had headaches, muscle aches, joint pain, fever, and chills. Ten days ago, she was hiking with her family in Mississippi. The patient has asthma that is treated with an albuterol inhaler. Her mother has a lung disease treated with methotrexate. The patient has smoked one pack of cigarettes daily for the past 10 years. Her temperature is 38°C (100.4°F). Physical examination shows slight wheezes throughout both lung fields. Laboratory studies and urinalysis are positive for polysaccharide antigen. Bronchoalveolar lavage using silver/PAS-staining shows macrophages filled with a dimorphic fungus with septate hyphae. Which of the following is the most likely cause of this patient's symptoms?\nOptions: A: Legionella pneumophila infection\tB: Pneumocystis pneumonia\tC: Histoplasma capsulatum infection\tD: Blastomyces dermatitidis infection\t\nThe right answer is C: Histoplasma capsulatum infection.", + "output": "The most likely cause of this patient's symptoms is C. Histoplasma capsulatum infection. The clinical presentation suggests an acute respiratory illness with systemic symptoms such as fever, chills, headache, and myalgia. Her recent hiking activity in Mississippi, an area endemic for Histoplasmosis, increases the likelihood of exposure to bird or bat droppings that contain the fungus Histoplasma capsulatum. Her symptoms started about a week after the potential exposure, fitting the incubation period for Histoplasmosis. The laboratory findings of a positive polysaccharide antigen test and the identification of macrophages filled with a dimorphic fungus with septate hyphae on bronchoalveolar lavage further support the diagnosis of Histoplasmosis. The presence of septate hyphae is characteristic of this fungus, differentiating it from other potential fungal causes." + }, + { + "input": "Question: A 22-year-old woman is brought to the emergency department because of a 2-day history of fever, intermittent rigors, and night sweats. She also has a 1-month history of progressive fatigue. Five weeks ago, she was hospitalized and received intravenous antibiotics for treatment of bacterial meningitis while visiting relatives in Guatemala. Her temperature is 39.4°C (102.9°F), pulse is 130/min, and blood pressure is 105/70 mm Hg. Examination shows pallor and scattered petechiae and ecchymoses. Laboratory studies show a hemoglobin concentration of 9.0 g/dL, a leukocyte count of 1,100/mm3 with 30% segmented neutrophils, and a platelet count of 20,000/mm3 . Blood cultures grow coagulase-negative staphylococci. The patient was most likely treated with which of the following antibiotics?\nOptions: A: Doxycycline\tB: Trimethoprim/sulfamethoxazole\tC: Linezolid\tD: Chloramphenicol\t\nThe right answer is D: Chloramphenicol.", + "output": "The patient's presentation of fever, fatigue, and the hematological findings of anemia, leukopenia, and thrombocytopenia, in addition to the growth of coagulase-negative staphylococci in blood cultures, are suggestive of secondary effects from antibiotic treatment. These symptoms and lab findings are consistent with drug-induced bone marrow suppression. Chloramphenicol is known for its potential to cause reversible or irreversible bone marrow suppression, leading to aplastic anemia and other blood dyscrasias. This could manifest as the observed cytopenias. Since the patient was previously treated for bacterial meningitis—a condition sometimes treated with chloramphenicol, especially outside of the United States—this antibiotic is the most likely cause of her current syndrome. Options A, B, and C typically do not cause such severe hematological abnormalities and are less commonly associated with treatment of bacterial meningitis." + }, + { + "input": "Question: A 27-year-old man presents to the emergency department. He was brought in by staff from the homeless shelter when they found him unresponsive. The patient is a known IV drug abuser but otherwise has an unknown past medical history. He currently attends a methadone clinic. His temperature is 99.5°F (37.5°C), blood pressure is 97/48 mmHg, pulse is 140/min, respirations are 29/min, and oxygen saturation is 98% on room air. Initial laboratory values are shown below.\n\nSerum:\nNa+: 139 mEq/L\nCl-: 100 mEq/L\nK+: 6.3 mEq/L\nHCO3-: 17 mEq/L\nGlucose: 589 mg/dL\n\nThe patient is given treatment. After treatment, his temperature is 99.5°F (37.5°C), blood pressure is 117/78 mmHg, pulse is 100/min, respirations are 23/min, and oxygen saturation is 98% on room air. His laboratory values are seen below.\n\nSerum:\nNa+: 139 mEq/L\nCl-: 100 mEq/L\nK+: 4.3 mEq/L\nHCO3-: 19 mEq/L\nGlucose: 90 mg/dL\n\nWhich of the following is the best next step in management?\nOptions: A: Insulin, potassium, IV fluids, and glucose\tB: IV fluids only\tC: Oral rehydration\tD: Supportive therapy and close monitoring\t\nThe right answer is A: Insulin, potassium, IV fluids, and glucose.", + "output": "The initial presentation of the patient with a high blood glucose level (589 mg/dL) indicates diabetic ketoacidosis (DKA), a common condition in patients with uncontrolled diabetes. The treatment for DKA includes insulin to lower blood glucose, IV fluids to rehydrate, and potassium to replace losses and prevent hypokalemia as insulin drives potassium into cells. Despite the glucose normalization (90 mg/dL), the patient still requires insulin to close the anion gap and resolve the acidosis indicated by a low bicarbonate (HCO3-) level. The inclusion of glucose in the treatment is crucial to prevent hypoglycemia as insulin continues to be administered. Hence, option A is the best next step in management because it addresses all aspects of DKA's ongoing treatment needs." + }, + { + "input": "Question: A 67-year-old man who was diagnosed with arthritis 16 years ago presents with right knee swelling and pain. His left knee was swollen a few weeks ago, but now with both joints affected, he has difficulty walking and feels frustrated. He also has back pain which makes it extremely difficult to move around and be active during the day. He says his pain significantly improves with rest. He also suffers from dandruff for which he uses special shampoos. Physical examination is notable for pitting of his nails. Which of the following is the most likely diagnosis?\nOptions: A: Psoriatic arthritis\tB: Arthritis mutilans\tC: Rheumatoid arthritis\tD: Mixed connective tissue disease\t\nThe right answer is A: Psoriatic arthritis.", + "output": "The presentation of arthritis with skin manifestations such as dandruff and pitted nails, alongside a fluctuating pattern of joint involvement, is highly indicative of psoriatic arthritis (PsA). PsA is a chronic inflammatory arthritis associated with the skin condition psoriasis, which often presents with skin scaling (dandruff-like) and nail changes like pitting. The resolution of pain with rest is more typical of PsA versus rheumatoid arthritis, which is generally characterized by persistent stiffness especially after rest. Arthritis mutilans is a severe, but less common form of PsA. Mixed connective tissue disease often presents with more systemic features rather than pitting nails and would not typically cause such improvement of pain with rest. Hence, psoriatic arthritis is the most likely diagnosis." + }, + { + "input": "Question: A 45-year-old G5P4105 presents to her gynecologist’s office with six months of increasingly heavy periods. She now soaks one super absorbent tampon every two hours for five days each cycle. Her cycles have also become irregular, with intermenstrual bleeding for the last two months. She now experiences significant dysmenorrhea, requiring 400 mg ibuprofen every four hours for the majority of each menses. In addition, she reports new onset mild dyspareunia with intercourse and a “heavy feeling” in her pelvis. She has also noticed increased urinary frequency but denies bowel changes. The patient has a past medical history of obesity and type II diabetes on metformin. Her last child was born four years ago, and she has had five spontaneous vaginal deliveries. At this office visit, temperature is 98.5°F (36.9°C), blood pressure is 137/84 mmHg, pulse is 87/min, and respirations are 14/min. Which of the following physical exam findings is most likely to be present in this patient?\nOptions: A: Globular 10-week sized uterus\tB: Adnexal mass\tC: Irregular 14-week sized uterus\tD: No remarkable physical exam finding\t\nThe right answer is A: Globular 10-week sized uterus.", + "output": "The patient's symptoms suggest the possibility of a common gynecological condition such as uterine fibroids, especially given her age and parity. Heavy and prolonged menstrual bleeding (menorrhagia), intermenstrual bleeding, dysmenorrhea, a sensation of pelvic heaviness, and increased urinary frequency are all symptoms consistent with fibroids. Fibroids are benign growths of the uterine muscle and commonly cause the uterus to enlarge. A \"globular\" shaped uterus implies a uniform enlargement typical of fibroids. Given these factors and the absence of other specific signs that would suggest an adnexal mass, malignancy, or irregular growth, a globular 10-week sized uterus (Option A) is the most likely finding on physical examination." + }, + { + "input": "Question: A 67-year-old man is hospitalized after several days of fever and increasing shortness of breath. Respiratory viral panel returns positive for influenza A and the patient is started on a standard five day course of oseltamivir. Despite therapy, his shortness of breath continues to worsen and his oxygen saturation decreases to the point where ICU transfer and intubation are required. Chest radiograph shows bilateral infiltrates. Clinical criteria are consistent with development of Acute Respiratory Distress Syndrome (ARDS). After several weeks of supportive therapy, the patient improves with lung function returning to near normal. Which of the following processes is involved in the regeneration of alveolar lining after damage to alveoli occurs?\nOptions: A: Sweeping of debris out of the alveoli by ciliated cells\tB: Excess mucus production by goblet cells\tC: Proliferation of surfactant-secreting cells\tD: Proliferation of club cells\t\nThe right answer is C: Proliferation of surfactant-secreting cells.", + "output": "The regeneration of alveolar lining following damage involves the proliferation of surfactant-secreting cells, known as type II pneumocytes or alveolar type II cells. These cells play a crucial role in repairing and regenerating the alveolar epithelium. After damage occurs, such as from ARDS, type II pneumocytes proliferate and differentiate into type I pneumocytes to restore the alveolar surface area necessary for gas exchange. They also secrete pulmonary surfactant to decrease surface tension within the alveoli, preventing alveolar collapse and further aiding in the breathing process. This regenerative ability is essential for recovery from alveolar damage. Options A, B, and D do not directly contribute to the regeneration of the alveolar lining." + }, + { + "input": "Question: A 17-year-old football player with no significant past medical history, social history, or family history presents to his pediatrician with itching in his groin. He says this started during summer workouts leading up to this season. He denies having any rash anywhere else on his body. The blood pressure is 123/78 mm Hg, pulse is 67/min, respiratory rate is 15/min, and temperature is 38.1°C (98.7°F). Physical examination reveals an erythematous, well-demarcated patch with peripheral scale on the left thigh, pubic region, and perineum. There is no apparent scrotal involvement with the rash. How can you confirm the suspected diagnosis?\nOptions: A: KOH examination of lesion scrapings\tB: Nikolsky's sign on physical exam\tC: Gram stain of skin scrapings\tD: AFB stain of skin scrapings\t\nThe right answer is A: KOH examination of lesion scrapings.", + "output": "The presentation of an erythematous, well-demarcated patch with peripheral scaling in the groin of a young athlete suggests a fungal infection such as tinea cruris (\"jock itch\"), which is common in athletes due to sweating and friction. The confirmation of a superficial fungal infection typically involves microscopic examination after a potassium hydroxide (KOH) preparation of lesion scrapings. The KOH dissolves human skin cells, leaving only fungal cells visible, allowing for the identification of dermatophytes (fungi that affect the skin). Nikolsky's sign, Gram stain, and AFB stain are not appropriate for this suspected diagnosis. Nikolsky's sign is used for blistering disorders, Gram stain for bacterial infections, and AFB stain for mycobacterial infections." + }, + { + "input": "Question: An otherwise healthy 26-year-old man comes to the physician for medication counseling after recently being diagnosed with schizophrenia. Risperidone therapy is initiated. This patient is at increased risk for which of the following adverse effects?\nOptions: A: Agranulocytosis\tB: Shortened QT interval\tC: Gynecomastia\tD: Weight loss\t\nThe right answer is C: Gynecomastia.", + "output": "Risperidone therapy, as an atypical antipsychotic, is associated with a risk of hyperprolactinemia due to its antagonistic effects on dopamine D2 receptors in the tuberoinfundibular pathway. Elevated prolactin levels can lead to gynecomastia in men. This is when the breast tissue enlarges, which is a recognized adverse effect of risperidone. Agranulocytosis is more typically associated with typical antipsychotics, such as clozapine. Antipsychotics can prolong the QT interval, not shorten it. Lastly, these medications are more likely to cause weight gain, not loss, due to their effects on appetite and metabolism." + }, + { + "input": "Question: A 4-year-old boy is brought to the emergency department because of severe abdominal pain and bilious vomiting for 6 hours. He has not had bowel movements in the past 24 hours. He appears ill. His temperature is 37.8°C (100°F) and pulse is 122/min. Examination shows a distended abdomen. There is tenderness to palpation in the lower abdomen; guarding and rebound tenderness are present. Bowel sounds are decreased. An x-ray of the abdomen shows dilated loops of bowel. He has been accompanied by his 14-year-old brother. The surgeon recommends an emergency laparotomy. The parents are away visiting friends and cannot be reached. Which of the following is the most appropriate next best step in management?\nOptions: A: Get consent from the patient's brother\tB: Get consent from the patient\tC: Perform emergency laparotomy\tD: Delay surgery until parental consent\t\nThe right answer is C: Perform emergency laparotomy.", + "output": "The most appropriate next step in management is option C, perform emergency laparotomy. The child is presenting with signs suggestive of intestinal obstruction, which can become life-threatening if not promptly treated. He has severe abdominal pain, bilious vomiting, no bowel movement for 24 hours, fever, tachycardia, abdominal distension, tenderness, guarding, rebound tenderness, and evidence of dilated bowel loops on x-ray. These clinical findings strongly suggest an acute surgical abdomen that requires immediate attention. In such emergent situations, where a delay in treatment can lead to significant morbidity or mortality, medical professionals are ethically and legally allowed to proceed with life-saving treatment without explicit parental consent. The 14-year-old brother is not legally able to provide consent and the patient is obviously too young to give informed consent. Therefore, initiating the emergency surgery is justified to prevent deterioration of the child’s condition." + }, + { + "input": "Question: A 3-week-old male newborn is brought to the hospital because of poor weight gain since birth. He was born at 38 weeks' gestation via normal vaginal delivery. He weighed 3005 g (6 lb, 10 oz) at birth and currently weighs 2835 g (6 lb, 4 oz). He has been latching on and breastfeeding well since birth. His mother has a history of Graves' disease and underwent near-total thyroidectomy in the second trimester of her pregnancy after her symptoms could not be controlled with antithyroid drugs. She is currently receiving L-thyroxine therapy. The patient's temperature is 38.9°C (102°F), pulse is 176/min, and respirations are 42/min. He appears irritable. Examination shows a diaphoretic infant with a paucity of subcutaneous fat. There is swelling of the neck at the midline. Which of the following is the most likely cause?\nOptions: A: Transplacental passage of thyroid peroxidase antibodies\tB: Transplacental passage of TSH receptor antibodies\tC: Transplacental viral infection\tD: Opiate use in the mother\t\nThe right answer is B: Transplacental passage of TSH receptor antibodies.", + "output": "The most likely cause of the newborn's symptoms is B. Transplacental passage of TSH receptor antibodies. Given that the mother has a history of Graves' disease, an autoimmune condition where the body produces antibodies against the thyroid-stimulating hormone (TSH) receptor, it is plausible that these antibodies have been transferred to the fetus transplacentally. These antibodies can stimulate the fetal thyroid gland leading to hyperthyroidism in the newborn, which can account for the poor weight gain, fever, tachycardia, and irritability. The neck swelling may indicate an enlarged thyroid gland or goiter. The other options are less likely given the specific maternal history and clinical presentation of the neonate." + }, + { + "input": "Question: A newborn is found to be extremely cyanotic immediately after birth. He then develops progressive respiratory failure and is admitted to the neonatal ICU. A single loud S2 heart sound is appreciated as well as a machine-like murmur at the left upper sternal border. Radiography shows an enlarged \"egg-shaped\" heart. The newborn is then taken for a atrial septostomy to alleviate the condition pending definitive surgical correction. Which of the following is the most likely cause of this newborn's condition?\nOptions: A: Coarctation of the aorta\tB: Persistent truncus arteriosus\tC: Transposition of great vessels\tD: Tricuspid atresia\t\nThe right answer is C: Transposition of great vessels.", + "output": "The most likely cause of this newborn's condition, given the presentation, is transposition of the great vessels (Option C). The findings of severe cyanosis immediately after birth and a single loud S2 heart sound are indicative of this diagnosis. In transposition of the great vessels, the aorta and pulmonary artery are reversed, leading to the circulation of deoxygenated blood within the systemic circulation, thereby causing cyanosis. The machine-like murmur can be associated with a patent ductus arteriosus which often coexists with this condition, providing a shunt that can somewhat alleviate cyanosis. The radiography showing an \"egg-shaped\" heart is also consistent with this condition. Atrial septostomy is performed to increase the mixing of oxygenated and deoxygenated blood at the atrial level, temporarily relieving cyanosis until definitive surgery (arterial switch procedure) can be done." + }, + { + "input": "Question: A 54-year-old male makes an appointment with his primary care physician due to chronic fatigue that has left him unable to hike or do other physically demanding activities with his friends. He has well-controlled hypertension and diabetes but has otherwise been healthy. He does not smoke but drinks socially with his friends. Physical exam reveals enlarged nodules that are not tender to palpation. A biopsy is obtained showing a characteristic change in chromosome 18. The regulation of which of the following proteins will be most abnormal in this patient?\nOptions: A: Caspase-9\tB: CD15\tC: Cyclin-dependent kinase 4\tD: Ras pathway transcription factors\t\nThe right answer is A: Caspase-9.", + "output": "A diagnosis of multiple endocrine neoplasia type 1 (MEN1), caused by mutations in the MEN1 gene which encodes the protein menin (located on chromosome 18), is suggested by the presentation and biopsy showing characteristic chromosomal changes. Menin regulates several proteins and pathways, including those involved in cell cycle control and apoptosis. Caspase-9 is a key protein in the intrinsic apoptotic pathway. Given the regulatory role of menin on apoptosis, a mutation in the MEN1 gene would lead to abnormal regulation of caspase-9. The other options (B: CD15, C: Cyclin-dependent kinase 4, D: Ras pathway transcription factors) are not directly regulated by menin." + }, + { + "input": "Question: A 50-year-old man visits his physician after 20 years of not seeking any medical care. He is concerned about his health after a colleague recently had a heart attack. The patient has no active complaints and says he feels healthy; however, he does not exercise regularly and lives a sedentary lifestyle. He is employed as an administrative position at a local college, and is seated at a desk most of the day. His father had a heart attack at age 54 and his mother is still alive with no health concerns. He does not smoke, only drinks socially, and does not use drugs. Today, his blood pressure is 130/90 mm Hg, pulse is 84/min, and respiratory rate is 14/min. Physical examination reveals an obese male with no significant findings. An ECG shows no abnormalities, and laboratory testing shows the following:\nLaboratory test\nSerum glucose (fasting) 105 mg/dL\nSerum electrolytes \nSodium 142 mEq/L\nPotassium 3.9 mEq/L\nChloride 101 mEq/L\nSerum creatinine 0.8 mg/dl\nBlood urea nitrogen 10 mg/dl\nCholesterol, total 250 mg/dL\nHDL-cholesterol 35 mg/dL\nLDL-cholesterol 186 mg/dL\nTriglycerides 170 mg/dL\nUrinalysis \nGlucose negative\nKetones negative\nLeucocytes negative\nNitrites negative \nRed blood cells (RBC) negative \nCasts negative \nWhich of the following lab abnormalities in this patient is an indication for treatment?\nOptions: A: Blood pressure reading\tB: Patient’s weight\tC: High LDL-cholesterol\tD: Serum glucose level\t\nThe right answer is C: High LDL-cholesterol.", + "output": "The patient's LDL-cholesterol level is 186 mg/dL, which is above the recommended threshold of 100 mg/dL for optimal cardiovascular health. Considering his family history of heart disease with his father having had a heart attack at a relatively young age, his sedentary lifestyle, and obesity, he is at an elevated risk for developing cardiovascular diseases himself. The high level of LDL-cholesterol can lead to the buildup of plaque in the arteries, increasing the risk of heart attacks and strokes. His blood pressure is at the higher end of the \"pre-hypertension\" range, but not yet in the range where antihypertensive treatment is imperative. His weight is a risk factor, but not a \"lab abnormality.\" Therefore, the most pressing laboratory indication for treatment is the high LDL-cholesterol level (Option C), which warrants dietary modification, increased physical activity, and potentially lipid-lowering medication to reduce cardiovascular risk." + }, + { + "input": "Question: A 32-year-old woman comes to the physician because of a 3-week history of intermittent loose stools and a 1.2-kg (2.6-lb) weight loss. She immigrated to the US from Uganda 6 weeks ago. Abdominal examination shows diffuse tenderness with no guarding or rebound. The liver is firm and palpable 3 cm below the right costal margin, and the spleen is palpable just below the left costal margin. Her leukocyte count is 12,800/mm3 (12% eosinophils). Stool culture shows several oval-shaped eggs with lateral spines. Microscopic examination of a liver biopsy specimen shows granulomatous inflammation with periportal fibrosis. Exposure to which of the following is most likely to have played a role in the development of this patient's symptoms?\nOptions: A: Undercooked pork meat\tB: Undercooked fish meat\tC: Dog feces\tD: Freshwater snails\t\nThe right answer is D: Freshwater snails.", + "output": "The patient presents with symptoms suggestive of a parasitic infection, particularly Schistosomiasis, also known as snail fever. The presence of oval-shaped eggs with lateral spines in her stool is characteristic of the Schistosoma species, which use freshwater snails as an intermediate host. The immigration from Uganda, a region where Schistosomiasis is endemic, further supports this diagnosis. The granulomatous inflammation with periportal fibrosis observed in the liver biopsy is indicative of the chronic stage of the disease, which can lead to hepatosplenomegaly, as seen with palpable liver and spleen. Exposure to freshwater snails carrying Schistosoma larvae while in Uganda is the most likely cause of her condition." + }, + { + "input": "Question: A 2-day-old male newborn is brought to the physician because of yellowing of the skin and sclerae for 16 hours. He had previously been well. He was born at 38 weeks' gestation via uncomplicated vaginal delivery and weighed 3.1 kg (6 lb 13 oz). The mother has no medical insurance and did not receive prenatal care. The newborn's 4-year-old brother has sickle cell disease. Examination shows jaundice. The abdomen is mildly distended. The liver is palpated 1 cm below the right costal margin and the spleen tip is palpated just below the left costal margin. Laboratory studies show:\nHemoglobin 11 g/dL\nReticulocytes 9%\nLeukocytes 9,100/mm3\nPlatelets 244,000/mm3\nMaternal blood group 0, Rh-negative\nAnti-Rh antibody titer positive\nFetal blood group B, Rh-negative\nSerum\nBilirubin, total 11.3 mg/dL\nDirect 0.3 mg/dL\nWhich of the following is the most likely cause of this patient's condition?\"\nOptions: A: RBC sickling\tB: Anti-D antibodies\tC: Biliary duct malformation\tD: Anti-B antibodies\t\nThe right answer is D: Anti-B antibodies.", + "output": "The most likely cause of this newborn's jaundice is D. Anti-B antibodies. This is due to ABO incompatibility, where the mother's blood group is O and the baby's is B. People with blood group O have both anti-A and anti-B antibodies, which can cross the placenta and cause hemolysis of the newborn's B-type red blood cells, leading to jaundice. Additionally, the newborn has an elevated reticulocyte count, which indicates increased red blood cell production as a response to hemolysis. The absence of maternal Rh antibodies and the baby being Rh-negative rules out Rh incompatibility as a cause. Other options like RBC sickling and biliary duct malformation are less likely since the symptoms and laboratory findings do not match those conditions." + }, + { + "input": "Question: A 29-year-old man comes in for evaluation of infertility. He has been trying to conceive for over 2 years with his wife and previous evaluation of his wife's fertility revealed no abnormalities. Physical exam reveals a tall man with long extremities, sparse body hair, gynecomastia, and small testes. Laboratory studies reveal increased serum follicle-stimulating hormone concentration and an increased estradiol:testosterone ratio. Genetic studies reveal a cytogenetic abnormality. If this abnormality was inherited from the patient's father, at which stage of spermatogenesis did this error most likely occur?\nOptions: A: Primary spermatocyte\tB: Secondary spermatocyte\tC: Spermatid\tD: Spermatozoon\t\nThe right answer is A: Primary spermatocyte.", + "output": "The patient's presentation with tall stature, long extremities, sparse body hair, gynecomastia, and small testes, combined with increased FSH and estradiol:testosterone ratio, is highly suggestive of Klinefelter syndrome (47,XXY karyotype). In this condition, the extra X chromosome is typically the result of nondisjunction—an error in chromosome separation—during meiosis. Since the genetic anomaly in Klinefelter syndrome can be inherited from the father, the nondisjunction error would have occurred in spermatogenesis at the primary spermatocyte stage, which is meiosis I. Meiosis I is where homologous chromosomes are separated, and so it is the phase during which nondisjunction leading to sex chromosome aneuploidies like Klinefelter's would most likely take place." + }, + { + "input": "Question: A 40-year-old man presents to the physician with progressive weight loss for the last 3 months. He also says he frequently sweats profusely at night and has a recurring low-grade fever, for which he takes acetaminophen. The patient denies any symptoms like cough, breathlessness, or gastrointestinal symptoms. His temperature is 37.1ºC (98.8ºF), pulse is 76/min, blood pressure is 116/78 mm Hg, and respiratory rate is 13/min. On physical examination, he has generalized pallor. Bilateral cervical lymphadenopathy is present. Examination of his abdomen reveals non-tender hepatosplenomegaly in the right upper quadrant. Laboratory evaluation confirms the diagnosis of Hodgkin’s lymphoma. Which of the following viral infections is most likely to have played a role in the pathogenesis of this patient’s malignancy?\nOptions: A: Epstein-Barr virus\tB: Human T-cell leukemia virus type 1\tC: Human herpesvirus-8\tD: Human papillomavirus type 16\t\nThe right answer is A: Epstein-Barr virus.", + "output": "The Epstein-Barr virus (EBV) is most likely to have played a role in the pathogenesis of this patient's Hodgkin's lymphoma. EBV is a common herpesvirus that often causes infectious mononucleosis and is associated with various malignancies, including Burkitt's lymphoma, nasopharyngeal carcinoma, and Hodgkin's lymphoma. Research suggests that EBV can be found in the Reed-Sternberg cells, the characteristic cells of Hodgkin's lymphoma, implying a causal relationship. Other viruses listed (HTLV-1, HHV-8, HPV-16) are also associated with different types of cancers but not as strongly with Hodgkin's lymphoma as EBV." + }, + { + "input": "Question: A 33-year-old nurse is referred to an infectious disease specialist after she exhibited a PPD skin test with 17 mm of induration. She denies any cough, shortness of breath, hemoptysis, weight loss, fatigue, fevers, or night sweats over the last several months. Her temperature is 97.0°F (36.1°C), blood pressure is 120/81 mmHg, pulse is 82/min, respirations are 15/min, and oxygen saturation is 98% on room air. An initial chest radiograph is unremarkable. Which of the following is the most appropriate management of this patient?\nOptions: A: Isoniazid\tB: No management indicated\tC: Repeat PPD in 1 week\tD: Rifampin, isoniazid, pyrazinamide, and ethambutol\t\nThe right answer is A: Isoniazid.", + "output": "This patient, a nurse, likely has a latent tuberculosis infection (LTBI) given the significant reaction to the PPD test (17 mm of induration) without any active symptoms or radiographic evidence of tuberculosis (TB). The size of the induration, at 17 mm, is well above the 15 mm threshold for considering treatment for LTBI in individuals with no risk factors, and given her occupation in healthcare, she's at a higher risk for TB exposure and subsequent reactivation. The most appropriate management is to initiate treatment with isoniazid for LTBI to prevent the development of active TB. The choice of isoniazid (option A) is based on the current guidelines for the treatment of LTBI. The need for other medications (option D) would arise only if she had active TB, which is not supported by her lack of symptoms and normal chest X-ray. No management (option B) would be inappropriate due to her increased risk and positive PPD. Repeating the PPD (option C) is unnecessary as the diagnosis of LTBI is already established." + }, + { + "input": "Question: An 81-year-old woman is brought to the emergency room by her son after witnessing the patient fall and hit her head. The son reports that the patient was in her usual state of health until she complained of chest palpitations. This startled her while she was climbing down the stairs and lead to a fall. Past medical history is significant for hypertension and atrial fibrillation. Medications are lisinopril, metoprolol, and warfarin. Temperature is 99°F (37.2°C), blood pressure is 152/96 mmHg, pulse is 60/min, respirations are 12/min, and pulse oximetry is 98% on room air. On physical examination, she is disoriented and at times difficult to arouse, the left pupil is 6 mm and non-reactive to light, and the right pupil is 2 mm and reactive to light. A right-sided visual field defect is appreciated on visual field testing. There is 1/5 strength on the right upper and lower extremity; as well as 5/5 strength in the left upper and lower extremity. A computerized tomography (CT) scan of the head is shown. Which of the following most likely explains this patient’s symptoms?\nOptions: A: Herniation of the uncus\tB: Herniation of the cingulate gyrus\tC: Occlusion of the basilar artery\tD: Occlusion of the anterior spinal artery\t\nThe right answer is A: Herniation of the uncus.", + "output": "The patient's clinical presentation is consistent with an intracranial hemorrhage causing increased intracranial pressure and subsequent herniation of the uncus. The symptoms of unilateral dilated pupil, a right-sided visual field defect, and right-sided hemiparesis all suggest compression of the oculomotor nerve and ipsilateral cerebral peduncle by a herniated uncus (part of the temporal lobe). These signs also are indicative of compression against the midbrain, which would not occur with cingulate gyrus herniation, seen typically in subfalcine herniation, which does not affect the oculomotor nerve in this manner. Additionally, the occlusion of major arteries like the basilar or anterior spinal would present with different neurologic deficits and would not account for the unilateral pupillary response and localized motor findings. Given her history of anticoagulant use for atrial fibrillation, the risk of bleeding is increased, making a hemorrhagic stroke leading to herniation more likely than an acute arterial occlusion." + }, + { + "input": "Question: A 65-year-old man presents to his primary care physician for a change in his behavior over the past few months. Initially, the patient was noted to be behaving inappropriately including using foul language and grabbing people unexpectedly. This has progressed to a worsening of his memory and trouble caring for himself. His temperature is 98.1°F (36.7°C), blood pressure is 162/103 mmHg, pulse is 83/min, respirations are 13/min, and oxygen saturation is 98% on room air. Physical exam is notable for an elderly man who laughs inappropriately at times and who is a poor historian. When he is engaged in conversation, he exhibits word finding difficulty and is rather inattentive. Which of the following is the most likely diagnosis?\nOptions: A: Alzheimer dementia\tB: Frontotemporal dementia\tC: Kluver-Bucy syndrome\tD: Vascular dementia\t\nThe right answer is B: Frontotemporal dementia.", + "output": "Frontotemporal dementia (FTD) is the most likely diagnosis for this patient, considering the onset of socially inappropriate behavior, changes in personality, and language difficulties, which are hallmark features of FTD. This type of dementia is characterized by progressive damage to the frontal and temporal lobes of the brain, areas responsible for behavior, language, and emotion regulation. Unlike Alzheimer's disease, which typically begins with memory loss, FTD often starts with changes in social and executive functions. The patient's lack of personal insight, inattention, and inappropriate laughter also point towards FTD. Moreover, Alzheimer's and Vascular dementia usually do not present primarily with behavioral changes, and Kluver-Bucy syndrome is rare and would involve additional symptoms like hyperorality and hypersexuality, which are not noted here." + }, + { + "input": "Question: A 71-year-old man comes to the physician for a routine visit. His psychiatric history is significant for major depression with psychotic features. He was diagnosed after his wife died from pancreatic cancer four years earlier. Three years ago, he deliberately overdosed on sleeping pills. Today, he feels well but still wakes up frequently at night and cannot sleep through the night. He lives alone in his apartment and has no children. He does not possess a firearm at home but has access to weapons at a local hunting club. The patient currently denies suicidal thoughts or thoughts of self-harm. He takes sertraline and olanzapine. He does not smoke or drink alcohol. Which of the following is the strongest risk factor for suicide in this patient?\nOptions: A: Previous suicide attempt\tB: History of psychosis\tC: Sleeping disorder\tD: Age\n\"\t\nThe right answer is A: Previous suicide attempt.", + "output": "The strongest risk factor for suicide in this patient is A. Previous suicide attempt. A history of a previous suicide attempt is one of the most significant predictors of future suicide risk. Although other factors such as age, history of psychosis, and sleeping disorders do contribute to the overall risk profile for suicide, none are as strong as a previous attempt. Additionally, the patient's history of major depression with psychotic features and the significant life stressor of his wife's death may further elevate risk, but the actual attempt he made three years ago is the most critical red flag for future suicide risk. Therefore, the prior act of intentionally overdosing on sleeping pills is the single most potent risk factor in this scenario." + }, + { + "input": "Question: A 55-year-old man presents to a rheumatologist with bilateral wrist and knee pain that has been present for the last 10 years. The patient was a professional athlete who retired at age 50. He also notes morning stiffness in his joints that lasts approx. 50 minutes. He denies joint swelling, color changes in his digits, dryness of mucous membranes, and constitutional symptoms such as fever and chills. On physical exam, the physician notes several non-tender nodules on the left 1st, 3rd, and 4th distal interphalangeal joints. There is also tenderness with palpation at the base of both thumbs. The patient’s knees are enlarged, bilaterally, and he has pain and crepitus on passive range of motion. Which of the following is the most likely diagnosis?\nOptions: A: Ankylosing spondylitis\tB: Gout\tC: Osteoarthritis\tD: Fibromyalgia\t\nThe right answer is C: Osteoarthritis.", + "output": "The most likely diagnosis for this patient, considering his clinical presentation, is osteoarthritis (C). Osteoarthritis is consistent with his history of being a professional athlete, which often leads to joint wear and tear over time. Key features supporting this diagnosis include his age, the chronic nature of his symptoms, morning stiffness that lasts less than an hour, and the presence of non-tender nodules (likely Heberden's nodes) on his distal interphalangeal joints, as well as tenderness at the base of the thumbs. Enlargement of the knees with pain and crepitus on movement further indicates osteoarthritic joint changes rather than other listed conditions like ankylosing spondylitis, gout, or fibromyalgia." + }, + { + "input": "Question: A previously healthy 19-year-old man comes to the physician with right-sided, dull, throbbing scrotal pain for 12 hours. He has also had a burning sensation on urination and increased urinary frequency for the past 3 days. He has not had urethral discharge. He is sexually active with one female partner and does not use condoms. Vital signs are within normal limits. Physical examination shows a tender right testicle; lifting it provides relief. The penis appears normal, with no discharge at the meatus. Laboratory studies show a normal complete blood count; urinalysis shows 3 WBC/hpf. A Gram stain of a urethral swab shows polymorphonuclear leukocytes but no organisms. Testicular ultrasound shows increased blood flow to the right testicle compared to the left. Which of the following is the most likely cause of this patient's symptoms?\nOptions: A: Neisseria gonorrhoeae infection\tB: Testicular tumor\tC: Chlamydia trachomatis infection\tD: Varicocele\t\nThe right answer is C: Chlamydia trachomatis infection.", + "output": "The patient's presentation of scrotal pain, urinary frequency, and burning sensation during urination, coupled with no visible discharge and relief of pain upon lifting the testicle (Prehn's sign), suggests epididymitis rather than gonococcal urethritis or a testicular tumor, which would likely present with different symptoms. The presence of WBCs in the urine and polymorphonuclear leukocytes on a Gram stain without organisms suggest an infection with an organism not visible on Gram stain, pointing away from Neisseria gonorrhoeae. Chlamydia trachomatis, which is not visible on Gram stain, often presents with these symptoms in a sexually active young male and is the most common cause of epididymitis in this demographic, making option C the most likely cause. Increased blood flow on ultrasound further supports an inflammatory process, consistent with chlamydial infection, rather than a varicocele or tumor." + }, + { + "input": "Question: A previously healthy 18-month-old boy is brought to the physician by his parents for evaluation of an abdominal mass noticed a few days ago. He appears markedly pale and lethargic. Examination shows a 6-cm, nonmobile mass in the left upper quadrant that crosses the midline. 24-hour urine collection shows elevated homovanillic acid and vanillylmandelic acid. Further evaluation including biopsy confirms the diagnosis of intermediate-risk neuroblastoma. The physician recommends the established standard treatment, which is initiation of neoadjuvant chemotherapy followed by surgical resection, if possible. After a thorough discussion of the risks and benefits of chemotherapy, the likelihood of unresectability without neoadjuvant treatment, and the prognosis without it, the patient's parents steadily refuse chemotherapy because they do not want their son to suffer the side effects. They prefer to take their son home for supportive care only. Which of the following is the most appropriate action by the physician?\nOptions: A: Help the parents to arrange supportive care at home\tB: Refer the patient to another oncologist\tC: Recommend for parents to take 2 weeks to think about decision\tD: Seek a court order for neoadjuvant chemotherapy\t\nThe right answer is D: Seek a court order for neoadjuvant chemotherapy.", + "output": "Choosing option D, seeking a court order for neoadjuvant chemotherapy, is the most appropriate action for the physician in this context. This is based on the principle of beneficence, where the healthcare provider has an obligation to act in the best interest of the patient. An intermediate-risk neuroblastoma in an 18-month-old patient has a potentially favorable prognosis with appropriate treatment, which includes neoadjuvant chemotherapy followed by surgical resection. By refusing chemotherapy, the parents are denying their child the standard care that could significantly improve his chances of recovery and survival. In such a case, where the decision by the parents poses a risk to the child's life and well-being, the physician has a duty to protect the child's interests. Seeking a court order is a measure of last resort when all attempts to reach an understanding with the parents have failed, and it is used to ensure that the child receives the necessary medical treatment to prevent harm." + }, + { + "input": "Question: A 65-year-old man with metastatic lung cancer has been experiencing severe, unremitting pain. He has required escalating doses of oral morphine, but is now having dose limiting side-effects. His pain management team recommends using a medication that can reduce his opioid need through interaction with the NMDA-receptor. Which of the following was the most likely recommended agent?\nOptions: A: Propofol\tB: Ketamine\tC: Fentanyl\tD: Midazolam\t\nThe right answer is B: Ketamine.", + "output": "Ketamine is an NMDA-receptor antagonist, which means it can help reduce opioid requirements by modulating the pain pathway differently from opioids. This can be effective in cases of severe pain where escalating doses of opioids lead to troubling side effects. Fentanyl is a potent opioid and would not reduce opioid need; it can actually exacerbate side effects. Propofol and midazolam are sedatives used for anesthesia or procedural sedation and do not interact with the NMDA receptor in a way that would reduce the need for opioids in pain management. Therefore, Ketamine (Option B) is the most likely recommended agent." + }, + { + "input": "Question: Please refer to the summary above to answer this question\nThe authors of the study have decided to conduct a follow-up analysis on their data. They decide to stratify their results by CD4+T-lymphocyte count at the time of diagnosis. Among patients with CD4+ cell counts below 200/mm3, cART adherence was a significant predictor of DLBCL risk (RR = 0.52, p = 0.01). However, among patients with CD4+ cell counts above 200/mm3, no relationship was found between DLBCL risk and cART adherence (RR = 0.96, p = 0.36). Which of the following explains for the difference observed between the two strata?\"\nOptions: A: Poor generalizability\tB: Stratified sampling\tC: Random error\tD: Effect modification\t\nThe right answer is D: Effect modification.", + "output": "Answer D (Effect modification) explains the observed difference between the two strata because it suggests that the effect of cART adherence on DLBCL risk is modified by the level of CD4+ T-lymphocyte count. The significant reduction in DLBCL risk with better cART adherence among patients with lower CD4+ counts (<200/mm3) indicates that when the immune system is more severely compromised, adherence to cART has a more pronounced beneficial effect on reducing lymphoma risk. In contrast, among patients with higher CD4+ counts (>200/mm3), the immune system is less compromised, possibly diminishing the relative impact of cART adherence on DLBCL risk." + }, + { + "input": "Question: A group of researchers conducted a study to determine whether there is an association between folic acid supplementation before pregnancy and autism spectrum disorder (ASD) in offspring. The researchers retrospectively surveyed 200 mothers with children diagnosed with ASD during the first 4 years of life and 200 mothers with healthy children. All participants were interviewed about their prenatal consumption of folic acid using standardized questionnaires. A 94% response rate was obtained from the surveys. The study ultimately found that folic acid supplementation was associated with lower rates of ASD in offspring (OR = 0.3, p < 0.01). Which of the following type of bias is most likely to have influenced these results?\nOptions: A: Interviewer bias\tB: Latency period\tC: Recall bias\tD: Survival bias\t\nThe right answer is C: Recall bias.", + "output": "The most likely type of bias that could have influenced these results is C, recall bias. Recall bias occurs when participants do not remember past events or exposures accurately. In this retrospective study, mothers were asked to remember their prenatal consumption of folic acid, which could have occurred many years before. Mothers of children with ASD might remember events differently, perhaps more meticulously due to the search for potential causes or contributing factors to their child's condition, compared to mothers of healthy children. This differential in memory accuracy can lead to an under- or overestimation of the association between folic acid supplementation and ASD." + }, + { + "input": "Question: A 35-year-old female is brought to the emergency department after being found unconscious. The patient is found to have a blood glucose level of 35 mg/dL. Hgb A1c was found to be 5.1%. C-peptide level was found to be decreased. The patient returned to her baseline after glucose replacement. She states that she has never had an episode like this before. She has no significant past medical history but reports a family history of diabetes in her mother. She states that she has been undergoing a difficult divorce over the past few months. What is the likely cause of this patient's condition?\nOptions: A: Insulinoma\tB: Factitious disorder\tC: Hypothyroidism\tD: Somatization disorder\t\nThe right answer is B: Factitious disorder.", + "output": "Factitious disorder seems to be the likely cause of this patient's hypoglycemia based on the given scenario. The normal Hgb A1c indicates this is not a chronic hyperglycemic state as seen with uncontrolled diabetes. The low C-peptide level suggests that the insulin in her system is exogenous, since C-peptide is released in equal amounts to insulin by the pancreas and would be elevated with endogenous hyperinsulinemia, such as with an insulinoma. Furthermore, the stress of a difficult divorce could trigger factitious disorder, where the patient self-administers insulin or other hypoglycemic agents to produce a medical condition. The medical history and stressor suggest a psychological cause rather than an organic one." + }, + { + "input": "Question: A 48-year-old man seeks evaluation at a clinic with a complaint of breathlessness for the past couple of weeks. He says that he finds it difficult to walk a few blocks and has to rest. He also complains of a cough for the past 3 months, which is dry and hacking in nature. The medical history is relevant for an idiopathic arrhythmia for which he takes amiodarone daily. He is a non-smoker and does not drink alcohol. He denies any use of illicit drugs. The vital signs are as follows: heart rate 98/min, respiratory rate 16/min, temperature 37.6°C (99.68°F), and blood pressure 132/70 mm Hg. The physical examination is significant for inspiratory crackles over the lung bases. An echocardiogram shows a normal ejection fraction. A chest radiograph is performed and shown below. Which of the following findings will most likely be noted on spirometry?\nOptions: A: Decreased FEV1 and normal FVC\tB: Decreased FEV1 and FVC with decreased FEV1/FVC ratio\tC: Decreased FEV1 and FVC with normal FEV1/FVC ratio\tD: Increased FEV1 and FVC\t\nThe right answer is C: Decreased FEV1 and FVC with normal FEV1/FVC ratio.", + "output": "The patient has symptoms of breathlessness, a persistent dry cough, and inspiratory crackles, which can be indicative of interstitial lung disease. Since he is taking amiodarone, a known side effect of this medication is amiodarone-induced pulmonary toxicity (AIPT), which can present with these clinical features. AIPT often manifests as interstitial pneumonitis or pulmonary fibrosis. On spirometry, interstitial lung diseases typically show a restrictive pattern characterized by reduced forced vital capacity (FVC) and forced expiratory volume in one second (FEV1), with a preserved or increased FEV1/FVC ratio as the reduction is proportionate. Therefore, choice C (Decreased FEV1 and FVC with normal FEV1/FVC ratio) is the most likely spirometry finding, reflecting a restrictive lung disease pattern potentially caused by amiodarone." + }, + { + "input": "Question: A 59-year-old man comes to the emergency department because of excruciating left knee pain for 4 days. He underwent a total knee arthroplasty of his left knee joint 4 months ago. He has hypertension and osteoarthritis. Current medications include glucosamine, amlodipine, and meloxicam. His temperature is 38.1°C (100.6°F), pulse is 97/min, and blood pressure is 118/71 mm Hg. Examination shows a tender, swollen left knee joint; range of motion is limited by pain. Analysis of the synovial fluid confirms septic arthritis, and the prosthesis is removed. Which of the following is the most likely causal organism?\nOptions: A: Staphylococcus epidermidis\tB: Escherichia coli\tC: Staphylococcus aureus\tD: Pseudomonas aeruginosa\t\nThe right answer is A: Staphylococcus epidermidis.", + "output": "The most likely causal organism for septic arthritis in a patient who has had recent joint replacement surgery is Staphylococcus epidermidis. This bacterium is part of the skin's normal flora and is a common contaminant during surgery. Despite being less virulent than Staphylococcus aureus, it is frequently associated with infections of indwelling devices, such as prosthetic joints, due to its ability to form biofilms on their surfaces. Additionally, the late presentation of infection, four months after surgery, is more characteristic of a low-virulence organism like S. epidermidis, as opposed to the quicker onset that might be seen with more virulent bacteria like S. aureus or Gram-negative rods." + }, + { + "input": "Question: A 32-year-old man comes to the physician for a pre-employment examination. He recently traveled to Guatemala. He feels well but has not seen a physician in several years, and his immunization records are unavailable. Physical examination shows no abnormalities. Serum studies show:\nAnti-HAV IgM Positive\nAnti-HAV IgG Negative\nHBsAg Negative\nAnti-HBs Positive\nHBcAg Negative\nAnti-HBc Negative\nHBeAg Negative\nAnti-HBe Negative\nAnti-HCV Negative\nWhich of the following best explains this patient's laboratory findings?\"\nOptions: A: Active hepatitis A infection\tB: Previous hepatitis A infection\tC: Chronic hepatitis B infection\tD: Chronic hepatitis C infection\t\nThe right answer is A: Active hepatitis A infection.", + "output": "The laboratory findings for this patient indicate an active hepatitis A infection. The reasons for this conclusion are as follows: The presence of Anti-HAV IgM antibodies suggests a recent infection with hepatitis A virus, as IgM is the first antibody produced in response to an acute infection. The absence of Anti-HAV IgG indicates that the patient has not developed immunity from a past infection or been vaccinated, as IgG antibodies would be present in these cases. Moreover, the negative results for HBsAg, Anti-HBc, HBeAg, Anti-HBe, and Anti-HCV rule out current infections with hepatitis B and C. Anti-HBs positivity suggests previous immunization against hepatitis B. Therefore, the best explanation for this patient's laboratory findings is an active hepatitis A infection (Option A)." + }, + { + "input": "Question: A group of researchers decided to explore whether the estimates of incidence and prevalence rates of systemic lupus erythematosus (SLE) were influenced by the number of years it took to examine administrative data. These estimates were not only based on hospital discharges, but also on physician billing codes. For study purposes, the researchers labeled incident cases at the initial occurrence of SLE diagnosis in the hospital database, while prevalent cases were those that were coded as harboring SLE at any time, with patients maintaining their diagnosis until death. Which statement is true regarding the relationship between incidence and prevalence rates during the time period that might be chosen for this specific study?\nOptions: A: Incidence rates will be higher during shorter time periods than longer periods.\tB: The prevalence of SLE during consecutively shorter time windows will be overestimated.\tC: The inclusion of attack rates would increase incidence estimates in longer time periods.\tD: The inclusion of attack rates would decrease incidence estimates in shorter time periods.\t\nThe right answer is A: Incidence rates will be higher during shorter time periods than longer periods..", + "output": "The relationship between incidence and prevalence rates is such that incidence reflects the number of new cases that arise in a specific time frame, while prevalence accounts for all existing cases at a given time, including both new and pre-existing ones. In this study, shorter time periods would likely lead to higher apparent incidence rates because there is less time for individuals to exit the \"at risk\" pool due to recovery or death, thus more cases seem new relative to the population at risk. In contrast, longer time periods allow for more turnover in the population, stabilization of case numbers, and potentially reduced incidence rates as once someone has developed SLE, they are no longer part of the population at risk for new cases. Therefore, statement A is true: Incidence rates will be higher during shorter time periods than longer periods." + }, + { + "input": "Question: A 41-year-old man presents to his primary care provider with abdominal pain. He says that the pain “comes and goes” throughout the day and usually lasts 20-30 minutes per episode. He can point to the spot 1-2 inches above the umbilicus where he feels the pain. He denies any feeling of regurgitation or nighttime cough but endorses nausea. He reports that he used to eat three large meals per day but has found that eating more frequently improves his pain. He tried a couple pills of ibuprofen with food over the past couple days and thinks it helped. He has gained four pounds since his past appointment three months ago. The patient denies any diarrhea or change in his stools. He has no past medical history. He drinks 5-6 beers on the weekend and has a 20 pack-year smoking history. He denies any family history of cancer. On physical exam, he is tender to palpation above the umbilicus. Bowel sounds are present. A stool guaiac test is positive. The patient undergoes endoscopy with biopsy to diagnose his condition.\n\nWhich of the following is most likely to be found on histology?\nOptions: A: Mucosal defect in the stomach\tB: Urease-producing organism in the small intestine\tC: PAS-positive material in the small intestine\tD: Crypt abscesses in the large intestine\t\nThe right answer is B: Urease-producing organism in the small intestine.", + "output": "The patient's symptoms: intermittent abdominal pain that improves with more frequent meals and use of ibuprofen suggest peptic ulcer disease (PUD). The endoscopic procedure was likely performed in part to investigate for gastric or duodenal ulcers. An important etiological factor for PUD is the infection by the urease-producing organism Helicobacter pylori (H. pylori) in the stomach or proximal small intestine (duodenum). The presence of this bacterium leads to mucosal inflammation and ulceration. Option B, \"Urease-producing organism in the small intestine,\" is consistent with H. pylori infection on biopsy, which is the most likely underlying cause for PUD. The other options (A, C, D) represent different gastrointestinal pathologies not suggested by the given clinical presentation." + }, + { + "input": "Question: A 43-year-old man with a history of hepatitis C and current intravenous drug use presents with 5 days of fever, chills, headache, and severe back pain. On physical exam, temperature is 100.6 deg F (38.1 deg C), blood pressure is 109/56 mmHg, pulse is 94/min, and respirations are 18/min. He is thin and diaphoretic with pinpoint pupils, poor dentition, and track marks on his arms and legs. A high-pitched systolic murmur is heard, loudest in the left sternal border and with inspiration. He is admitted to the hospital and started on broad-spectrum antibiotics. One of the blood cultures drawn 12 hours ago returns positive for Staphylococcus aureus. Which of the following is the most appropriate next step to confirm the diagnosis?\nOptions: A: Repeat blood cultures now\tB: Repeat blood cultures 24 hours after initial cultures were drawn\tC: Repeat blood cultures 48 hours after initial cultures were drawn\tD: Do not repeat blood cultures\t\nThe right answer is A: Repeat blood cultures now.", + "output": "Choosing to repeat blood cultures immediately (Option A) is the most appropriate next step to confirm the diagnosis of likely infective endocarditis, given the patient's clinical presentation and positive blood culture for Staphylococcus aureus. With his history of intravenous drug use, fever, characteristic murmur, and septic presentation, infective endocarditis is a high concern. Prompt blood cultures are essential for identifying the causative organism, determining its antibiotic susceptibility, and ensuring effective treatment. Continuous bacteremia is expected in infective endocarditis, thus a timely repeat of blood cultures will likely yield positive results, confirming ongoing infection, which is critical for appropriate management." + }, + { + "input": "Question: A 35-year-old woman presents with severe fear reactions to seeing dogs after moving into a new suburban neighborhood. She states that she has always had an irrational and excessive fear of dogs but has been able to avoid it for most of her life while living in the city. When she sees her neighbors walking their dogs outside, she is terrified and begins to feel short of breath. Recently, she has stopped picking up her children from the bus stop and no longer plays outside with her children in order to avoid seeing any dogs. Which of the following would be the best definitive treatment for this patient?\nOptions: A: Selective serotonin reuptake inhibitors (SSRIs)\tB: Short-acting benzodiazepines\tC: Systematic desensitization\tD: Cognitive behavioral therapy\t\nThe right answer is C: Systematic desensitization.", + "output": "Systematic desensitization is the best definitive treatment for this patient as it is specifically designed to address phobias such as her severe fear of dogs. This therapeutic approach involves gradually exposing the patient to the fear-inducing stimulus (in this case, dogs) under controlled conditions while teaching relaxation techniques to cope with the anxiety. It allows the patient to slowly build tolerance and reduce fear responses. Cognitive behavioral therapy is also effective for phobias, but systematic desensitization is more tailored to address specific fears by combining gradual exposure with relaxation. SSRIs and benzodiazepines may temporarily alleviate symptoms but do not target the root cause of the phobia, nor do they offer a definitive solution." + }, + { + "input": "Question: A 13-year-old boy is brought to the emergency department by his mother because of a 6-hour history of severe eye pain and blurry vision. He wears soft contact lenses and has not removed them for 2 days. Ophthalmologic examination shows a deep corneal ulcer, severe conjunctival injection, and purulent discharge on the right. Treatment with topical ciprofloxacin is initiated. A culture of the ocular discharge is most likely to show which of the following?\nOptions: A: Gram-negative, non-maltose fermenting diplococci\tB: Gram-negative, lactose-fermenting bacilli\tC: Gram-negative, oxidase-positive bacilli\tD: Gram-positive, optochin-sensitive diplococci\n\"\t\nThe right answer is C: Gram-negative, oxidase-positive bacilli.", + "output": "A deep corneal ulcer in a young contact lens wearer who has not removed his lenses for an extended period is strongly suggestive of a Pseudomonas aeruginosa infection. This pathogen is a common cause of contact lens-related eye infections and can lead to severe keratitis. Pseudomonas aeruginosa is a gram-negative, oxidase-positive bacillus that thrives in moist environments, including those found on the surface of contact lenses and lens cases that are not properly cleaned. The severe eye pain, blurry vision, and purulent discharge are consistent with this type of bacterial infection. Therefore, a culture of the ocular discharge is most likely to show Gram-negative, oxidase-positive bacilli (Option C)." + }, + { + "input": "Question: A 9-month-old boy is brought to the physician by his mother because of intermittent watery diarrhea for several months. Pregnancy and delivery were uncomplicated. He was diagnosed with eczematous dermatitis at 3 months old. His height and weight are below the 5th percentile. Immunologic evaluation shows a defect in activated regulatory T cells. A genetic analysis shows a mutation in the FOXP3 gene. This patient is at increased risk for which of the following?\nOptions: A: Hemorrhagic diathesis\tB: Ocular telangiectasias\tC: Autoimmune endocrinopathy\tD: Retained primary teeth\t\nThe right answer is C: Autoimmune endocrinopathy.", + "output": "The patient presents with eczematous dermatitis, immunologic abnormalities, and failure to thrive—all consistent with Immune dysregulation, Polyendocrinopathy, Enteropathy, X-linked syndrome (IPEX), caused by mutations in the FOXP3 gene. FOXP3 is essential for the normal function of regulatory T cells (Tregs) which are crucial for maintaining immune tolerance. The defective Tregs lead to a breakdown of self-tolerance and result in various autoimmune disorders. Among the options listed, autoimmune endocrinopathy (C) closely aligns with the manifestation of IPEX syndrome and the role of FOXP3. Options A, B, and D are not typically associated with FOXP3 mutations or IPEX." + }, + { + "input": "Question: A 60-year-old man with a history of osteoarthritis has been awaiting hip replacement surgery for 3 years. During his annual physical, he reports that he has been taking over the counter pain medications, but that no amount of analgesics can relieve his constant pain. Laboratory results reveal that his renal function has deteriorated when compared to his last office visit 2 years ago. Serum creatinine is 2.0 mg/dL, and urinalysis shows 1+ proteinuria. There are no abnormalities seen on microscopy of the urine. A renal biopsy shows eosinophilic infiltration and diffuse parenchymal inflammation. What is the most likely explanation for this patient's deterioration in renal function?\nOptions: A: Focal segmental glomerulosclerosis\tB: Ischemic acute tubular necrosis\tC: Nephrotoxic acute tubular necrosis\tD: Toxic tubulointerstitial nephritis\t\nThe right answer is D: Toxic tubulointerstitial nephritis.", + "output": "The most likely explanation for the patient's deterioration in renal function is D. Toxic tubulointerstitial nephritis. This condition can often result from the chronic use of nonsteroidal anti-inflammatory drugs (NSAIDs), which are common over-the-counter pain medications. The patient's history of unrelenting pain and the long-term use of analgesics suggest possible excessive consumption of NSAIDs. Laboratory findings such as an elevated serum creatinine, proteinuria, and particularly eosinophilic infiltration on renal biopsy support the diagnosis of an allergic interstitial nephritis, rather than glomerular diseases like focal segmental glomerulosclerosis (A) or vascular conditions like ischemic acute tubular necrosis (B). Nephrotoxic acute tubular necrosis (C) could also arise from toxin exposure but is less likely given the biopsy results showing an inflammatory, rather than necrotic, pattern." + }, + { + "input": "Question: A 27-year-old HIV positive female gave birth to a 7lb 2oz (3.2 kg) baby girl. The obstetrician is worried that the child may have been infected due to the mother's haphazard use of her anti-retroviral medications. Which of the following assays would a hospital use detect the genetic material of HIV if the child had been infected?\nOptions: A: Enzyme-linked immunosorbent assay (ELISA)\tB: Rapid HIV antibody test\tC: Polymerase chain reaction\tD: Southern blot\t\nThe right answer is C: Polymerase chain reaction.", + "output": "Option C, Polymerase Chain Reaction (PCR), is the best choice for detecting the genetic material of HIV in a newborn. ELISA and rapid HIV antibody tests, options A and B, detect antibodies to HIV, which are produced by the immune system in response to infection. In a newborn, maternal antibodies can persist for several months and can lead to false-positive results, as they reflect maternal rather than infant infection. Option D, Southern blot, is typically used for DNA fragment analysis and is less relevant in direct viral detection. PCR is highly sensitive and specific for identifying the presence of HIV's genetic material (RNA or DNA) directly, which makes it ideal for diagnosing HIV in newborns where serological tests cannot differentiate between maternal antibodies and those produced by an actual infection in the infant." + }, + { + "input": "Question: A previously healthy 2-year-old boy is brought to the physician because of a 10-day history of unsteady gait, frequent falls, and twitching of the extremities. Physical examination shows bilateral saccadic eye movement in all directions and brief, involuntary muscle contractions of the trunk and limbs. There is an ill-defined, nontender mass in the upper right abdomen. He undergoes surgical resection of the tumor. Histopathologic examination of this mass is most likely to show which of the following?\nOptions: A: Numerous immature lymphocytes in a starry sky pattern\tB: Abortive glomeruli and tubules in a spindle cell stroma\tC: Small blue cells arranged in rosettes around a central neuropil\tD: Hepatocytes in fetal and embryonic stages of differentiation\t\nThe right answer is C: Small blue cells arranged in rosettes around a central neuropil.", + "output": "The described symptoms and findings—unsteady gait, twitching extremities, bilateral saccadic eye movements, along with a nontender abdominal mass—suggest a diagnosis of neuroblastoma, a common extracranial solid cancer in children. Neuroblastomas often present with opsoclonus-myoclonus syndrome, which is characterized by the neurological signs seen in this patient. The histological feature of small blue cells arranged in rosettes around a central neuropil is typical for neuroblastoma, with the cells representing undifferentiated or poorly differentiated neuroblasts, and the \"rosettes\" being indicative of their attempt to form neural structures. This aligns with the clinical presentation and the likely diagnosis." + }, + { + "input": "Question: A 52-year-old fisherman presents to the clinic for an evaluation of a pigmented lesion on his neck. He states that he first noticed the lesion last year, but he believes that it has been slowly growing in size. Dermatopathology determines that the lesion contains neoplastic cells of melanocytic origin. Which of the following characteristics of the lesion would likely be found on physical examination?\nOptions: A: Brown-black color in one area of the lesion to red-white in a different area\tB: Macule that is 5mm in diameter\tC: Well-circumscribed papule with smooth borders\tD: Itching and pain to palpation\t\nThe right answer is A: Brown-black color in one area of the lesion to red-white in a different area.", + "output": "The given information suggests the presence of a melanocytic neoplasm, which can be indicative of melanoma, especially considering the patient's occupation as a fisherman, which involves significant sun exposure—an established risk factor for skin cancers, including melanoma. Key clinical features of melanoma include variability in color, with uneven distribution of pigmentation, often described as different shades of brown, black, tan, red, white, or blue. Option A (\"Brown-black color in one area of the lesion to red-white in a different area\") best matches this description of asymmetrical color variation, a hallmark of melanoma, making it a characteristic likely to be found on physical examination. Options B, C, and D may describe benign lesions and lack specific features suggestive of melanoma." + }, + { + "input": "Question: An 18-month-old girl is brought to the pediatrician by her mother for vaginal bleeding. The mother states that she noticed the bleeding today, which appeared as brown discharge with clots in the patient’s diaper. The mother denies frequent nosebleeds or easy bruising. She also denies any known trauma. She does mention that the patient has been limping and complaining of left leg pain since a fall 2 months ago. On physical exam, there are multiple 2-3 cm hyperpigmented patches on the patient’s trunk. There is bilateral enlargement of the breasts but no pubic hair. The vaginal orifice is normal and clear with an intact hymen. A plain radiograph of the left lower leg shows patchy areas of lytic bone and sclerosis within the femoral metaphysis. Which of the following is associated with the patient’s most likely diagnosis?\nOptions: A: Bitemporal hemianopsia\tB: Hypertension\tC: Pheochromocytoma\tD: Polyostotic fibrous dysplasia\t\nThe right answer is D: Polyostotic fibrous dysplasia.", + "output": "The patient's presentation suggests a condition known as McCune-Albright syndrome (MAS), which is associated with the option D, polyostotic fibrous dysplasia. The key symptoms contributing to this diagnosis are the multiple hyperpigmented patches (café-au-lait spots), which are often seen in MAS. Additionally, the limping and pain following a fall with radiographic findings of patchy lytic bone and sclerosis are indicative of fibrous dysplasia, where normal bone is replaced with fibrous tissue, leading to pain, deformity, and fractures. The vaginal bleeding and breast enlargement without pubic hair suggest precocious puberty, another component of MAS. The other options (A-C) are not typically associated with the described symptoms and findings. Bitemporal hemianopsia is associated with lesions affecting the optic chiasm, hypertension could have several other causes, and pheochromocytoma is a neuroendocrine tumor with symptoms not described here." + }, + { + "input": "Question: A 72-year-old woman is admitted to the hospital for treatment of unstable angina. Cardiac catheterization shows occlusion that has caused a 50% reduction in the diameter of the left circumflex artery. Resistance to blood flow in this vessel has increased by what factor relative to a vessel with no occlusion?\nOptions: A: 64\tB: 16\tC: 8\tD: 4\t\nThe right answer is B: 16.", + "output": "The relationship between vessel diameter and blood flow resistance is given by Poiseuille's Law, which states that resistance is inversely proportional to the fourth power of the radius of a vessel (resistance ∝ 1/radius^4). Therefore, if the diameter (which is twice the radius) is reduced by 50%, the radius is reduced to 50%, or 0.5 of its original size. The new resistance is 1/(0.5)^4 = 1/0.0625 = 16 times the original resistance. Hence, the resistance to blood flow in this artery has increased by a factor of 16, making option B the correct answer." + }, + { + "input": "Question: A 5-year-old boy is brought to the physician because of an irregular gait 3 days after receiving age-appropriate vaccinations. Examination of the lower extremities shows no redness or swelling. When the child stands on his right leg, his left leg drops and his pelvis tilts towards the left. Sensation to light touch is normal in both legs. This patient's symptoms are most likely due to the injection of the vaccine into which of the following locations?\nOptions: A: Inferolateral quadrant of the right buttock\tB: Inferomedial quadrant of the right buttock\tC: Inferomedial quadrant of the left buttock\tD: Superomedial quadrant of the right buttock\t\nThe right answer is D: Superomedial quadrant of the right buttock.", + "output": "The patient's symptoms suggest a lesion affecting the superior gluteal nerve. The Trendelenburg sign, where the pelvis tilts towards the contralateral side when standing on one leg due to weakness of the gluteus medius and minimus muscles, is indicative of this. These muscles are innervated by the superior gluteal nerve, which could be damaged if an injection were given improperly in the upper buttock region. The correct option, D (Superomedial quadrant of the right buttock), indicates the site where an injection could potentially damage the superior gluteal nerve, leading to the observed gait abnormality. Other options would not typically affect this nerve." + }, + { + "input": "Question: A 26-year-old woman comes to the clinic for an annual wellness examination. She is healthy with no prior history of significant illness. She exercises 3-4 times a week and eats a plant-based diet with no carbonated drinks. When asked if anything is bothering her, she reports that she has been having recurring episodes of sneezing, congestion, and itchy eyes for the past year. She denies any fever, sick contacts, cough, headaches, chest pain, urinary symptoms, or constipation/diarrhea during these episodes. She is told to take a medication to alleviate her symptoms as needed. What is the likely mechanism of action of the medication in question?\nOptions: A: Alpha-adrenergic agonist\tB: Competitive blockage of muscarinic receptors\tC: Disruption of disulfide bonds\tD: Short acting beta-2 agonist\t\nThe right answer is A: Alpha-adrenergic agonist.", + "output": "The symptoms of sneezing, congestion, and itchy eyes with no associated systemic symptoms such as fever or cough are suggestive of allergic rhinitis. The lack of systemic symptoms helps rule out infection or other systemic diseases. Allergic rhinitis is often treated symptomatically with medications that relieve the congestion and other related symptoms. An alpha-adrenergic agonist, such as pseudoephedrine or phenylephrine, is commonly used in such conditions because it causes vasoconstriction of the nasal mucosa blood vessels, which leads to a decrease in congestion. This mechanism differs from the action of antihistamines (which block histamine receptors), muscarinic receptor blockers (which affect parasympathetic signaling), disulfide bond disruptors (which affect mucous viscosity), and beta-2 agonists (which are more commonly used to relieve bronchospasms in conditions such as asthma). Therefore, the alpha-adrenergic agonist's primary mechanism relevant here is vasoconstriction in nasal passages to alleviate symptoms." + }, + { + "input": "Question: A 31-year-old woman comes to the physician because of a 5-month history of intermittent flank pain. Over the past 2 years, she has had five urinary tract infections. Her blood pressure is 150/88 mm Hg. Physical examination shows bilateral, nontender upper abdominal masses. Serum studies show a urea nitrogen concentration of 29 mg/dL and a creatinine concentration of 1.4 mg/dL. Renal ultrasonography shows bilaterally enlarged kidneys with multiple parenchymal anechoic masses. Which of the following is the most likely diagnosis?\nOptions: A: Medullary sponge kidney\tB: Autosomal dominant polycystic kidney disease\tC: Autosomal recessive polycystic kidney disease\tD: Obstructive cystic dysplasia\t\nThe right answer is B: Autosomal dominant polycystic kidney disease.", + "output": "The most likely diagnosis is B. Autosomal dominant polycystic kidney disease (ADPKD). The patient's age is typical for ADPKD presentation, which often becomes clinically evident in adulthood. Intermittent flank pain and a history of urinary tract infections suggest a chronic kidney condition. The presence of bilateral upper abdominal masses that are nontender and the finding of enlarged kidneys with multiple cystic lesions on ultrasonography are characteristic of ADPKD. The elevated blood pressure and abnormal renal function tests (urea and creatinine) also support the diagnosis, reflecting the progressive nature of impaired kidney function in ADPKD. Autosomal recessive polycystic kidney disease would typically present earlier in life, making it less likely in a 31-year-old. Medullary sponge kidney and obstructive cystic dysplasia are not associated with the bilateral enlarged kidneys with multiple cysts as described." + }, + { + "input": "Question: A 27-year-old woman, gravida 2, para 1, at 40 weeks' gestation is admitted to the hospital in active labor. The patient reports severe pelvic pain. Pregnancy has been complicated by gestational diabetes. Pregnancy and delivery of her first child were uncomplicated. Current medications include insulin, folic acid, and a multivitamin. Vital signs are within normal limits. The cervix is 100% effaced and 10 cm dilated; the vertex is at -1 station. The fetal heart rate is reactive with no decelerations. Epidural anesthesia is performed and the patient's symptoms improve. Ten minutes later, the patient has dizziness. Her pulse is 68/min, respirations are 16/min, and blood pressure is 90/60 mm Hg. Intravenous fluid resuscitation is begun. Which of the following is the most likely underlying cause of the patient's hypotension?\nOptions: A: Sympathetic block\tB: Hypovolemia\tC: Acute pulmonary hypertension\tD: Aortocaval compression\t\nThe right answer is A: Sympathetic block.", + "output": "The most likely underlying cause of the patient's hypotension is Sympathetic block, option A. Epidural anesthesia involves injecting anesthetic near the nerves that transmit pain from the birth canal to the brain. This can lead to vasodilation in the lower extremities and subsequent pooling of blood due to blockage of sympathetic nerve impulses that normally cause blood vessels to constrict. The resultant decrease in venous return to the heart lowers cardiac output, leading to hypotension. The immediate onset of symptoms following epidural administration supports this diagnosis, as opposed to the other options which are less likely to occur suddenly post-epidural." + }, + { + "input": "Question: A 6-year-old African American boy is referred to the hospital by his family physician for jaundice, normocytic anemia, and severe bone pain. He has a history of several episodes of mild bone pain in the past treated with over the counter analgesics. On physical examination, the child is icteric with nonspecific pain in his hands. His hands are swollen, tender, and warm. There is no chest pain, abdominal pain, fever, or hematuria. A complete metabolic panel and complete blood count with manual differential are performed:\nTotal bilirubin\n8.4 mg/dL\nWBC\n9,800/mm3\nHemoglobin \n6.5 g/dL\nMCV 82.3 fL\nPlatelet count 465,000/mm3\nReticulocyte 7%\nPeripheral blood smear shows multiple clumps of elongated and curved cells and erythrocytes with nuclear remnant. The patient's hemoglobin electrophoresis result is pictured below. What is the most likely cause of his condition?\nOptions: A: Sickle cell trait\tB: Sickle cell disease\tC: Hemoglobin F\tD: HbC\t\nThe right answer is B: Sickle cell disease.", + "output": "The most likely cause of this patient's condition is B. Sickle cell disease. The clinical presentation of severe bone pain, normocytic anemia, and jaundice is characteristic of sickle cell disease. Swollen, tender, and warm hands suggest dactylitis, a common early manifestation in children with sickle cell disease. The laboratory findings of a low hemoglobin of 6.5 g/dL, high reticulocyte count indicating hemolytic anemia, and the presence of sickle cells and erythrocytes with nuclear remnants (Howell-Jolly bodies) on the peripheral blood smear corroborate the diagnosis. Hemoglobin electrophoresis, although not shown, would likely reveal an elevated percentage of hemoglobin S (HbS), which confirms sickle cell disease, rather than just the sickle cell trait, which typically presents with less severe symptoms and a lower percentage of HbS." + }, + { + "input": "Question: A 55-year-old man presents to the emergency department with complaints of nausea, vomiting, palpitations, and dizziness, which have lasted for the past 3 hours. He was diagnosed with heart failure 1 year ago. During his last visit to his primary care physician, he was doing well and the lab results were normal. He reports that he has been taking low-dose aspirin and digoxin regularly for 1 year, and verapamil was recently added to prevent his frequent migraine headaches. An electrocardiogram is performed urgently and shows paroxysmal atrial tachycardia with block. Suspecting digitalis toxicity, the emergency medicine physician sends blood to the lab for a serum digoxin level, which is 3.7 ng/mL (therapeutic range: 0.8–2 ng/mL). Which of the following mechanisms most likely explains the development of digitalis toxicity in this patient?\nOptions: A: Drug-induced hypokalemia\tB: Increased intestinal absorption of digoxin\tC: Inhibition of CYP 3A4 isoenzyme\tD: Inhibition of P-glycoprotein\t\nThe right answer is D: Inhibition of P-glycoprotein.", + "output": "In this case, the patient is exhibiting signs of digitalis toxicity, as evidenced by his symptoms and serum digoxin level, which is above the therapeutic range. The addition of verapamil, a known inhibitor of P-glycoprotein, is the most likely explanation for the increased serum levels of digoxin. P-glycoprotein is a membrane transporter responsible for the efflux of digoxin and other drugs from cells, particularly in the renal tubules and intestinal epithelium. When P-glycoprotein is inhibited by verapamil, the renal clearance of digoxin decreases, leading to an accumulation of the drug in the bloodstream and thus toxicity. The other options do not directly relate to the interaction with verapamil that would increase digoxin levels." + }, + { + "input": "Question: A 66-year-old woman presents to her primary care provider with several days of left flank pain radiating to the abdomen and groin. The patient states that she has noticed a pink tinge to her urine as well. Of note, she has not had any fevers, but endorses several months of fatigue and constipation. The patient was previously healthy except for mild untreated hypertension. On exam, her temperature is 98.6°F (37.0°C), blood pressure is 130/84 mmHg, pulse is 76/min, and respirations are 12/min. On further workup, the patient is found to have calcium oxalate nephrolithiasis with hypercalciuria. Blood studies demonstrate increased parathyroid hormone (PTH) and hypercalcemia. Which of the following is the most likely cause?\nOptions: A: Primary hyperparathyroidism\tB: Pseudopseudohypoparathyroidism\tC: Secondary hyperparathyroidism\tD: Tertiary hyperparathyroidism\t\nThe right answer is A: Primary hyperparathyroidism.", + "output": "The most likely cause is A. Primary hyperparathyroidism. The symptoms of flank pain and pink-tinged urine suggest the presence of calcium stones, which are supported by the finding of calcium oxalate nephrolithiasis with hypercalciuria. The increased PTH and hypercalcemia are characteristic of primary hyperparathyroidism, where there's overproduction of PTH from the parathyroid glands, often due to a benign adenoma. In contrast, secondary hyperparathyroidism is typically a response to chronic hypocalcemia from conditions like renal failure, and tertiary hyperparathyroidism occurs when long-standing secondary hyperparathyroidism leads to autonomous gland activity. Pseudopseudohypoparathyroidism is a genetic condition that does not feature elevated PTH or hypercalcemia." + }, + { + "input": "Question: A 29-year-old man is referred by his marriage counselor to the outpatient psychiatry clinic. The patient’s wife is with him and states that her husband is always complaining and critical of others. He was recently fired from his job to which he claims that his boss was jealous of his hard work. He also does not trust his neighbors and thinks they are out to get all the nice things he has. His wife also says that he has begun to doubt her fidelity and believes that even the marriage counselor is on her side. Which of the following psychiatric disorders also belongs to the same cluster of symptoms?\nOptions: A: Schizotypal personality disorder\tB: Obsessive-compulsive personality disorder\tC: Antisocial personality disorder\tD: Personality disorder not otherwise specified\t\nThe right answer is A: Schizotypal personality disorder.", + "output": "The described symptoms align with Cluster A personality disorders, characterized by odd, eccentric thinking or behavior. Schizotypal personality disorder, option A, falls within this cluster and includes symptoms such as paranoia, distrust, social anxiety, and odd beliefs or magical thinking. The patient's suspicion of his neighbors, his boss's alleged jealousy, and doubts regarding his wife's fidelity suggest paranoia. His critical nature of others may reflect the social deficits associated with schizotypal personality disorder. Obsessive-compulsive personality disorder (option B) is more about orderliness and perfectionism, antisocial (option C) is linked with disregard for others, and personality disorder not otherwise specified (option D) is a non-specific diagnosis that does not directly imply the presence of Cluster A symptoms observed in the patient. Thus, schizotypal personality disorder is most aligned with the shared symptomatology." + }, + { + "input": "Question: A 22-year-old gravida 1 presents to her physician at 15 weeks gestation for a prenatal appointment. She complains of a rash involving her chest, face, and arms, a watery nasal discharge, and mild bilateral knee pain. She has had these symptoms for about 5 days. The symptoms do not seem to bother her too much, but she is concerned for the baby. She had contact with her younger sister, who also had a rash and was diagnosed with rubella infection about 10 days ago at a family gathering. She cannot confirm her vaccination history. Her vital signs are as follows: blood pressure, 110/70 mmHg; heart rate, 89/min; respiratory rate, 12/min; and temperature, 37.6℃ (99.7℉). Examination shows a moderately dense maculopapular lacy rash spread over the patient’s trunk, extremities, and face. No lymph node, liver, or spleen enlargement is noted. The knee joints appear normal.\nSerology performed 1 year ago Current serology\nRubella IgM - negative Rubella IgM - negative\nRubella IgG - 1:128 Rubella IgG - 1:64\nRubella IgG avidity - high Rubella IgG avidity - high\nWhat is the proper next step in the management of this woman?\nOptions: A: Reassure and recommend vaccination against rubella postpartum\tB: Recommend pregnancy termination\tC: Arrange a chorionic villus sampling\tD: Recommend additional serologic testing for parvovirus B19\t\nThe right answer is D: Recommend additional serologic testing for parvovirus B19.", + "output": "The woman's presentation is consistent with a viral rash and her exposure to her sister with rubella raises concern for a rubella infection, which can cause congenital rubella syndrome, a serious fetal condition. However, her negative rubella IgM and stable high-avidity rubella IgG indicate a past infection or immunization, not an acute infection. Since her rubella status suggests immunity, concern shifts to other infections that can present similarly, such as parvovirus B19, which can result in fetal hydrops or miscarriage. Thus, additional serologic testing for parvovirus B19 (Option D) is recommended to rule out this potential cause of fetal harm. Options A, B, and C are not appropriate given the current serology results and clinical scenario." + }, + { + "input": "Question: A 55-year-old man presents with a 2-month history of an increasing sensation of fullness in the upper left side of his abdomen. He complains of increasing fatigue and dyspnea. He has no history of serious illness and takes no medications. His temperature is 36.7°C (98.1°F), pulse is 90/min, respiratory rate is 18/min, and blood pressure is 125/70 mm Hg. His conjunctivae are pale. The examination of the heart and lungs shows no abnormalities. The splenic margin is palpable 8 cm (3.1 in) below the costal margin. No abnormal lymph nodes are found. Laboratory studies show:\nHemoglobin 8 g/dL\nMean corpuscular volume 90 μm3\nLeukocyte count 3,000/mm3\nPlatelet count 85,000/mm3\nPeripheral blood smear shows small lymphocytes with cell membrane projections. Bone marrow aspiration is unsuccessful. Cell immunophenotyping is positive for CD25. Which of the following is the most effective pharmacotherapy at this time?\nOptions: A: Cladribine\tB: Fludarabine\tC: Prednisone\tD: Thalidomide\t\nThe right answer is A: Cladribine.", + "output": "The presentation suggests a lymphoproliferative disorder, with symptoms of splenomegaly (fullness in the abdomen), anemia (pale conjunctivae and fatigue), and findings of thrombocytopenia and leukopenia. The peripheral blood smear showing small lymphocytes with cell membrane projections, and the positive CD25 immunophenotyping, are characteristic of hairy cell leukemia (HCL). Cladribine, option A, is the most effective pharmacotherapy for HCL, as it's a purine analogue effective in inducing long-term remission in the majority of HCL patients. Fludarabine is less commonly used first-line. Prednisone and thalidomide aren't typically indicated in the management of HCL." + }, + { + "input": "Question: A 3-month-old girl is brought to the physician because of a productive cough for 5 days. Over the past month, she has had several episodes of watery stools. She is exclusively breastfed every 3–4 hours for 15–20 minutes. She was delivered vaginally at 38 weeks' gestation at home and has not yet been evaluated by a physician. The mother reports that her child is not gaining weight. The mother had no prenatal care. The infant is at the 5th percentile for height and weight. The infant has not received any immunizations. Her temperature is 38.5°C (101.3°F), pulse is 155/min, respirations are 45/min, and blood pressure is 88/50 mm Hg. Oral examination shows white plaques covering the tongue and the palate. Rales are heard bilaterally on cardiopulmonary examination. Cervical and inguinal lymphadenopathy is present. Which of the following is most likely to confirm the diagnosis?\nOptions: A: Karyotyping of infant's chromosomes\tB: DNA test for CFTR mutation\tC: Fluorescent treponemal antibody absorption test\tD: Polymerase chain reaction for viral genes\t\nThe right answer is D: Polymerase chain reaction for viral genes.", + "output": "The infant's clinical presentation (productive cough, failure to thrive, lymphadenopathy, and oral thrush) suggests an immunocompromised state, likely congenital or perinatal infection. Considering the mother's lack of prenatal care and the absence of immunizations, an infection like HIV should be considered. HIV can cause persistent infections and opportunistic infections (such as oral thrush) in infants. A polymerase chain reaction (PCR) for viral genes is the most appropriate test to confirm the diagnosis of a viral infection, such as HIV, which aligns with the clinical symptoms and findings. Karyotyping and a DNA test for CFTR would test for genetic abnormalities, whereas a fluorescent treponemal antibody absorption test would test for syphilis, which is less consistent with the presented symptoms." + }, + { + "input": "Question: A 6-year-old boy is brought in by his parents to a pediatrician’s office for persistent fever. His temperature has ranged from 38.6°C–39.5°C (101.5°F–103.1°F) over the past week. He was diagnosed with a gene defect on chromosome 7, which has caused dysfunction in a transmembrane protein. This defect has resulted in several hospitalizations for various respiratory infections. At the present time, he is struggling with a cough with thick purulent sputum. A sputum sample is sent for culture and the patient is started on vancomycin.The pediatrician tells the patient’s family that there is a collection of mucus in the respiratory tree which increases the patient's chances of future infections. He will have to undergo chest physiotherapy and they should report any signs of infection immediately to a medical professional. Which of the following pathogens will most likely infect this patient in adulthood?\nOptions: A: Staphylococcus\tB: Pseudomonas\tC: Burkholderia\tD: Candida\t\nThe right answer is B: Pseudomonas.", + "output": "The patient's symptoms and gene defect consistent with cystic fibrosis suggest Pseudomonas aeruginosa as the most likely pathogen. Individuals with cystic fibrosis have abnormal mucus secretion due to a faulty CFTR protein, which creates a conducive environment for chronic infections. Pseudomonas is a common bacterium causing persistent respiratory infections in older children and adults with cystic fibrosis due to its ability to thrive in the thick mucus present in their lungs. The other options, while possible, are less characteristic for the chronic and recurrent respiratory infections seen in cystic fibrosis. Staphylococcus is more common in younger patients, Burkholderia is less prevalent, and Candida, a fungus, isn't typically involved in the initial lung disease progression." + }, + { + "input": "Question: A six-year-old boy with a history of asthma currently uses an albuterol inhaler as needed to manage his asthma symptoms. His mother brings him into your office because she feels she has had to increase the patient’s use of his inhaler to four times per week for the past month. She also reports that he has woken up three times during the night from his symptoms this month. The boy reports that he is upset because he can’t always keep up with his friends in the playground. His past medical history is significant for allergic rhinitis. The patient’s temperature is 98°F (36.6°C), blood pressure is 110/70 mmHg, pulse is 88/min, and respirations are 18/min with an oxygen saturation of 98% O2 on room air. Auscultation of his lungs reveals bilateral late expiratory wheezes.\n\nWhat changes should be made to his current asthma treatment regimen?\nOptions: A: Maintain current therapy\tB: Add fluticasone daily\tC: Add salmeterol twice daily\tD: Add zileuton twice daily\t\nThe right answer is B: Add fluticasone daily.", + "output": "The boy's increased use of albuterol to four times a week and nighttime awakenings suggest his asthma is not well-controlled on his current as-needed regimen. According to asthma management guidelines, the use of a short-acting beta-agonist (SABA) like albuterol more than twice a week for symptom control (not including use for prevention of exercise-induced bronchospasm) indicates the need for regular long-term control therapy. Fluticasone, a corticosteroid, is a first-line controller medication that reduces inflammation in the airways, leading to improved control of asthma symptoms and decreased exacerbation frequency. Option B is the appropriate step up in therapy for a patient with persistent, mild to moderate asthma." + }, + { + "input": "Question: A 23-year-old woman comes to the physician because of progressive fatigue and painful swelling of her right knee for 3 weeks. She works as a professional ballet dancer and says, “I'm always trying to be in shape for my upcoming performances.” She is 163 cm (5 ft 4 in) tall and weighs 45 kg (99 lb); BMI is 17 kg/m2. Physical examination shows tenderness and limited range of motion in her right knee. Oral examination shows bleeding and swelling of the gums. There are diffuse petechiae around hair follicles on her abdomen and both thighs. Laboratory studies show a prothrombin time of 12 seconds, an activated partial thromboplastin time of 35 seconds, and a bleeding time of 11 minutes. Arthrocentesis of the right knee shows bloody synovial fluid. The patient’s condition is most likely associated with a defect in a reaction that occurs in which of the following cellular structures?\nOptions: A: Lysosomes\tB: Rough endoplasmic reticulum\tC: Nucleus\tD: Extracellular space\t\nThe right answer is B: Rough endoplasmic reticulum.", + "output": "The patient presents with symptoms like gum bleeding, petechiae, prolonged bleeding time, and bloody synovial fluid, suggesting a defect in the coagulation process. Given these findings, we should consider a deficiency in coagulation factors, many of which are synthesized in the liver as precursor proteins which then undergo post-translational modification. The rough endoplasmic reticulum (RER) is responsible for the synthesis of these proteins. Vitamins, especially vitamin K, are crucial for the modification of these factors in the liver. The patient's low BMI and rigorous physical activity suggest a potential for nutritional deficiency, which could include vitamin K, leading to impaired coagulation factor production in the RER. Hence, option B (Rough endoplasmic reticulum) is the most likely answer in relation to the patient's condition." + }, + { + "input": "Question: A 15-year-old boy is brought to the physician for evaluation of a learning disability. His teachers have reported hyperactivity during class, difficulties with social interaction, and poor scores on reading and writing assessments. Molecular analysis shows an increased number of CGG trinucleotide repeats. Which of the following findings are most likely to be seen on physical examination of this patient?\nOptions: A: Frontal balding and cataracts\tB: Long face and large everted ears\tC: Almond-shaped eyes and downturned mouth\tD: Thin upper lip and receding chin\t\nThe right answer is B: Long face and large everted ears.", + "output": "The presence of an increased number of CGG trinucleotide repeats is suggestive of Fragile X Syndrome (FXS), a genetic disorder that causes intellectual disability and behavioral challenges. The typical physical features associated with FXS, which are likely to be observed on physical examination, include a long face, large everted ears, and macroorchidism (enlarged testicles) after puberty. While this patient's symptoms of hyperactivity, social interaction difficulties, and specific learning disabilities are consistent with FXS, the key for the diagnosis is the unique physical presentation outlined in option B. None of the other options listed are commonly associated with the physical manifestations of FXS." + }, + { + "input": "Question: A 51-year-old man presents to his primary care provider complaining of malaise. He returned from a research trip to Madagascar 2 weeks ago and has since developed a worsening fever with a maximum temperature of 102.2°F (39°C). He also reports some swelling around his neck and groin. He works as a zoologist and was in rural Madagascar studying a rare species of lemur. His past medical history is notable for hypertension and gout. He takes lisinopril and allopurinol. His temperature is 101.9°F (38.3°C), blood pressure is 145/85 mmHg, pulse is 110/min, and respirations are 22/min. On exam, he has painful erythematous cervical, axillary, and inguinal lymphadenopathy. Black hemorrhagic eschars are noted on his fingertips bilaterally. The pathogen responsible for this patient’s condition produces a virulence factor that has which of the following functions?\nOptions: A: Cleave immunoglobulin\tB: Inhibit phagocytosis\tC: Inhibit ribosomal function\tD: Trigger widespread inflammation\t\nThe right answer is B: Inhibit phagocytosis.", + "output": "The patient's symptoms, including fever, lymphadenopathy, and black eschars, particularly following travel to Madagascar, suggest a possible infection with Yersinia pestis, which causes bubonic plague. This pathogen is endemic in rural Madagascar and transmitted through flea bites, which aligns with his recent outdoor research activities. The painful lymphadenopathy in the groin and neck (buboes) and fever are classic signs of this disease. Y. pestis has a virulence factor called the Yersinia outer protein E (YopE) which inhibits phagocytosis by disrupting the actin cytoskeleton of phagocytes, thereby preventing the immune system from effectively engulfing and destroying the bacteria. Therefore, option B (Inhibit phagocytosis) is correct, as it corresponds to the mechanism of the virulence factor produced by the pathogen implicated in this scenario." + }, + { + "input": "Question: A 45-year-old African-American male presents to the family medicine physician to assess the status of his diabetes. After reviewing the laboratory tests, the physician decides to write the patient a prescription for miglitol and states that it must be taken with the first bite of the meal. Which of the following bonds will no longer be cleaved when the patient takes miglitol?\nOptions: A: Phosphodiester bonds\tB: Glycosidic bonds\tC: Cystine bonds\tD: Hydrogen bonds\t\nThe right answer is B: Glycosidic bonds.", + "output": "Miglitol is an alpha-glucosidase inhibitor, a type of oral antidiabetic medication that helps manage blood sugar levels in type 2 diabetes patients. It functions by hindering the alpha-glucosidase enzymes found in the intestinal brush border. These enzymes are responsible for the cleavage of glycosidic bonds during carbohydrate digestion, thus slowing down the breakdown of starches and disaccharides into glucose. As a result, postprandial (after eating) glucose absorption is delayed, which helps in controlling blood sugar spikes. Other options, including phosphodiester bonds, cystine bonds, and hydrogen bonds, are not directly affected by miglitol; hence, glycosidic bonds (Option B) are the correct choice. This bond must be specified because miglitol specifically inhibits the enzyme that cleaves this type of bond in carbohydrates during digestion." + }, + { + "input": "Question: A previously healthy 24-year-old man comes to the physician 1 day after the onset of burning pain, swelling, and multiple blisters over the left index finger. He works as a dental assistant and is concerned about not being able to work. The patient wonders if the symptoms might be related to a hunting trip he returned from 1 week ago because he remembers seeing a lot of poison oak. He is sexually active with one female partner, and they do not use condoms. His father has a history of thromboangiitis obliterans. He has smoked one pack of cigarettes daily for 5 years. He drinks two to four beers on social occasions. Vital signs are within normal limits. Physical examination shows swelling and erythema of the pulp space of the left index finger; there are multiple 3-mm vesicles. Laboratory studies show a leukocyte count of 12,000 cells/mm3. In addition to supportive therapy, which of the following is the most appropriate next step in management?\nOptions: A: Oral acyclovir\tB: Oral cephalexin\tC: Smoking cessation\tD: Washing thoroughly\n\"\t\nThe right answer is A: Oral acyclovir.", + "output": "The presentation of burning pain, swelling, and vesicles on the finger of a young, otherwise healthy man is highly suggestive of herpetic whitlow. Herpetic whitlow is a viral infection caused by the herpes simplex virus and can be acquired through contact with infected bodily fluids, which is plausible in his occupation as a dental assistant. The patient's recent history of potential exposure to poison oak points toward a differential diagnosis, but the described symptoms are more consistent with a herpetic infection rather than contact dermatitis. Sexual activity without condoms raises the possibility of exposure to herpes simplex virus, further supporting the diagnosis. The high leukocyte count is consistent with an infection. Oral acyclovir is an antiviral medication used to treat herpes simplex virus infections, and it is the most appropriate next step in management to relieve symptoms and promote healing. Options B (antibiotics), C (smoking cessation), and D (washing) are not immediately relevant to treating the likely herpetic infection." + }, + { + "input": "Question: A 24-year-old man is brought to the emergency room by the police after a fall. The patient smells of alcohol and is slurring his words. You find out that he has recently been fired from his job as a salesperson for months of tardiness and poor performance. The police tell you that his girlfriend, who was there at the time of the fall, mentioned that the patient has been struggling with alcohol for at least a year. Upon physical examination, the patient becomes agitated and starts yelling. He accuses his ex-boss of being a cocaine addict and says he couldn’t keep up sales percentages compared to someone using cocaine. Which of the following psychiatric defense mechanisms is the patient demonstrating?\nOptions: A: Denial\tB: Displacement\tC: Projection\tD: Regression\t\nThe right answer is C: Projection.", + "output": "The patient is demonstrating \"projection,\" which is a psychological defense mechanism where an individual attributes their own unacceptable thoughts, feelings, or motives to another person. By accusing his ex-boss of being a cocaine addict and blaming his inability to keep up sales percentages on that accusation, the patient is projecting his own issues with substance abuse (alcohol) onto his boss. This helps him deny his responsibility in his poor job performance and tardiness. He externalizes the cause of his failures instead of acknowledging his struggle with alcohol, which is negatively impacting his life and work. Therefore, option C, Projection, is the correct answer." + }, + { + "input": "Question: A 42-year-old man presents to a neurologist for evaluation of severe headache. He started experiencing episodic headaches and palpitations 3 months ago; however, he thought that these symptoms were simply related to the stress of moving and starting a new job. He has continued to experience these intermittent headaches and has also noticed that he sweats profusely during these episodes. On presentation his temperature is 99.1°F (37.3°C), blood pressure is 163/112 mmHg, pulse is 112/min, and respirations are 15/min. Physical exam reveals pallor and perspiration but is otherwise unremarkable. The most likely cause of this patient's symptoms is produced by cells located in which region of the histology slide shown in figure A?\nOptions: A: A\tB: B\tC: C\tD: E\t\nThe right answer is D: E.", + "output": "The presentation of episodic headaches, palpitations, profuse sweating, high blood pressure, and tachycardia in a 42-year-old man strongly suggests a pheochromocytoma, which is a catecholamine-secreting tumor commonly found in the adrenal medulla. On histology slides of the adrenal gland, the medulla is the area within the gland that contains chromaffin cells, which are the cells responsible for the production of catecholamines. Based on these clinical features and the typical histological location of such a tumor, the correct answer is the region where these chromaffin cells are found, which is likely to be indicated as option D in figure A, representing the adrenal medulla." + }, + { + "input": "Question: A 62-year-old woman presents to the emergency department for vision changes. About 90 minutes ago she was on the couch watching television with her husband when she suddenly noticed that she couldn't see the screen normally. She also felt a little dizzy. She has never had anything like this happen before. Her general physical exam is unremarkable. Her neurologic exam is notable for loss of vision in the right visual field of both eyes with macular sparing, though extraocular movements are intact. A computed tomography (CT) scan of the head shows no acute intracranial process. Angiography of the brain will most likely show a lesion in which of the following vessels?\nOptions: A: Internal carotid artery\tB: Anterior cerebral artery\tC: Posterior cerebral artery\tD: Basilar artery\t\nThe right answer is C: Posterior cerebral artery.", + "output": "The patient is presenting with symptoms indicative of a homonymous hemianopia with macular sparing, which is typically associated with a vascular lesion affecting the posterior cerebral artery (PCA). The PCA supplies blood to the occipital lobe, where the primary visual cortex is located. Since the macula (central vision) is spared, this suggests that the blood supply to the central visual field, which is redundantly supplied by both PCAs, is intact. Additionally, the lack of acute findings on the CT scan helps to rule out large vessel occlusions such as those that may occur in the internal carotid artery or basilar artery, thus making the posterior cerebral artery the most likely site of lesion causing these symptoms." + }, + { + "input": "Question: A 5-year-old male presents to the pediatrician with a 10-day history of cough that is worse at night. The patient has a history of mild intermittent asthma and has been using his albuterol inhaler without relief. He has also been complaining of headache and sore throat, and his mother has noticed worsening rhinorrhea. The patient’s past medical history is otherwise unremarkable, and he has no known drug allergies. In the office, his temperature is 101.8°F (38.8°C), blood pressure is 88/65 mmHg, pulse is 132/min, and respirations are 16/min. The patient has purulent mucus draining from the nares, and his face is tender to palpation over the maxillary sinuses. His pharynx is erythematous with symmetric swelling of the tonsils. On lung exam, he has moderate bilateral expiratory wheezing.\n\nWhich of the following is the best next step in management?\nOptions: A: Amoxicillin\tB: Amoxicillin-clavulanic acid\tC: Clindamycin\tD: Levofloxacin\t\nThe right answer is B: Amoxicillin-clavulanic acid.", + "output": "The patient presents with symptoms suggestive of a bacterial sinus infection, supported by purulent nasal discharge, facial tenderness over the sinuses, and a history of worsening rhinorrhea, along with systemic signs such as fever. Given his age and presentation, a common causative agent would be Streptococcus pneumoniae or Haemophilus influenzae, both of which can produce beta-lactamase. Amoxicillin-clavulanic acid (Choice B) is therefore a suitable choice as it combines amoxicillin with clavulanate, a beta-lactamase inhibitor, thus extending the antibiotic's coverage to beta-lactamase producing organisms. Clindamycin (Choice C) does not cover the typical pathogens of sinusitis as well, and levofloxacin (Choice D) is not recommended in children due to potential adverse effects on developing cartilage." + }, + { + "input": "Question: A 65-year-old man, known to have chronic obstructive lung disease for the last 3 years, presents to the emergency department with a cough, breathing difficulty, and increased expectoration for a day. There is no history of fever. His regular medications include inhaled salmeterol and inhaled fluticasone propionate. He was a chronic smoker but quit smoking 2 years ago. His temperature is 37.1°C (98.8°F), the pulse rate is 88/min, the blood pressure is 128/86 mm Hg, and the respirations are 30/min. On physical examination, the use of the accessory muscles of respiration is evident and the arterial saturation of oxygen is 87%. Auscultation of the chest reveals the presence of bilateral rhonchi with diminished breath sounds in both lungs. Which of the following medications, if administered for the next 2 weeks, is most likely to reduce the risk of subsequent exacerbations over the next 6 months?\nOptions: A: Oral prednisone\tB: Supplemental oxygen\tC: Montelukast\tD: Nebulized N-acetylcysteine\t\nThe right answer is A: Oral prednisone.", + "output": "The patient presents with an exacerbation of chronic obstructive pulmonary disease (COPD), indicated by worsening cough, breathing difficulty, and increased sputum production without fever. The choice of oral prednisone is justified because systemic corticosteroids are commonly used to treat acute exacerbations of COPD. They reduce airway inflammation and improve lung function, which can alleviate symptoms and prevent further immediate deterioration. A course of oral steroids, like prednisone, is recommended for short-term treatment to decrease the risk of a relapse or another exacerbation within the next few months. The other options are less effective in reducing the risk of near-future exacerbations in the context of an acute COPD exacerbation." + }, + { + "input": "Question: A 22-year-old woman presents to the emergency department for abdominal pain. The patient has been trying to get pregnant and was successful recently, but now she is experiencing abdominal pain, contractions, and a bloody vaginal discharge. According to her last appointment with her primary care provider, she is 10 weeks pregnant. The patient has a past medical history of constipation, anxiety, and substance abuse. She is not currently taking any medications. Her temperature is 99.5°F (37.5°C), blood pressure is 107/58 mmHg, pulse is 90/min, respirations are 17/min, and oxygen saturation is 98% on room air. Physical exam is notable for a bloody vaginal discharge and an open cervix. The patient is initially extremely grateful for the care she is receiving and thanks the doctor and the nursing staff for saving her baby. Upon hearing the news that she is having a spontaneous abortion, the patient becomes angry and aggressive and states that her physician and the medical staff are incompetent, evil, and she is going to sue all of them. The patient is left to grieve but upon entering the room again you notice that the patient has new lacerations over her wrists and that some of the medical equipment in the room is broken. Which of the following is the most likely diagnosis?\nOptions: A: Antisocial personality disorder\tB: Borderline personality disorder\tC: Normal grief response\tD: Post-traumatic stress disorder\t\nThe right answer is B: Borderline personality disorder.", + "output": "The most likely diagnosis is B, Borderline personality disorder (BPD). The rapid oscillation between gratitude and aggressive accusations following emotionally distressing news is characteristic of the instability in interpersonal relationships and self-image seen in BPD. Her intense anger and erratic behavior, including damaging property and self-harm after a stressful event, align with the impulsive actions and self-harming behavior that are among the diagnostic criteria for BPD. Normal grief might involve sadness and mourning but would not commonly precipitate such extreme behaviors. Antisocial personality disorder and PTSD do not align as closely with the presented symptoms in the acute setting." + }, + { + "input": "Question: A 5-year-old male is brought to the pediatrician with complaints of a painful mouth/gums, and vesicular lesions on the lips and buccal mucosa for the past 4 days. The patient has not been able to eat or drink due to the pain and has been irritable. The patient also reports muscle aches. His vital signs are as follows: T 39.1, HR 110, BP 90/62, RR 18, SpO2 99%. Physical examination is significant for vesicular lesions noted on the tongue, gingiva, and lips, with some vesicles having ruptured and ulcerated, as well as palpable cervical and submandibular lymphadenopathy. Which of the following is the most likely causative organism in this patient's presentation?\nOptions: A: CMV\tB: HIV\tC: HSV-1\tD: HSV-2\t\nThe right answer is C: HSV-1.", + "output": "The most likely causative organism in this patient's presentation is Herpes Simplex Virus-1 (HSV-1). The clinical presentation of painful mouth/gums, vesicular lesions on the lips and buccal mucosa, and the inability to eat or drink due to pain is suggestive of primary herpetic gingivostomatitis, which is commonly caused by HSV-1, especially in pediatric populations. The presence of fever (T 39.1), irritability, and muscle aches also support a viral etiology. HSV-2 typically causes genital lesions. CMV and HIV are less likely to present with acute vesicular oral lesions in an otherwise healthy child. HSV-1's tropism for oral mucosa and its typical presentation in this age group confirm it as the most likely suspect." + }, + { + "input": "Question: Patients with the diagnosis of sickle cell anemia make a specific type of hemoglobin known as HgbS. This mutation results in the sickling of their red blood cells when exposed to inciting factors such as hypoxic conditions. Patients are often treated with hydroxyurea, which has which of the following direct effects on their hemoglobin physiology?\nOptions: A: Decreases oxygen carrying capacity of hemoglobin\tB: Increases levels of fetal hemoglobin (HgbF)\tC: Decreases levels of HgbS\tD: Decreases levels of fetal hemoglobin (HgbF)\t\nThe right answer is B: Increases levels of fetal hemoglobin (HgbF).", + "output": "Hydroxyurea is used in the treatment of sickle cell anemia because it increases the production of fetal hemoglobin (HgbF). HgbF has a higher affinity for oxygen compared to HgbS and does not sickle under low oxygen conditions. By increasing HgbF, hydroxyurea reduces the proportion of sickle hemoglobin (HgbS) in the red blood cells and therefore the sickling and subsequent complications associated with sickle cell anemia. It does not directly decrease the oxygen-carrying capacity of hemoglobin or reduce levels of HgbS; rather, it dilutes the effect of HgbS by increasing the relative concentration of HgbF." + }, + { + "input": "Question: A 35-year-old woman presents to the physician because of episodes of difficulty swallowing for the past 3 months. She feels solid food getting stuck in her chest behind the sternum. She does not have any issues with liquids. She has no coughing or nasal regurgitation. She has no hoarseness or weight loss. She reports occasional heartburn that has lasted for about a year. Her past medical history is significant for asthma and eczema. She has no history of any serious illness and takes no medications. Her vital signs are within normal limits. Physical examination shows no abnormal findings. An endoscopic image of the esophagus is shown. Mucosal biopsy shows eosinophilic infiltration. Which of the following is the most appropriate pharmacotherapy at this time?\nOptions: A: Budesonide\tB: Fluconazole\tC: Nitroglycerin\tD: Omeprazole\t\nThe right answer is D: Omeprazole.", + "output": "The patient's symptoms and the eosinophilic infiltration found in the mucosal biopsy are suggestive of eosinophilic esophagitis (EoE), an immune/antigen-mediated esophageal condition. However, her symptoms also point towards gastroesophageal reflux disease (GERD), as she experiences difficulty swallowing solid foods, occasional heartburn, and the sensation of food sticking in her chest, all common in GERD. Omeprazole (D) is a proton pump inhibitor (PPI) that is often used as a first-line treatment for GERD to suppress gastric acid production and can also help in distinguishing GERD from EoE. If symptoms and histologic findings improve with PPI therapy, GERD is likely the primary diagnosis, not EoE. Budesonide (A) is a corticosteroid that might be used for EoE, but PPIs are typically tried first due to the overlap of symptoms. Fluconazole (B) is an antifungal and would not be indicated here. Nitroglycerin (C) is used for angina and is unrelated to the patient's condition." + }, + { + "input": "Question: A 62-year-old man with a history of chronic bronchitis comes to the physician because of a 1-month history of worsening shortness of breath and cough productive of thick sputum. He smoked one pack of cigarettes daily for 20 years but quit 5 years ago. Physical examination shows an increased anteroposterior chest diameter and coarse crackles in the lower lung fields bilaterally. Treatment with a drug that directly antagonizes the effects of vagal stimulation on the airways is begun. Which of the following drugs was most likely started?\nOptions: A: Fluticasone\tB: Montelukast\tC: Tiotropium\tD: Cromolyn\t\nThe right answer is C: Tiotropium.", + "output": "The symptoms and history provided describe a patient with chronic obstructive pulmonary disease (COPD), likely due to chronic bronchitis exacerbated by his history of smoking. Treatment with a drug that antagonizes the effects of vagal stimulation would be aimed at reducing bronchoconstriction. Tiotropium (Option C) is a long-acting muscarinic antagonist (LAMA) that inhibits the action of acetylcholine at the muscarinic receptors in the airways, leading to relaxation and dilation of the airways, which can alleviate symptoms of COPD. Fluticasone (A) is a corticosteroid; Montelukast (B) is a leukotriene receptor antagonist; Cromolyn (D) stabilizes mast cells, none of which directly antagonize vagal effects." + }, + { + "input": "Question: A 38-year-old nursing home worker presents to the clinic with complaints of fever, loss of appetite, fatigue, and productive cough for the past couple of months. His fever is low-grade and sputum is often blood-tinged. He has lost 6.8 kg (15.0 lb) during this period and complains of profound night sweats. A plain radiograph of the patient’s chest shows consolidation in the apical part of the right lung. Baseline investigations show the following:\nComplete blood count\nHemoglobin 11 g/dL\nWhite blood cells \nTotal count 16,000/mm3\nDifferential count \nNeutrophils 35%\nLymphocytes 54%\nEosinophils 11%\nErythrocyte sedimentation rate 84 mm\nThe physician suspects that the patient is suffering from a chronic lung infection. Which of the following statements best describes the type of lung inflammation in this patient?\nOptions: A: There are small granulomas with few epithelioid cells along with fibrosis.\tB: It has a granuloma with Anitchov cells around a core of fibrinoid collagen necrosis.\tC: It consists of a largely circumscribed granuloma with epithelioid cells with Langhans cells.\tD: This type of granulomatous inflammation is also seen in histoplasmosis.\t\nThe right answer is C: It consists of a largely circumscribed granuloma with epithelioid cells with Langhans cells..", + "output": "The clinical presentation of a chronic productive cough, low-grade fever, weight loss, night sweats, and apical lung consolidation, along with a heightened erythrocyte sedimentation rate (ESR) and blood-tinged sputum, is classic for tuberculosis (TB). The presence of granulomas with epithelioid cells and Langhans-type giant cells is a hallmark of TB infection. Option C describes this type of granuloma, indicating the involvement of the immune response typically associated with TB. The other options describe other types of granulomas not specific to TB or are associated with different diseases, making them less likely in this clinical scenario based on the given information." + }, + { + "input": "Question: While in the ICU, a 62-year-old male undergoes placement of a Swan-Ganz catheter to evaluate his right heart pressures. All pressures are found to be within normal limits, and the cardiology fellow records a pulmonary wedge pressure of 10 mmHg. Which of the following are normal values for the pressures that will be obtained from this patient's right ventricle?\nOptions: A: 25/10 mmHg\tB: 25/5 mmHg\tC: 10/0 mmHg\tD: 100/70 mmHg\t\nThe right answer is B: 25/5 mmHg.", + "output": "Right ventricular pressures are typically much lower than those of the left heart due to the lower resistance of the pulmonary circulation. The normal range for right ventricular systolic pressure is 15-30 mmHg, and for diastolic pressure, it's 3-8 mmHg. Option A (25/10 mmHg) could be considered borderline normal, but Option B (25/5 mmHg) fits squarely within the typical range and is the most accurate representation of normal right ventricular pressures. Option C (10/0 mmHg) is too low, suggesting hypotension or measurement error, and Option D (100/70 mmHg) aligns with systemic arterial pressures, not the right ventricle. Therefore, B is the correct choice." + }, + { + "input": "Question: A 46-year-old woman with a history of previously well-controlled HIV infection comes to the physician for follow-up after a health maintenance examination last week. She is currently unemployed and lives in a low-income neighborhood with her 3 children. For the past 3 years, her HIV RNA viral load was undetectable, but last week, her viral load was 8,391 copies/mL (N < 50). Current medications include dolutegravir, tenofovir, and emtricitabine. On questioning, she says that she misses her medications every other day. Which of the following responses by the physician is most appropriate?\nOptions: A: \"\"\"Let's talk about what makes it difficult for you to take your medications.\"\"\"\tB: \"\"\"Are you aware that it is essential to take your medications every day?\"\"\"\tC: \"\"\"The social worker can help subsidize next month's medications.\"\"\"\tD: \"\"\"We should go over the instructions on how to take your medications again.\"\"\"\t\nThe right answer is A: \"\"\"Let's talk about what makes it difficult for you to take your medications.\"\"\".", + "output": "Option A, \"Let's talk about what makes it difficult for you to take your medications,\" is the most appropriate response. This option addresses the adherence issue in a non-judgmental and open manner, inviting the patient to discuss any barriers she may be facing, such as psychological, social, or economic challenges. Given the context that she lives in a low-income neighborhood and is unemployed, there may be complex reasons contributing to her inconsistent medication adherence. By understanding these factors, the physician can work collaboratively with the patient to overcome barriers and improve treatment outcomes. Options B, C, and D do not adequately address the potential underlying issues affecting adherence and may come across as directive or dismissive of the patient's individual circumstances." + }, + { + "input": "Question: A 56-year-old woman presents to a physician with severe pain and swelling of the left inguinal area for 3 days. She has a fever and malaise. Last week she noted several painless red papules on her left thigh when she was on a summer trip to Madagascar. She has no history of serious illnesses and is on no medications. There has been no recent contact with any animals or pets. The temperature is 38.6℃ (101.5℉), the pulse is 78/min, the respiration rate is 12/min, and the blood pressure is 110/65 mm Hg. Swelling of the left inguinal area was noted; however, there were no skin changes. Several large, tense, and tender lymph nodes with a boggy consistency were palpated in the inguinal region. The right inguinal area is normal on physical exam. There was no lymphadenopathy in other areas. No abnormalities existed in the lungs, heart, and abdomen. Microscopic examination of pus from the inguinal lymph nodes revealed gram-negative Coccobacilli. Serum anti-F1 titers show a 4-fold rise. Which of the following is the most appropriate pharmacotherapy at this time?\nOptions: A: Azithromycin\tB: Imipenem\tC: Streptomycin\tD: No pharmacotherapy\t\nThe right answer is C: Streptomycin.", + "output": "This patient's presentation suggests bubonic plague, especially given the history of travel to Madagascar, presence of fever, tender lymphadenopathy, and gram-negative coccobacilli on microscopic examination. A 4-fold rise in serum anti-F1 titers also supports this diagnosis, since F1 is the capsule antigen of Yersinia pestis, the causative organism of plague. Streptomycin (Option C) is the drug of choice for treating plague. Azithromycin (Option A) and Imipenem (Option B) can be effective against a variety of bacteria but are not the primary choices for plague. No pharmacotherapy (Option D) would be inappropriate given the severity of the infection." + }, + { + "input": "Question: A 60-year-old woman is brought to the emergency department because of sudden, painless loss of vision in her right eye that occurred 30 minutes ago while watching TV. She has coronary artery disease, hypertension, and type 2 diabetes mellitus; she has had trouble adhering to her medication regimen. Her blood pressure is 160/85 mm Hg. Examination shows 20/50 vision in the left eye and no perception of light in the right eye. Direct pupillary reflex is present in the left eye, but absent in the right eye. Accommodation is intact bilaterally. Intraocular pressure is 16 mm Hg in the left eye and 18 mm Hg in the right eye. Fundoscopic examination of the right eye shows a pale, white retina with a bright red area within the macula. The optic disc appears normal. Which of the following is the most likely diagnosis?\nOptions: A: Retinal detachment\tB: Central retinal artery occlusion\tC: Acute angle-closure glaucoma\tD: Anterior ischemic optic neuropathy\t\nThe right answer is B: Central retinal artery occlusion.", + "output": "The sudden, painless loss of vision, alongside the fundoscopic findings of a pale, white retina with a cherry-red spot at the macula, is pathognomonic for central retinal artery occlusion (CRAO). These findings, in combination with the patient's absent direct pupillary reflex in the affected eye, suggest that the retinal circulation has been compromised. Her personal history of coronary artery disease, hypertension, and diabetes increases her risk for vascular occlusions. Option A, retinal detachment, would typically present with visual field defects and possibly pain. Acute angle-closure glaucoma (C) generally presents with pain and elevated intraocular pressure, which this patient does not have. Anterior ischemic optic neuropathy (D) usually presents with optic disc swelling, which is not described here." + }, + { + "input": "Question: A 24-year-old woman is brought to the emergency room (ER) by her co-workers after they found her unconscious in her cubicle when they returned from lunch. They tell you that she has diabetes but do not know anything more about her condition. The woman’s vital signs include: pulse 110/min, respiratory rate 24/min, temperature 36.7°C (98.0°F), and blood pressure 90/60 mm Hg. On physical examination, the patient is breathing heavily and gives irrelevant responses to questions. The skin and mucous membranes appear dry. Examination of the abdomen reveals mild diffuse tenderness to palpation. Deep tendon reflexes in the extremities are 1+ bilaterally. Laboratory studies show:\nFinger stick glucose 630 mg/dL\nArterial blood gas analysis:\npH 7.1\nPO2 90 mm Hg\nPCO2 33 mm Hg\nHCO3 8 mEq/L\nSerum:\nSodium 135 mEq/L\nPotassium 3.1 mEq/L\nChloride 136 mEq/L\nBlood urea nitrogen 20 mg/dL\nSerum creatinine 1.2 mg/dL\nUrine examination shows:\nGlucose Positive\nKetones Positive\nLeukocytes Negative\nNitrite Negative\nRBCs Negative\nCasts Negative\nThe patient is immediately started on a bolus of intravenous (IV) 0.9% sodium chloride (NaCl). Which of the following is the next best step in the management of this patient?\nOptions: A: Infuse NaHCO3 slowly\tB: Switch fluids to 0.45% NaCl\tC: Start IV insulin infusion\tD: Replace potassium intravenously\t\nThe right answer is D: Replace potassium intravenously.", + "output": "The patient's lab results indicate severe hyperglycemia (630 mg/dL) and metabolic acidosis (pH 7.1, HCO3 8 mEq/L), which are consistent with diabetic ketoacidosis (DKA). IV insulin is essential to reduce blood glucose; however, starting insulin without addressing hypokalemia (potassium 3.1 mEq/L) could lead to life-threatening hypokalemia, as insulin drives potassium into cells. The administration of IV insulin must be done after ensuring adequate serum potassium levels. Normal saline has been started to treat dehydration and to restore perfusion. Therefore, the next step after initiating fluid resuscitation is to replace potassium intravenously to prevent hypokalemia when insulin therapy begins." + }, + { + "input": "Question: A 63-year-old man with diverticular disease comes to the emergency department because of painless rectal bleeding, dizziness, and lightheadedness for 2 hours. His temperature is 37.6°C (99.6°F), pulse is 115/min, respirations are 24/min, and blood pressure is 86/60 mm Hg. He appears pale. Physical examination shows bright red rectal bleeding. Colonoscopy shows profuse diverticular bleeding; endoscopic hemostasis is performed. After initiating fluid resuscitation, the patient becomes hemodynamically stable. The following day, laboratory studies show:\nHemoglobin 8 g/dL\nLeukocyte count 15,500/mm3\nPlatelet count 170,000/mm3\nSerum\nUrea nitrogen 60 mg/dL\nCreatinine 2.1 mg/dL\nBilirubin\nTotal 1.2 mg/dL\nIndirect 0.3 mg/dL\nAlkaline phosphatase 96 U/L\nAlanine aminotransferase (ALT, GPT) 2,674 U/L\nAspartate aminotransferase (AST, GOT) 2,254 U/L\nWhich of the following cells in the patient's liver were most likely damaged first?\"\nOptions: A: Periportal hepatocytes\tB: Hepatic sinusoidal endothelial cells\tC: Hepatic Kupffer cells\tD: Centrilobular hepatocytes\t\nThe right answer is D: Centrilobular hepatocytes.", + "output": "Option D, Centrilobular hepatocytes, is the most likely answer because the elevated levels of alanine aminotransferase (ALT) and aspartate aminotransferase (AST) indicate significant hepatocellular injury. Centrilobular hepatocytes, located around the central vein in the liver lobule, are most susceptible to hypoxic injury due to their location farthest from the portal triads, which contain the oxygen-rich blood supply. In this case, the patient's acute blood loss and subsequent hypotension would compromise blood flow to the liver, creating a hypoxic environment that preferentially damages the centrilobular hepatocytes. This would release high levels of ALT and AST, as seen in the laboratory results." + }, + { + "input": "Question: A 21-year-old man is brought to the emergency department by his mother after being found lying unconscious next to a bottle of pills. The patient’s mother mentions that he has been diagnosed with major depressive disorder 3 years ago for which he was taking a medication that resulted in only a slight improvement in his symptoms. She says he still found it difficult to concentrate on his studies and did not participate in any social events in college. He didn’t have many friends and was often found sitting alone in his room. He has also threatened to take his life on several occasions, but she did not think he was serious. While recording his vital signs, the patient goes into a coma. His ECG shows a QT interval of 450 milliseconds. Which of the following medications did this patient most likely overdose on?\nOptions: A: Bupropion\tB: Sertraline\tC: Venlafaxine\tD: Clomipramine\t\nThe right answer is D: Clomipramine.", + "output": "Clomipramine, option D, is the likeliest cause of overdose in this scenario. The ECG showing a QT interval of 450 milliseconds is indicative of clomipramine's known side effect of QT prolongation, which can lead to cardiac arrhythmias and coma in overdose situations. Clomipramine is a tricyclic antidepressant (TCA), and TCAs are notorious for their cardiotoxicity in overdoses. Additional information supporting this includes the patient's history of major depressive disorder, poor response to previous medication likely leading to continued use or increased dosage of a potent TCA like clomipramine, social withdrawal, and previously voiced threats of suicide, suggesting intentional overdose as a possibility. Other medications listed are less likely to cause such a significant QT prolongation or are less commonly associated with fatal overdoses." + }, + { + "input": "Question: A 24-year-old woman presents to the emergency department with a severe headache. She states it is 10/10 in intensity and states that it is associated with chewing. She describes it as a dull pain over the sides of her head. The patient is otherwise healthy and is not currently taking any medications. Her temperature is 97.0°F (36.4°C), blood pressure is 111/74 mmHg, pulse is 83/min, respirations are 13/min, and oxygen saturation is 98% on room air. Physical exam is notable for pain and tenderness over the mandibular and temporal region that is worsened when the patient opens and closes their mouth. Which of the following is the most likely diagnosis?\nOptions: A: Migraine headache\tB: Temporal arteritis\tC: Temporomandibular joint dysfunction\tD: Tension headache\t\nThe right answer is C: Temporomandibular joint dysfunction.", + "output": "The patient's symptoms of a severe dull headache associated with chewing and tenderness in the temporal and mandibular regions that worsens with mouth movements are characteristic of temporomandibular joint (TMJ) dysfunction. Migraine is typically associated with pulsating pain, photophobia, or phonophobia, and not specifically with chewing. Temporal arteritis would be unusual in a young person and typically presents with systemic symptoms and markedly elevated inflammatory markers. A tension headache usually manifests as a band-like pain without the specific exacerbation by jaw movement. Therefore, given the provided clinical presentation, TMJ dysfunction is the most likely diagnosis." + }, + { + "input": "Question: A 55-year-old woman presents with severe pruritus and fatigue. She denies any similar symptoms in the past. No significant past medical history. Upon physical examination, scleral icterus is present, and significant hepatosplenomegaly is noted. Mild peripheral edema is also present. Laboratory findings are significant for elevated serum levels of bilirubin, aminotransferases, alkaline phosphatase (ALP), γ-glutamyl transpeptidase (GGTP), immunoglobulins, as well as cholesterol (especially HDL fraction). Antiviral antibodies are not present. The erythrocyte sedimentation rate is also elevated. Anti-mitochondrial antibodies are found. A liver biopsy is performed, and the histopathologic examination shows bile duct injury, cholestasis, and granuloma formation. Which of the following is the most likely diagnosis in this patient?\nOptions: A: Primary biliary cirrhosis\tB: Hemolytic anemia\tC: Hepatitis A\tD: Gilbert syndrome\t\nThe right answer is A: Primary biliary cirrhosis.", + "output": "The most likely diagnosis for this patient is primary biliary cirrhosis (PBC), which is an autoimmune liver disease characterized by progressive destruction of the bile ducts, leading to cholestasis. The symptoms such as pruritus, fatigue, and the presence of scleral icterus along with laboratory findings of elevated bilirubin and cholesterol levels including HDL, point towards a cholestatic liver disease. The absence of viral antibodies rules out viral hepatitis, such as hepatitis A. The presence of anti-mitochondrial antibodies is highly specific for PBC. Furthermore, the liver biopsy showing bile duct injury, cholestasis, and granuloma formation confirms the diagnosis of PBC, distinguishing it from other differential diagnoses." + }, + { + "input": "Question: Three days after undergoing an open cholecystectomy, an obese 57-year-old woman has fever, chills, and a headache. She has right-sided chest pain that increases on inspiration and has had a productive cough for the last 12 hours. She had an episode of hypotension after the operation that resolved with intravenous fluid therapy. She underwent an abdominal hysterectomy 16 years ago for multiple fibroids of the uterus. She has smoked one pack of cigarettes daily for 17 years. She appears uncomfortable. Her temperature is 39°C (102.2°F), pulse is 98/min, respirations are 18/min, and blood pressure is 128/82 mm Hg. Inspiratory crackles are heard at the right lung base. The abdomen is soft and nontender. There is a healing surgical incision below the right ribcage. The remainder of the examination shows no abnormalities. Which of the following is the most likely diagnosis?\nOptions: A: Pulmonary embolism\tB: Subphrenic abscess\tC: Pneumonia\tD: Pneumothorax\t\nThe right answer is C: Pneumonia.", + "output": "The patient presents with fever, chills, productive cough, and inspiratory right-sided chest pain alongside inspiratory crackles at the right lung base, suggesting an infectious process in the lung, typically pneumonia. The development of these symptoms within a few days post-surgery, her history of smoking, and recent general anesthesia, which can impair cough reflex and mucus clearance, all contribute to her increased risk for postoperative pneumonia. Although other options could be possible, the absence of specific signs for pulmonary embolism (e.g., hemoptysis, evidence of DVT), subphrenic abscess (e.g., localized abdominal pain, rebound tenderness), and pneumothorax (e.g., hyperresonance, decreased breath sounds) make pneumonia the most likely diagnosis." + }, + { + "input": "Question: A 58-year-old woman presents to the emergency department with difficulty breathing and a sensation that her heart was racing for the past 3 days. She adds that she has lost weight over the last 7 weeks, despite a good appetite, and is anxious most of the time with difficulty sleeping at night. She has smoked 10 cigarettes per day for the past 15 years. Her blood pressure is 100/55 mmHg, temperature is 36.5°C (97.7°F), and pulse is irregular with a rate of 140–150/min. On physical examination she is thin, frail, and appears anxious. Her palms are sweaty and there are fine tremors on extension of both hands. She has a palpable smooth thyroid mass. Examination of the eyes reveals bilateral exophthalmos. An electrocardiogram is obtained and shown in the picture. Which of the following has a strong positive correlation with this patient’s heart rhythm?\nOptions: A: Digoxin blood level\tB: PR interval\tC: Age\tD: Amiodarone blood level\t\nThe right answer is C: Age.", + "output": "The patient presents with symptoms suggestive of hyperthyroidism, including weight loss with good appetite, anxiety, difficulty sleeping, sweaty palms, fine tremors, and an irregularly high pulse rate. The presence of a smooth thyroid mass and exophthalmos further supports this diagnosis. These symptoms may lead to atrial fibrillation, which correlates with the irregular pulse recorded. Although age can be a risk factor for atrial fibrillation, this patient's presentation points towards a hyperthyroid state as the primary cause. The other options (A. Digoxin and D. Amiodarone blood levels, B. PR interval) do not have a direct linkage to this case or the heart rhythm described. Hyperthyroidism is known to increase the risk of atrial fibrillation, particularly in older patients, making C. Age the best choice among the given options." + }, + { + "input": "Question: A 34-year-old man comes to the physician because of frequent headaches and blurry vision during the past 4 months. He has also had difficulties achieving an erection over the past few weeks. Physical examination shows a temporal visual field deficit bilaterally. An MRI of the brain shows an intrasellar mass. The mass is most likely derived from which of the following types of cells?\nOptions: A: Thyrotrophs\tB: Lactotrophs\tC: Somatotrophs\tD: Gonadotrophs\t\nThe right answer is B: Lactotrophs.", + "output": "The patient's symptoms suggest a condition that is compressing the optic chiasm, causing bilateral temporal visual field deficits (bitemporal hemianopia). This, combined with the location of the mass (intrasellar), is suggestive of a pituitary adenoma. Among the listed cell types, lactotrophs, which secrete prolactin, are most commonly associated with pituitary adenomas that present with the described symptoms. Prolactin-secreting pituitary adenomas can cause headaches and visual disturbances due to pressure on the optic chiasm. They can also cause decreased libido and erectile dysfunction, as hyperprolactinemia can inhibit the secretion of gonadotropin-releasing hormone (GnRH), ultimately leading to reduced testosterone levels. Hence, the most likely derived cells for the intrasellar mass are lactotrophs, making option B the correct choice." + }, + { + "input": "Question: A 38-year-old woman comes to the physician because of a 1-month history of a painless, nonpruritic skin lesion on her right thigh. She initially thought it was an insect bite, but it has slowly increased in size over the past weeks. Her temperature is 36.7°C (98°F), pulse is 75/min, and blood pressure is 128/76 mm Hg. Physical examination shows a 0.8-cm hyperpigmented papule. When the skin lesion is squeezed, the surface retracts inwards. A photograph of the lesion is shown. Which of the following is the most likely diagnosis?\nOptions: A: Dermatofibroma\tB: Cherry hemangioma\tC: Actinic keratosis\tD: Seborrheic keratosis\t\nThe right answer is A: Dermatofibroma.", + "output": "The most likely diagnosis is A. Dermatofibroma, based on the presentation and physical examination findings. Dermatofibroma typically presents as a slow-growing, firm, hyperpigmented papule that can resemble an insect bite. It is often painless and nonpruritic. A distinctive feature of dermatofibroma is the dimpling sign, also known as the \"dimple sign\" or \"Fitzpatrick sign,\" which occurs when the lesion is pinched and the center retracts inwards, as noted in the physical examination. Cherry angiomas are usually bright red, and actinic keratosis and seborrheic keratosis often have different surface characteristics and do not demonstrate this retraction." + }, + { + "input": "Question: A group of investigators is studying the association between a fire retardant chemical used on furniture and interstitial lung disease. They use hospital records to identify 50 people who have been diagnosed with interstitial lung disease. They also identify a group of 50 people without interstitial lung disease who are matched in age and geographic location to those with the disease. The participants' exposure to the chemical is assessed by surveys and home visits. Which of the following best describes this study design?\nOptions: A: Case-control study\tB: Case series\tC: Retrospective cohort study\tD: Randomized controlled trial\t\nThe right answer is A: Case-control study.", + "output": "The described study design is best characterized as a case-control study. This type of study involves comparing individuals with a specific condition (in this case, interstitial lung disease) to those without the condition. The group with the condition are the 'cases,' while the matched individuals without the condition are the 'controls.' By matching participants on age and geographic location, the researchers minimize confounding variables that could otherwise influence the results. The comparison aims to assess prior exposure to the fire retardant chemical, focusing on examining whether such exposure is more common among cases than controls, which could suggest an association with the disease. Other options do not match the described methodology: a case series would not include controls, a retrospective cohort study would follow groups over time, and a randomized controlled trial would involve an intervention." + }, + { + "input": "Question: A 50-year-old man presents to the office with complaints of fever and chills for 4 weeks. He adds that he is fatigued all the time and has generalized weakness. He has drenching night sweats and has had 2 episodes of non-bilious vomiting over the past few days. He traveled to the Netherlands for 4 days a month ago. His symptoms started a few days after he returned home. Laboratory testing revealed the following:\nHemoglobin 11.2 g/dL\nHematocrit 29%\nLeukocyte count 2,950/mm3\nNeutrophils 59%\nBands 3%\nEosinophils 1%\nBasophils 0%\nLymphocytes 31%\nMonocytes 4%\nPlatelet count 60,000/mm3\nUnconjugated bilirubin 12 mg/dL\nAlanine aminotransferase 200 IU/L\nAspartate aminotransferase 355 IU/L\nThe peripheral blood smear showed basophilic ring- and pear-shaped structures inside many red cells and extracellular basophilic rings on Wright-Giemsa staining. Further evaluation revealed parasitemia and a few schistocytes, poikilocytes, and merozoites in tetrad formation. The findings are most consistent with which diagnosis?\nOptions: A: Malaria\tB: Babesiosis\tC: Tuberculosis\tD: Lyme disease\t\nThe right answer is B: Babesiosis.", + "output": "The symptoms suggested by the patient, including fever, chills, fatigue, generalized weakness, night sweats, and vomiting, alongside the presence of anemia, thrombocytopenia, elevated liver enzymes, and recent travel history to the Netherlands, can be associated with several infections. However, the laboratory findings, particularly the identification of basophilic ring- and pear-shaped structures inside many red cells, and extracellular basophilic rings on Wright-Giemsa staining, are indicative of a parasitic infection involving red blood cells. The peripheral blood smear revealing tetrad formation is highly specific for Babesia microti, the causative agent of babesiosis, rather than Plasmodium (malaria), Mycobacterium tuberculosis (tuberculosis), or Borrelia burgdorferi (Lyme disease). Therefore, these hematological and visual diagnosis indicators lead to the conclusion that option B, Babesiosis, is the correct answer." + }, + { + "input": "Question: A 1-month-old girl is brought to the pediatrician by her parents. They are concerned that she becomes lethargic and irritated between meals. They found that feeding her often with small servings helps. She was born at 39 weeks via spontaneous vaginal delivery and is meeting all developmental milestones. Her mother has one brother that occasionally requires blood transfusions. Today, her blood pressure is 55/33 mm Hg, his heart rate is 120/min, respiratory rate is 40/min, and temperature of 37.0°C (98.6°F). On physical exam, the infant is irritated. She is slightly jaundiced. Her heart has a regular rate and rhythm and her lungs are clear to auscultation bilaterally. Her blood work shows normocytic anemia with elevated reticulocyte count and decreased haptoglobin. Sickle cell anemia and other hemoglobinopathies are also ruled out. A Coombs test is negative. Red blood cell osmotic fragility gap is normal. The physician determined that these findings are related to an enzyme deficiency. Which of the following allosterically inhibits this enzyme?\nOptions: A: Alanine\tB: Adenosine monophosphate\tC: Fructose 1,6-bisphosphate\tD: Fructose 2,6-bisphosphate\t\nThe right answer is A: Alanine.", + "output": "The infant's symptoms of lethargy and irritability that improve with frequent feedings coupled with the family history (a maternal uncle needing blood transfusions) suggest an inherited enzymatic defect of red blood cells, particularly pyruvate kinase deficiency, which is the most likely diagnosis given the clinical and laboratory findings. Pyruvate kinase is an enzyme critical for ATP production in glycolysis. Inhibition of pyruvate kinase leads to decreased ATP, hindering the red blood cell's ability to maintain its shape, causing hemolysis as observed with the normocytic anemia, elevated reticulocyte count, and decreased haptoglobin. Alanine acts as an allosteric inhibitor of pyruvate kinase, thus option A is the correct answer. It inhibits the enzyme, therefore, exacerbating the condition when the levels increase between meals, as seen in this patient." + }, + { + "input": "Question: A 51-year-old man presents the emergency room with chest pain. He mentions that the pain started several hours ago and radiates to his left neck and shoulder. He also mentions that he has some difficulty in breathing. He says that he has had similar chest pains before, but nothing seemed to be wrong at that time. He was diagnosed with high cholesterol during that episode and was prescribed medication. He also has a 3-year history of gastritis. The blood pressure is 130/80 mm Hg, respirations are 18/min, and the pulse is 110/min. He seems a little anxious. The physical examination reveals no significant abnormalities. An ECG shows slight changes in the leads. His physician talks to him about the benefits of taking low-dose aspirin daily. Which of the following would be a contraindication to the use of aspirin in this patient?\nOptions: A: ECG changes\tB: Hypercholesterolemia\tC: Gastritis\tD: Increased pulse rate\t\nThe right answer is C: Gastritis.", + "output": "C. Gastritis is the correct answer because aspirin can exacerbate gastric mucosal irritation and lead to an increased risk of gastric or peptic ulcers. In patients with a history of gastritis, using aspirin, especially on a daily basis, could potentially worsen their underlying condition. Aspirin inhibits the production of prostaglandins which protect the stomach lining, and its use can lead to gastrointestinal bleeding. The other options, such as ECG changes, hypercholesterolemia, and increased pulse rate, are not direct contraindications to the use of low-dose aspirin for cardiac protection." + }, + { + "input": "Question: In 2005, a group of researchers believed that prophylactically removing the ovaries and fallopian tubes (bilateral salpingo-oophorectomy) in BRCA-mutation positive women would reduce the chance that they developed breast cancer. To test this hypothesis, they reviewed a database of women who were known to be BRCA-mutation positive and divided the group into those with breast cancer and those without breast cancer. They used data in the registry and surveys about peoples’ surgical history to compare the proportion of each population that had undergone a bilateral salpingo-oophorectomy. Based on this data, they reported that women undergoing the procedure had a lower chance of developing breast cancer later in life with an odds ratio of 0.46. This is an example of what type of study design?\nOptions: A: Meta-analysis\tB: Cross-sectional\tC: Case-control\tD: Genome-wide association study\t\nThe right answer is C: Case-control.", + "output": "The researchers' approach is an example of a case-control study design. This type of study contrasts individuals with a condition (cases, in this instance, women with breast cancer) with those without it (controls, in this scenario, BRCA-mutation positive women without breast cancer). They then look retrospectively to assess exposure to a risk factor, which is, in this case, the bilateral salpingo-oophorectomy. The use of odds ratio also points toward a case-control study, as it is a measure commonly used to compare the likelihood of an event in the case group versus the control group. The design is retrospective and observational, aimed at identifying factors that may contribute to a medical condition by comparing subjects with and without the condition, thus fitting the characteristics of a case-control study." + }, + { + "input": "Question: A 64-year-old man with a history of type 2 diabetes mellitus is referred to a urologist. The patient has had pain when urinating and difficulty starting a stream of urine for over 4 months now. He is bothered because he has to urinate about 9 times every day, including several times when he wakes up at night. A digital rectal examination revealed multiple hard nodules in the prostate gland. A CT scan shows a nodule in the right lung that measures 3 cm. An ultrasound scan of his urinary bladder and prostate shows residual urine of > 200 mL and heterogeneous findings of the prostate gland. Biopsy reveals grade 2 prostate adenocarcinoma. Follow-up 9 months later shows his prostate cancer is well controlled with goserelin. If one considers pulmonary nodules that are the same size as this patient’s, which of the following additional findings on CT scans would most likely prompt the removal of such nodules?\nOptions: A: ‘Popcorn’ pattern\tB: Subsolid component\tC: Extending fine striations\tD: Doubling time of < 1 month\t\nThe right answer is C: Extending fine striations.", + "output": "The patient’s primary diagnosis is grade 2 prostate adenocarcinoma, but the presence of a lung nodule raises concerns of possible metastasis or a second primary cancer. When evaluating lung nodules, certain features on imaging are more suggestive of a malignancy. A 'popcorn' calcification pattern (A) is typically associated with a benign hamartoma. Subsolid components (B) in nodules can be suggestive of malignancy but aren't as definitive as other signs. Extending fine striations (C) can indicate spiculation, which is a characteristic often associated with malignant lesions and hence could prompt removal due to the higher suspicion of cancer. A rapid doubling time of less than one month (D) suggests a highly aggressive tumor, but this finding alone without structural indications of malignancy may not necessarily lead to immediate removal without further diagnostics. Option C, with evidence of spiculation, commonly prompts surgical removal or biopsy because spiculated nodules have a higher likelihood of being malignant." + }, + { + "input": "Question: A 58-year-old man comes to the emergency department because of increasing shortness of breath and a nonproductive cough for the last week. Three weeks ago, he had a fever and a cough for 6 days after he returned from a trip to Southeast Asia. He has had a 4-kg (9-lb) weight loss over the past 3 months. He has bronchial asthma and hypertension. He has smoked one pack of cigarettes daily for 41 years. Current medications include an albuterol inhaler and enalapril. His temperature is 37.6°C (99.7°F), pulse is 88/min, respirations are 20/min, and blood pressure is 136/89 mm Hg. There is dullness to percussion, decreased breath sounds, and decreased tactile fremitus over the left lung base. Cardiac examination shows no abnormalities. Chest x-ray of this patient is most likely to show which of the following?\nOptions: A: Elevation of diaphragm\tB: Ground glass appearance\tC: Widened intercostal spaces\tD: Blunting of costophrenic angle\t\nThe right answer is D: Blunting of costophrenic angle.", + "output": "The patient's symptoms of increasing shortness of breath, a nonproductive cough, and weight loss, combined with his travel history, smoking habits, and pre-existing bronchial asthma, suggest a possible respiratory infection or a complication such as a lung abscess or tuberculosis. The presence of dullness to percussion, decreased breath sounds, and decreased tactile fremitus at the lung base is indicative of pleural effusion or consolidation. This would likely result in the obliteration of the normally sharp costophrenic angle on an x-ray, which is seen as blunting – choice (D). The other options do not correspond as directly with the patient's clinical signs." + }, + { + "input": "Question: A 31-year-old man comes to the physician because of a 4-week history of a painless lump near the left wrist and tingling pain over his left hand. Physical examination shows a transilluminating, rubbery, fixed, non-tender mass over the lateral volar aspect of the left wrist. There is decreased sensation to pinprick on the thumb, index finger, middle finger, and radial half of the ring finger of the left hand. The tingling pain is aggravated by tapping over the swelling. Which of the following adjacent structures is at risk of entrapment if this mass persists?\nOptions: A: Ulnar artery\tB: Flexor pollicis longus tendon\tC: Flexor carpi radialis tendon\tD: Ulnar nerve\t\nThe right answer is B: Flexor pollicis longus tendon.", + "output": "The patient's symptoms and physical exam findings suggest a ganglion cyst, which is often a benign, transilluminating, rubbery lump located near the wrist. The presence of decreased sensation in a distribution consistent with the median nerve, along with tingling pain induced by tapping (Tinel's sign), implies median nerve compression. The flexor pollicis longus tendon is innervated by the median nerve and runs close to the site where ganglion cysts often occur. Therefore, if the mass persists, it may further compress the median nerve affecting the structures it innervates, including the flexor pollicis longus tendon, rather than the ulnar artery (A), flexor carpi radialis tendon (C), or ulnar nerve (D), which are anatomically separate from the site of the described mass." + }, + { + "input": "Question: A 40-year-old patient is brought into the emergency department after suffering a motor vehicle crash where he was pinned underneath his motorcycle for about 30 minutes before a passerby called 911. While evaluating him per your institution's trauma guidelines, you discover pain upon palpation of his right lower extremity which is much larger than his left counterpart. The patient admits to decreased sensation over his right lower extremity and cannot move his leg. There are no palpable dorsalis pedis or posterior tibial pulses on this extremity, and it is colder and paler in comparison to his left side. Measured compartment pressure of his distal right leg is 35 mm Hg. What is the next best step in this patient's care?\nOptions: A: Emergent fasciotomy\tB: Venous doppler\tC: Arteriogram\tD: External fixation\t\nThe right answer is A: Emergent fasciotomy.", + "output": "The patient's clinical presentation is highly suggestive of acute compartment syndrome, which is a surgical emergency. Features such as the severely swollen limb, pain on palpation, decreased sensation, inability to move the leg, absent pulses, and a cold pale appearance are classical. The 35 mm Hg compartment pressure confirms the diagnosis since it is well above the normal range (usually less than 20 mm Hg), and pressures above 30 mm Hg are typically considered an indication for fasciotomy. Immediate fasciotomy (Option A) is essential to relieve the pressure, restore circulation, and prevent irreversible damage to the muscles and nerves within the compartment. Other options, such as venous Doppler, arteriogram, and external fixation, may be useful in different contexts but do not address the urgency to alleviate compartment pressure and are therefore not the next best steps in this scenario." + }, + { + "input": "Question: A 29-year-old female presents to her primary care provider with an aching pain in her left leg that has progressively gotten worse over several days. She recently had an unfortunate sporting incident resulting in large bruises along both legs and lower thighs. An X-ray after the event was negative for fracture. Past medical history is positive for systemic lupus erythematosus. She also has a history of one spontaneous abortion at 12 weeks gestation. Today, her heart rate is 90/min, respiratory rate is 17/min, blood pressure is 119/80 mm Hg, and temperature is 37.0°C (98.6°F). On physical examination, her left leg appears pink and slightly swollen. Homan’s sign is positive. A Doppler ultrasound reveals a thrombus in the left popliteal vein. Her lab results are as follows:\nPlatelets 250,000/mm3\nProthrombin Time 14 sec\nPartial Thromboplastin Time 90 sec\nMixing study (PTT) 89 sec\nWhat is the most likely cause of the patient’s condition?\nOptions: A: Antibodies directed against platelet glycoprotein IIb/IIIa\tB: Antibodies directed against phospholipids\tC: Antibodies directed against endothelial cells\tD: Vitamin K deficiency\t\nThe right answer is B: Antibodies directed against phospholipids.", + "output": "The patient's history of systemic lupus erythematosus (SLE), a known risk factor for antiphospholipid syndrome (APS), combined with a past spontaneous abortion, strengthens the suspicion of APS, which is associated with antibodies against phospholipids (Option B). The laboratory findings of a prolonged partial thromboplastin time (PTT) that does not correct with a mixing study suggest the presence of an inhibitor, such as the lupus anticoagulant, which is a type of antiphospholipid antibody. Since antibodies directed against platelet glycoprotein IIb/IIIa (Option A) typically cause thrombocytopenia and bleeding, and the patient's platelet count is normal, this option is less likely. Phospholipid antibodies are more congruent with the clinical picture than antibodies against endothelial cells (Option C) or vitamin K deficiency (Option D), which usually present with different clinical and laboratory profiles." + }, + { + "input": "Question: A 25-year-old female presents to her primary care physician complaining of double vision. She first started seeing double after a long day at her work as a radiation technologist. Since then, her vision has appeared to worsen in the evenings. She has also felt worsening fatigue despite no change in her work hours or sleep habits. She has a history of type I diabetes mellitus and her most recent hemoglobin A1c was 7.4%. Her family history is notable for Sjogren’s syndrome in her mother and hypertension, hyperlipidemia, and carotid dissection in her father. Her temperature is 98.9°F (37.2°C), blood pressure is 105/70 mmHg, pulse is 75/min, and respirations are 16/min. On examination, her pupils are equally round and reactive to light. Extraocular movements are intact. Mild ptosis is noted bilaterally but worse on the left. A visual acuity test reveals no abnormalities. Which of the following medications is most appropriate in the management of this patient?\nOptions: A: Pyridostigmine\tB: Physostigmine\tC: Hydrocortisone\tD: Azathioprine\t\nThe right answer is A: Pyridostigmine.", + "output": "The patient's symptoms are indicative of myasthenia gravis (MG), particularly given the pattern of muscle weakness that worsens with activity and improves with rest, as in the worsening of her double vision in the evenings. MG is an autoimmune disorder where antibodies attack the acetylcholine receptors at the neuromuscular junction, resulting in muscle weakness. Pyridostigmine, an acetylcholinesterase inhibitor, is a first-line treatment because it increases the concentration of acetylcholine at the neuromuscular junction, hence improving neuromuscular transmission and alleviating symptoms. The patient's history of type I diabetes is consistent with a predisposition to autoimmunity, which could include MG. Physostigmine is not indicated as it can cross the blood-brain barrier and cause central nervous system side effects. Hydrocortisone is a steroid, used for its immunosuppressive properties, but it's not a first-line acute treatment for MG. Azathioprine is an immunosuppressant used for long-term management, not initial treatment. Given the clinical presentation and her work environment, option A, Pyridostigmine, is the most appropriate medication to manage this patient's symptoms." + }, + { + "input": "Question: A 24-year-old woman presents to the emergency department after an episode of altered mental status. She was at a party when she suddenly fell to the floor. Her friends state that she was not responsive and was acting oddly. After the episode, the patient seemed confused and had no memory of the event. The patient has a past medical history of substance abuse, depression, and anxiety. Her temperature is 98.6°F (37.0°C), blood pressure is 117/69 mmHg, pulse is 80/min, respirations are 12/min, and oxygen saturation is 99% on room air. Physical exam reveals a girl who seems confused and responds to questions slowly. Her gait is stable, and she demonstrates 5/5 strength in her upper and lower extremities. Laboratory values and a urine dipstick test are performed as seen below:\n\nToxicology:\nBenzodiazepines: Positive\nMarijuana: Positive\nCocaine: Negative\nPhencyclidine: Negative\n\nUrine Dipstick:\nAppearance: Dark\nRed blood cells: Positive\nGlucose: Negative\n\nWhich of the following is the best next step in management?\nOptions: A: Diazepam\tB: IV fluids and mannitol\tC: Phenytoin\tD: Supportive therapy and observation\t\nThe right answer is B: IV fluids and mannitol.", + "output": "Choosing option B, IV fluids and mannitol, suggests a concern for acute management of potential complications such as rhabdomyolysis, indicated by the dark urine and the presence of red blood cells. The episode of altered mental status could be related to her substance use, and her history plus toxicology results point to recent benzodiazepine and marijuana usage, however, this does not typically result in the urine findings. The supportive therapy (option D) is important, but given the urine findings, one must address the potential for acute kidney injury due to myoglobinuria, and mannitol can be used to help preserve renal function alongside hydration. Options A (Diazepam) and C (Phenytoin) are less preferable because there is no indication of ongoing seizures that would necessitate these medications." + }, + { + "input": "Question: An 8-year-old boy is brought to the physician because he has excessively dry skin. His mother says that he has had dry, scaly skin since he was 6 months old. She says it is worse in winter and better in summer. There is no associated itching, redness, or discharge. Examination reveals generalized fine scales with coarse skin all over the body, except in the skin creases of the neck and in the cubital and popliteal fossae (see image). There are multiple papular elevations at the base of hair follicles and darkening of the palmar and plantar creases. Which of the following is the most likely diagnosis?\nOptions: A: Psoriasis\tB: Lichen simplex\tC: Ichthyosis vulgaris\tD: Pityriasis versicolor\t\nThe right answer is C: Ichthyosis vulgaris.", + "output": "Ichthyosis vulgaris is the most likely diagnosis for a couple of reasons. Firstly, the chronic nature of the condition, with onset in infancy and a distinctive worsening during winter months, aligns with known presentation patterns of ichthyosis vulgaris. Additionally, the presentation of dry, scaly skin without significant inflammation or discharge also points towards this condition. The presence of scales that spare the skin creases and the association with hyperlinearity of the palms and soles are characteristically found in ichthyosis vulgaris. The other conditions listed do not typically present with this pattern of generalized scaling without significant inflammation or localized to hair follicles, which is referred to as keratosis pilaris and commonly seen in ichthyosis vulgaris." + }, + { + "input": "Question: A mother brings her 6-year-old daughter to the pediatrician to discuss her child's sleeping troubles. Three times now in the last month the child has begun screaming and crying in the middle of the night. When the parents rush to the child's room, they find her crying inconsolably. They try to ask her what is wrong, but she does not respond and persists crying for approximately 20 minutes, after which she goes back to sleeping comfortably. Upon awaking the following morning, she has no recollection of the night's events. Which of the following is true about these episodes?\nOptions: A: They will be persistent into adulthood\tB: They occur only during non-REM sleep\tC: They are associated with child abuse\tD: They are associated with later development of a mood disorder\t\nThe right answer is B: They occur only during non-REM sleep.", + "output": "The described episodes are indicative of night terrors, which occur during non-REM (Rapid Eye Movement) sleep. Night terrors, or sleep terrors, are a type of sleep disorder occurring within the first few hours of falling asleep, during the transition from deeper non-REM sleep stages to lighter sleep stages. They are generally not associated with underlying psychological issues or child abuse, and while they can be distressing, they typically diminish as children grow older and do not necessarily persist into adulthood. There is no strong evidence that they are directly associated with the later development of mood disorders. Thus, option B is the correct answer." + }, + { + "input": "Question: Two hours after admission to the hospital for dialysis, a 63-year-old woman has severe pain in her lower back. The pain is 8 out of 10 in intensity and radiates down the legs. She underwent a laparoscopic cholecystectomy 4 years ago. She has hypertension, chronic kidney disease, type 2 diabetes mellitus, and major depressive disorder. She lives with her daughter and reports frequent fights with her at home. Her current medications include sertraline, insulin, enalapril, sodium bicarbonate, and sevelamer. She appears uncomfortable. Her temperature is 37.3°C (99.1°F), pulse is 102/min, respirations are 15/min, and blood pressure is 132/94 mm Hg. There is severe tenderness on palpation over the L2 vertebra; range of motion is limited. Neurologic examination shows no focal findings. Laboratory studies show:\nHemoglobin 10.1 g/dL\nLeukocyte count 8,700/mm3\nPlatelet count 130,000/mm3\nSerum\nNa+ 136 mEq/L\nCl- 101 mEq/L\nK+ 5.1 mEq/L\nCalcium 8.2 mg/dL\nUrea nitrogen 33 mg/dL\nGlucose 190 mg/dL\nCreatinine 3.8 mg/dL\nAn x-ray of the spine shows a wedge compression fracture of the L2 vertebra. Which of the following is the most likely explanation for these findings?\"\nOptions: A: Tertiary hyperparathyroidism\tB: Secondary hyperparathyroidism\tC: Elder abuse\tD: Primary hyperparathyroidism\t\nThe right answer is B: Secondary hyperparathyroidism.", + "output": "The most likely explanation for the findings is secondary hyperparathyroidism (Option B). This patient has chronic kidney disease (CKD), which is indicated by the elevated creatinine level. In CKD, the kidneys fail to convert enough vitamin D into its active form and also fail to excrete phosphate effectively. This leads to hypocalcemia and hyperphosphatemia which, in turn, stimulates the parathyroid glands to secrete more parathyroid hormone (PTH). The increased PTH attempts to correct the low calcium levels by increasing bone resorption, which can lead to weakening of the bones and the presented wedge compression fracture of the L2 vertebra due to the increased fragility. Tertiary hyperparathyroidism (Option A) involves hyperplasia of the parathyroid glands and high calcium levels; elder abuse (Option C) could potentially cause fractures but is not suggested by the given biochemical results; primary hyperparathyroidism (Option D) typically presents with hypercalcemia, which is not seen here." + }, + { + "input": "Question: An investigator is studying the function of different enzymes in various human cell types. A subset of normal human cells with an elevated intracellular concentration of bisphosphoglycerate mutase is harvested and isolated. Which of the following is most likely to be absent in these cells?\nOptions: A: α-ketoglutarate dehydrogenase\tB: Pyruvate kinase\tC: Glutathione reductase\tD: Carbonic anhydrase\t\nThe right answer is A: α-ketoglutarate dehydrogenase.", + "output": "Bisphosphoglycerate mutase is an enzyme involved in the glycolytic pathway, specifically in the regulation of oxygen release from hemoglobin by modifying 2,3-bisphosphoglycerate (2,3-BPG) levels. Elevated levels of this enzyme would indicate a cell's heightened demand for oxygen release or an adaptation to hypoxic conditions. α-ketoglutarate dehydrogenase, listed as option A, is part of the citric acid cycle (TCA cycle), which is linked to oxygen utilization in cellular respiration. When cells favor glycolysis over aerobic respiration (possibly due to hypoxic conditions), the TCA cycle would be downregulated, making it reasonable to predict the absence of TCA cycle enzymes like α-ketoglutarate dehydrogenase in these cells." + }, + { + "input": "Question: A 56-year-old woman is brought to the emergency department because of a 2-day history of fever, chills, nausea, and flank pain. She has multiple sclerosis and a chronic indwelling bladder catheter for neurogenic bladder dysfunction. One week ago, she was discharged from the hospital after treatment for a multiple sclerosis flare. Her temperature is 39.3°C (102.8°C). Physical examination shows marked tenderness in the right costovertebral area. Urine cultures grow a non-lactose fermenting, oxidase-positive, gram-negative rod. The causal organism of this patient's current condition most likely produces which of the following substances?\nOptions: A: Lipoteichoic acid\tB: K capsule\tC: Toxin B\tD: Pyoverdine\t\nThe right answer is D: Pyoverdine.", + "output": "The causal organism indicated by the scenario is Pseudomonas aeruginosa, a common non-lactose fermenting, oxidase-positive, gram-negative rod that can cause urinary tract infections, especially in individuals with indwelling catheters. Pseudomonas is known to produce a siderophore called pyoverdine, which aids in iron acquisition from the host, crucial for bacterial growth and virulence. The patient's symptoms, fever, chills, nausea, and flank pain are consistent with a Pyelonephritis, often caused by Pseudomonas in catheterized patients. Options A (Lipoteichoic acid) and C (Toxin B) are associated with Gram-positive bacteria and Clostridioides difficile, respectively. Option B (K capsule) denotes a capsule type (not a specific substance) found in various bacteria, including E. coli, but it is not specific to the organism implied by the question. Pyoverdine (Option D) is the correct choice because it is a substance produced by Pseudomonas aeruginosa." + }, + { + "input": "Question: A 5-year-old Syrian immigrant and her family present for an immigration physical. Her mother describes photophobia, bilateral lacrimation, eye itching, and eyelid swelling for the last week. She is known to have an allergy to cephalosporins. She was born at 39 weeks gestation via spontaneous vaginal delivery. She has met all of her developmental milestones and most of her vaccines. Her vital signs are as follows: blood pressure 105/60 mm Hg, heart rate 98/min, respiratory rate 18/min, and temperature 37.0°C (98.6°F). The physical examination is mostly normal except for her eye exam as presented in the image below. Which of the following statements is true regarding treatment of this patient’s condition?\nOptions: A: A third-generation cephalosporin would be the best treatment option in case the child did not have an actual allergy.\tB: The child should be treated with topical bacitracin.\tC: The best treatment option is oral tetracycline.\tD: The child should be administered a single dose of oral azithromycin.\t\nThe right answer is D: The child should be administered a single dose of oral azithromycin..", + "output": "The child presents with symptoms typical of bacterial conjunctivitis, such as photophobia, bilateral lacrimation, eye itching, and eyelid swelling. Treatment for bacterial conjunctivitis often includes antibiotics. The stated allergy to cephalosporins eliminates option A, even though third-generation cephalosporins might be effective in the absence of an allergy. Topical bacitracin, as per option B, is not the best treatment for conjunctivitis, as it's primarily used for superficial skin infections. Oral tetracycline (option C) is not recommended for children under 8 years old due to the risk of permanent tooth discoloration and enamel hypoplasia. Therefore, the correct answer is D, a single dose of oral azithromycin, which is effective against the common bacterial causes of conjunctivitis and is safe for use in children. It is a broad-spectrum antibiotic and is preferred in this case due to its ease of administration and recommended dose regimen for pediatric patients." + }, + { + "input": "Question: Public health researchers are reviewing data from an outbreak of hepatitis A that was traced to contaminated produce from a popular restaurant. A case series investigation with prospective and retrospective follow-up found that 500 individuals dined at the restaurant during the week of study. The incidence of hepatitis A infection was found to be 25%, and the total number of deaths recorded among the incident cases was 5 individuals. What was the case-fatality rate for the hepatitis A outbreak?\nOptions: A: 4%\tB: 10%\tC: 20%\tD: 25%\t\nThe right answer is A: 4%.", + "output": "The case-fatality rate is calculated by dividing the number of deaths by the number of individuals with the disease and then multiplying by 100 to get a percentage. Here, there were 5 deaths among the incident cases of hepatitis A. With an incidence rate of 25%, 500 diners led to 125 cases of hepatitis A (500 x 0.25). To find the case-fatality rate, we divide the number of deaths (5) by the number of hepatitis A cases (125), which equals 0.04. Multiply by 100 to convert to a percentage, which gives us a 4% case-fatality rate, corresponding to Option A." + }, + { + "input": "Question: A 23-year-old woman visits her obstetrician for an antenatal appointment. She is at the 24th week of gestation. She has had type 1 diabetes mellitus for 5 years and is on insulin. She also has mitral stenosis, for which she has been advised to undergo a balloon valvotomy. Her temperature is 37.1°C (98.7°F), blood pressure is 120/60 mm Hg, and pulse is 90/minute. Her random blood glucose is 220 mg/dL and HbA1C is 8.5%. She wants to discuss possible complications concerning her pregnancy. Which of the following cardiac complications is her infant at high risk for?\nOptions: A: Hypertrophic obstructive cardiomyopathy\tB: Transient hypertrophic cardiomyopathy\tC: Tetralogy of Fallot\tD: Mitral valve prolapse\t\nThe right answer is B: Transient hypertrophic cardiomyopathy.", + "output": "Transient hypertrophic cardiomyopathy is the most likely complication for the infant in a scenario where the mother has poorly controlled type 1 diabetes mellitus. This condition is associated with fetal hyperglycemia that results from the mother's high blood glucose levels. The fetal beta-cell hyperplasia and subsequent insulin release can lead to cardiac hypertrophy, particularly of the interventricular septum. The high HbA1C level of 8.5% indicates poor glycemic control over time, which increases the risk of this fetal cardiac complication. The other options, while possible cardiac anomalies, are not directly associated with maternal diabetes." + }, + { + "input": "Question: An 18-year-old woman is brought to the emergency department because of lightheadedness and a feeling of dizziness. She has had nausea, occasional episodes of vomiting, myalgia, and a generalized rash for the past week. She also reports feeling lethargic. She has no shortness of breath. There is no family history of serious illness. She appears ill. Her temperature is 39.1°C (102.3°F), pulse is 118/min, and blood pressure is 94/60 mm Hg. Cardiac examination shows no abnormalities. There is a widespread erythematous rash on the trunk and extremities with skin peeling on the palms and soles. Laboratory studies show:\nHemoglobin 13.6 g/dL\nLeukocyte count 19,300/mm3\nPlatelet count 98,000/mm3\nSerum\nUrea nitrogen 47 mg/dL\nGlucose 88 mg/dL\nCreatinine 1.8 mg/dL\nTotal bilirubin 2.1 mg/dL\nAST 190 U/L\nALT 175 U/L\nUrinalysis shows no abnormalities. Further evaluation of this patient's history is most likely to reveal which of the following?\"\nOptions: A: Recent hiking trip\tB: Intravenous heroin abuse\tC: Exposure to a patient with mengingococcemia\tD: Currently menstruating\t\nThe right answer is D: Currently menstruating.", + "output": "The patient’s symptoms and laboratory findings suggest a diagnosis of toxic shock syndrome (TSS), which is known to be associated with tampon use, especially if left in place for a prolonged period. The widespread erythematous rash, desquamation on the palms and soles, and fever, along with multi-organ involvement (elevated liver enzymes, renal impairment, thrombocytopenia), are characteristic of TSS. This condition is more common among menstruating women and can be caused by superantigen exotoxins produced by Staphylococcus aureus or Streptococcus pyogenes. Option D (Currently menstruating) is the most likely scenario to reveal in her history that would explain her clinical presentation. Options A, B, and C are less likely because the clinical picture and lab results are more specifically indicative of TSS." + }, + { + "input": "Question: A 39-year-old woman comes to the physician because of fever, generalized fatigue, and chills for 1 week. She has been having dull right-side abdominal pain during this period. She has nausea. She has no history of travel outside the United States. She has type 1 diabetes mellitus. Current medications include insulin and an oral contraceptive. She appears ill. Her temperature is 40°C (104°F), pulse is 118/min, and blood pressure is 106/70 mm Hg. On examination, the liver is palpated 2–3 cm below the right costal margin and is tender. There are decreased breath sounds over the right lung base. The remainder of the examination shows no abnormalities. Laboratory studies show:\nHemoglobin 14.1 g/dL\nLeukocyte count 17,100/mm3\nSegmented neutrophils 74%\nEosinophils 1%\nLymphocytes 20%\nMonocytes 5%\nSerum\nGlucose 79 mg/dL\nCreatinine 1.1 mg/dL\nTotal bilirubin 0.9 mg/dL\nAlkaline phosphatase 180 U/L\nAST 69 U/L\nALT 100 U/L\nA right upper quadrant ultrasound is shown. Which of the following is the most appropriate next step in management?\"\nOptions: A: Discontinue the oral contraceptive\tB: Obtain hepatoiminodiacetic acid scan\tC: Perform endoscopic retrograde cholangiopancreatography\tD: Perform percutaneous drainage\n\"\t\nThe right answer is D: Perform percutaneous drainage\n\".", + "output": "The patient presents with signs and symptoms of an intra-abdominal infection, likely a hepatic abscess given the fever, right-sided abdominal pain, tender hepatomegaly, and elevated inflammatory markers (leukocytosis with neutrophilia). This is supported by the decreased breath sounds over the right lung base, indicating a possible subphrenic abscess affecting the lower lung. The right upper quadrant ultrasound likely confirmed the presence of an abscess. Percutaneous drainage (Option D) is the most appropriate management for a hepatic abscess, offering both diagnostic information (by obtaining cultures of the aspirated material) and therapeutic relief by draining the infected fluid. The other options are less appropriate given the current clinical scenario and likely diagnosis. Discontinuing the oral contraceptive (Option A) is not immediately pertinent, obtaining a hepatoiminodiacetic acid scan (Option B) is used for diagnosing cholecystitis which is not suggested here, and performing endoscopic retrograde cholangiopancreatography (Option C) is indicated for pancreaticobiliary diseases, which the presentation does not primarily suggest." + }, + { + "input": "Question: A 3-year-old boy is brought to the emergency department by ambulance after a motor vehicle accident. He is unconscious upon arrival and is found to have severe internal bleeding due to trauma. He is taken for emergency surgery and severely damaged tissues are removed. After surgery, the boy is taken into the PICU for recovery. Over the subsequent week, serial labs are drawn showing target cells, thrombocytosis, and leukocytosis. He is started on a prophylactic regimen that is continued upon discharge. Which of the following is consistent with the mechanism of action of the drug that was most likely prescribed in this case?\nOptions: A: 30S ribosome inhibitor\tB: 50S ribosome inhibitor\tC: Cell wall cross-linking inhibitor\tD: DNA gyrase inhibitor\t\nThe right answer is C: Cell wall cross-linking inhibitor.", + "output": "The boy likely received a prophylactic antibiotic regimen to prevent a bacterial infection following surgery, especially considering the conditions like severe trauma and tissue removal, which can predispose to infection. Option C, \"Cell wall cross-linking inhibitor,\" is consistent with the mechanism of action for beta-lactam antibiotics, which includes penicillins and cephalosporins. These antibiotics are commonly used prophylactically in surgical patients to prevent infections due to their broad spectrum of activity and efficacy in preventing surgical site infections. Options A and B represent mechanisms of action for different classes of antibiotics, and option D (DNA gyrase inhibitor) corresponds to fluoroquinolones, which are not typically used as first-line prophylactic agents in pediatric surgical patients." + }, + { + "input": "Question: A 5-year-old girl is brought to her pediatrician for evaluation of difficulties noted at school. Her kindergarten teacher voiced her concern that the child ‘spaces out’ often and thinks she may have ADHD. The parents deny any history of head trauma or infection. Her blood pressure is 106/74 mm Hg, the heart rate is 69/min, and the respiratory rate is 14/min. Physical exam demonstrates a happy child who has reached developmental milestones. Laboratory results are within normal limits. EEG demonstrates a 3-Hz spike and wave pattern (see image). The decision is made to start the child on medical therapy. What should be advised to the parents as a potential side effect of the first-line medication?\nOptions: A: Nystagmus\tB: GI upset\tC: Facial rash and blood in the urine\tD: Weight gain and change in hair color\t\nThe right answer is B: GI upset.", + "output": "The EEG findings of a 3-Hz spike and wave pattern are indicative of absence seizures, a type of generalized epilepsy commonly seen in children. The first-line medication for absence seizures is often ethosuximide, which has a well-known side effect profile that includes gastrointestinal (GI) upset, such as nausea, vomiting, and abdominal pain. While other antiepileptic drugs (AEDs) have their own side effect profiles, such as nystagmus (dose-related with phenytoin), facial rash and blood in the urine (potential signs of an allergic reaction to medications like lamotrigine or with renal issues), and weight gain and change in hair color (seen with valproic acid), the most common and well-recognized side effect associated with ethosuximide is GI upset, which is why it is the advised potential side effect to the parents (Option B)." + }, + { + "input": "Question: While traveling abroad a physician is asked to attend a meeting regarding healthcare in the region. The rate of chlamydial infection is exceptionally high in the area, and the local government is wanting to test a new treatment for these infections. To test the new therapy, the physician is asked to oversee the operations of testing this new treatment. When asking what this would entail, the officials tell the physician that they plan to infect the local prison population with chlamydia and then test the new treatment on these individuals. Which of the following is the best response from the physician?\nOptions: A: “I can not help you due to the ethical principle of autonomy.”\tB: “I can not help you due to the ethical principle of justice.”\tC: “I can not help you due to the ethical principle of nonmaleficence.”\tD: “I can not help you due to the ethical principle of beneficence.”\t\nThe right answer is C: “I can not help you due to the ethical principle of nonmaleficence.”.", + "output": "Option C, \"I cannot help you due to the ethical principle of nonmaleficence,\" is the most appropriate choice as the physician recognizes the proposal to intentionally infect individuals with disease violates the ethical principle of \"do no harm.\" Nonmaleficence is a fundamental ethical principle in healthcare that mandates healthcare providers to avoid causing harm to patients. Deliberately infecting people, even if they are prisoners, with a disease for the purpose of testing a treatment directly contradicts this principle. It is unethical and harmful to use individuals as test subjects against their will and without their informed consent, which could lead to legal and moral repercussions. This principle takes precedence over other considerations, making it the primary reason for refusing participation." + }, + { + "input": "Question: Three patients present to the pediatrician for routine well-child visits. The first child’s mother reports that he seems to be breastfeeding well and voids at least eight times per day. His stools are a yellow color, and he does not seem to be straining. In the office, he can lift his head and chest when in the prone position, and he can track an object with his eyes to the midline. His hands remain fisted 50% of the time.\n\nThe second child’s parents report that she is doing well at home and seems to enjoy playing with her older brother. In the office, she can roll from supine to prone and can transfer her rattle from hand to hand. She cannot yet say any words.\n\nThe third child’s father reports that he is starting to explore the world around him. He has begun taking independent steps at home. In the office, he seems to recognize the name of several objects, but he cannot follow the direction to “grab the ball.” All three children are assessed as developmentally normal.\n\nWhich of the following represents these patient's respective ages?\nOptions: A: Ages 1 month, 4 months, and 9 months\tB: Ages 2 months, 4 months, and 12 months\tC: Ages 2 months, 6 months, and 9 months\tD: Ages 2 months, 6 months, and 12 months\t\nThe right answer is D: Ages 2 months, 6 months, and 12 months.", + "output": "The first child's ability to lift his head and chest and track objects to the midline suggests motor skills typical of a 2-month-old, and fisted hands 50% of the time is normal for this age. The second child's ability to roll from supine to prone and transfer objects between hands suggests development typical of a 6-month-old, although the lack of speech is not concerning at this age. The third child's beginning to take independent steps and recognition of object names suggests developmental milestones of a 12-month-old, despite not being able to follow a direction to \"grab the ball,\" which may not be fully developed until closer to 18 months. Therefore, option D, which lists ages 2 months, 6 months, and 12 months, is the correct answer for the respective ages of the patients." + }, + { + "input": "Question: A 45-year-old African-American woman comes to the physician after an episode of hemoptysis. Over the past 8 months, she has had a nonproductive cough, fatigue, and a 9-kg (20-lb) weight loss. She drinks a glass of wine every night and does not smoke. She has a history of hypertension and osteoarthritis. She does not take any medications. She has never traveled out of the country. She works as an accountant. Her temperature is 37.0°C (98.6°F), pulse is 94/min, and blood pressure is 130/90 mm Hg. Lungs are clear to auscultation. An x-ray of the chest shows a 2.5-cm nodule with irregular borders at the apex of the left lung with no mediastinal or hilar enlargement. Which of the following is the most likely diagnosis?\nOptions: A: Carcinoid tumor\tB: Sarcoidosis\tC: Squamous cell carcinoma of the lung\tD: Adenocarcinoma of the lung\t\nThe right answer is D: Adenocarcinoma of the lung.", + "output": "The most likely diagnosis, given the patient's symptoms and radiographic findings, is adenocarcinoma of the lung (Option D). The patient has experienced significant weight loss, fatigue, and a prolonged cough, which are common symptoms associated with lung cancer. Hemoptysis is a concerning symptom often associated with malignancies. The fact that the patient has a nodule with irregular borders, which is a typical characteristic of malignancy, makes lung cancer a more likely diagnosis than benign conditions like sarcoidosis. Adenocarcinoma is the most common type of lung cancer in non-smokers, particularly women, which fits the patient's profile. Carcinoid tumors are usually less aggressive and associated with a better prognosis, less often presenting with systemic symptoms like weight loss. While squamous cell carcinoma is also a possibility, adenocarcinoma more commonly presents as peripheral lung nodules, especially in non-smokers." + }, + { + "input": "Question: A 70-year-old man with hyperlipidemia and hypertension returns to his cardiologist for ongoing blood pressure management. He reports feeling fine with no trouble with his medications. He exercises regularly and sleeps well without snoring or daytime somnolence. His blood pressure measurements in the past have ranged from 160 - 170/80 - 100 mmHg. Today, his pressure is 150/100 mmHg with a pulse of 65/min while on hydrochlorothiazide and ramipril. Physical exam reveals a II/VI early systolic murmur with split S2 is appreciated at the right upper sternal border. There is unremarkable carotid, renal, or abdominal bruits. What is the most common cause of his hypertension?\nOptions: A: Primary hyperaldosteronism\tB: Hypothyroidism\tC: Stiffening of the aorta\tD: Aortic stenosis\t\nThe right answer is C: Stiffening of the aorta.", + "output": "The most common cause of hypertension in an elderly patient like the one described is stiffening of the aorta (option C). As people age, their blood vessels tend to lose elasticity, which can lead to increased systemic vascular resistance and subsequently, hypertension. This can be exacerbated by other factors like hyperlipidemia. While aortic stenosis (option D) may cause hypertension and can present with an early systolic murmur, it isn't the most common cause of hypertension in the elderly. Primary hyperaldosteronism (option A) and hypothyroidism (option B) are less likely given the patient's stable condition, absence of symptoms related to these conditions, and no evident abnormalities upon physical examination that would suggest these diagnoses. The patient's regular exercise, good sleep patterns, and lack of other systemic findings further support stiffening of the aorta as the most probable underlying cause of his hypertension." + }, + { + "input": "Question: A 68-year-old man presents with a 6-month history of worsening fatigue and weight loss. He is especially concerned about a recent episode in which he noticed a blurring of his vision and headache; he also notes that his “memory is not as good as it used to be,” which his wife confirms. Physical exam is notable only for pallor and general weakness. Laboratory studies reveal anemia as well as an elevated erythrocyte sedimentation rate. Serum protein electrophoresis (SPEP) reveals a sharp, narrow spike of monoclonal IgM; serum IgM concentration is 6.3 g/dL. What is the most likely diagnosis in this case?\nOptions: A: IgM monoclonal gammopathy of undetermined significance (MGUS)\tB: Multiple myeloma\tC: Waldenstrom’s macroglobulinemia\tD: Non-Hodgkin’s lymphoma\t\nThe right answer is C: Waldenstrom’s macroglobulinemia.", + "output": "The most likely diagnosis in this case is Waldenstrom’s macroglobulinemia (C). This conclusion is drawn considering the patient's symptoms of fatigue, weight loss, visual disturbances, headache, and cognitive changes, combined with laboratory findings of anemia, elevated erythrocyte sedimentation rate (ESR), and a sharp monoclonal IgM spike on serum protein electrophoresis. Waldenstrom’s macroglobulinemia is characterized by the proliferation of B cells that secrete monoclonal IgM, responsible for the hyperviscosity syndrome that can lead to the patient's symptoms. While MGUS and multiple myeloma also involve monoclonal gammopathies, they typically present with different patterns on SPEP and clinical features. Non-Hodgkin’s lymphoma is less likely given the specific finding of elevated monoclonal IgM." + }, + { + "input": "Question: A 40-year-old woman presents to the emergency department with severe left upper quadrant pain (duration 3 hours, stabbing quality, 10/10 on the pain scale). Past medical history is significant for sickle cell anemia. Physical examination is significant for severe tenderness to palpation in the left upper quadrant. Significant splenomegaly is also noted. The patient is admitted to the hospital for close observation and placed on deep vein thrombosis (DVT) prophylaxis as part of a routine protocol. Laboratory findings drawn sometime after admission demonstrate a normal prothrombin time (PT) and elevated partial thromboplastin time (PTT). Which of the following factors is most directly affected by the DVT prophylaxis?\nOptions: A: VII\tB: VIIa\tC: X\tD: XII\t\nThe right answer is C: X.", + "output": "The most common form of DVT prophylaxis in a hospitalized patient is the administration of low molecular weight heparin (LMWH) or unfractionated heparin. Heparins primarily enhance the activity of antithrombin III, which in turn inactivates several clotting factors, particularly factor Xa. The elevation of the partial thromboplastin time (PTT) is due to the inhibition of factor Xa, given that PTT is a test that reflects the integrity of the intrinsic and common coagulation pathways, where factor X is a key component. Therefore, out of the listed options, factor X (option C) is most directly affected by the DVT prophylaxis, in this case, due to heparin administration. Factors VII, VIIa, and XII are less directly affected or related to the extrinsic pathway and prekallikrein respectively, making C the correct choice." + }, + { + "input": "Question: A 16-year-old female presents to her physician’s office after noticing a round lump in her left breast 2 months ago. She reports that the lump seemed to enlarge and became tender just preceding her last 2 menses. It is otherwise painless, and the patient denies any discharge or skin changes. She has no past medical history but her grandmother, age 72, was just diagnosed with invasive ductal carcinoma of the breast. The patient is an avid softball player at her high school and denies alcohol, smoking, or illicit drug use. On exam, the breasts appear symmetric and normal. A 3-cm round, mobile mass is palpated in the upper outer quadrant of the left breast. There is slight tenderness to deep palpation of the mass. There is no axillary lymphadenopathy on either side. Which of the following is the most likely outcome of this patient’s condition?\nOptions: A: This mass will decrease in size if the patient starts oral contraceptives\tB: This mass slightly increases this patient’s risk of breast cancer in the future\tC: This mass will most likely decrease in size or disappear over time\tD: If this mass grows rapidly to greater than 5 cm, radiation and chemotherapy are indicated\t\nThe right answer is C: This mass will most likely decrease in size or disappear over time.", + "output": "The most likely outcome for this patient is option C, that the mass will most likely decrease in size or disappear over time. This prediction is based on the fact that the mass fluctuates with the patient's menstrual cycle, indicating it could be fibrocystic in nature – a common and typically benign condition. Such lumps often change in size and tenderness in response to hormonal fluctuations and can resolve spontaneously. There is no immediate family history of breast cancer, and her age, plus the lack of concerning features (e.g., fixed mass, nipple discharge, or lymphadenopathy), also suggest a benign process. Options A and B do not align with the history or presentation, and option D is inconsistent with the current clinical evidence, reserved for aggressive malignant conditions, which this does not appear to be." + }, + { + "input": "Question: A 28-year-old woman presents to her primary care physician for evaluation of amenorrhea. Her last period occurred 4 months ago but she has not had sex in the last year and home pregnancy tests have come back negative. She first started having periods at age 13 and her periods have been unremarkable up until they stopped 4 months ago. Her past medical history is significant for radius and ulnar forearm fractures that she sustained in a car crash 3 months ago. She didn't see the car coming and has had increasing difficulty noticing objects in her peripheral vision. She also notes having recurrent headaches 5 months ago. Otherwise she has been healthy with no previous pregnancies. Physical exam reveals galactorrhea and temporal field visual defects. Which of the following mechanisms is most likely responsible for this patient's amenorrhea?\nOptions: A: Inhibition of gonadotropin-releasing hormone release\tB: Intrauterine scar tissue accumulation\tC: Nondisjunction of chromosomes\tD: Nutritional imbalance\t\nThe right answer is A: Inhibition of gonadotropin-releasing hormone release.", + "output": "The patient's amenorrhea, galactorrhea, and temporal field visual defects suggest a pituitary adenoma, likely a prolactinoma. Prolactinomas secrete excess prolactin, which inhibits gonadotropin-releasing hormone (GnRH) release. This inhibition in turn decreases the secretion of follicle-stimulating hormone (FSH) and luteinizing hormone (LH), leading to amenorrhea. The recurrent headaches and visual field defects can be explained by the mass effect of the tumor on local structures, such as the optic chiasm. Option B is unlikely without a history of intrauterine procedures, option C is unrelated to her symptoms, and the patient's nutritional history hasn't been discussed, making D less likely." + }, + { + "input": "Question: A 66-year-old man presents to the emergency department for a cough and fatigue. The patient was brought in from a nursing home with documentation stating that he has seemed confused for the past day according to the staff. The patient has a past medical history of diabetes and hypertension. He is currently taking insulin, metformin, lisinopril, and atorvastatin. His temperature is 102°F (38.9°C), blood pressure is 107/58 mmHg, pulse is 120/min, respirations are 15/min, and oxygen saturation is 98% on room air. Physical exam reveals crackles on pulmonary exam and S4 on cardiac auscultation. Which of the following is the next best step in management?\nOptions: A: Azithromycin and admission to the medical floor\tB: Azithromycin and discharge\tC: Azithromycin, moxifloxacin, and admission to the intensive care unit\tD: Moxifloxacin and admission to the medical floor\t\nThe right answer is D: Moxifloxacin and admission to the medical floor.", + "output": "The patient presents with signs of a probable lower respiratory tract infection, suggested by cough, fever, and pulmonary crackles, and also has systemic symptoms like fatigue and confusion, which in the elderly can indicate a serious infection. His vital signs show tachycardia and a fever but normal oxygen saturation on room air, which does not necessarily indicate an immediate need for intensive care. Therefore, aggressive outpatient management (option B) is not adequate, and ICU admission (option C) may not be warranted yet. His comorbidities of diabetes and hypertension, as well as the current presentation, suggest that an atypical organism could be responsible; thus, azithromycin alone (option A) may not cover potential resistant organisms or typical pathogens causing community-acquired pneumonia. Moxifloxacin is a fluoroquinolone with good coverage for both typical and atypical pathogens and is recommended for patients with comorbidities. Hence, the next best step is to admit the patient to the medical floor for further observation, monitoring, and treatment with moxifloxacin (option D)." + }, + { + "input": "Question: A 45-year-old woman with hypothyroidism comes to the physician because of progressive fatigue, lethargy, and epigastric pain after eating. Physical examination shows pale conjunctivae. Laboratory studies show decreased serum hemoglobin levels and increased serum gastrin levels. Esophagogastroduodenoscopy shows inflammation of the gastric body and fundus. A photomicrograph of a biopsy specimen taken from the gastric antrum is shown. Which of the following is the most likely cause of this patient's symptoms?\nOptions: A: Mucosal cell hyperplasia\tB: Enterochromaffin-like cell hyperplasia\tC: Parietal cell destruction\tD: Chief cell destruction\t\nThe right answer is C: Parietal cell destruction.", + "output": "The patient's symptoms of progressive fatigue, lethargy, and pale conjunctivae, along with her laboratory findings of decreased hemoglobin and increased serum gastrin levels, suggest anemia and possibly an autoimmune condition affecting the stomach. The inflammation seen in the gastric body and fundus during the esophagogastroduodenoscopy, along with the history of hypothyroidism, which is an autoimmune disease, points toward an autoimmune gastritis. Parietal cell destruction is a common outcome of autoimmune gastritis, leading to reduced intrinsic factor and subsequent pernicious anemia, which would explain her anemia and fatigue. This also leads to achlorhydria (lack of stomach acid), which causes increased gastrin levels as a compensatory mechanism, further suggesting that option C, Parietal cell destruction, is the most likely cause of her symptoms." + }, + { + "input": "Question: An investigator is studying traumatic knee joint injuries in college athletes. Images and tissue samples are collected from 4 athletes with cartilaginous injury and 4 athletes with bone fractures. After 8 weeks, the athletes with bone fractures show almost complete resolution, while the athletes with cartilaginous injuries show only minimal signs of healing. The investigator hypothesizes that this is due to the absence of progenitor cells for matrix regeneration. Transplant of which of the following tissues would most likely promote healing in the group with cartilaginous injury?\nOptions: A: Perichondrium\tB: Proteoglycans\tC: Osteoblasts\tD: Mature chondrocytes\t\nThe right answer is A: Perichondrium.", + "output": "The investigator's hypothesis revolves around the presence of progenitor cells necessary for matrix regeneration in traumatic knee joint injuries. The perichondrium is a layer of dense irregular connective tissue that surrounds cartilage in various locations and contains progenitor cells capable of differentiating into chondrocytes, which are cells responsible for synthesizing new cartilage matrix. In contrast, proteoglycans are simply a component of the matrix itself, not cells that can aid in regeneration. Osteoblasts are involved in bone formation, not cartilage repair. Mature chondrocytes could potentially aid in repair but are less likely than progenitor cells to repopulate a damaged area. Thus, transplanting perichondrium (Option A) would introduce a source of progenitor cells that can regenerate the cartilaginous matrix, thereby promoting healing in the group with cartilaginous injury." + }, + { + "input": "Question: A 47-year-old woman comes to the emergency department because of worsening pain, discoloration, and swelling of her right hand. Two days ago, she was camping outdoors in Texas with her two young children. She was awoken in the middle of the night by “something crawling on my hand.” The following morning, she saw a wound on her hand that was initially painless, but over the next several hours she noticed reddening and blistering. Yesterday, the lesion turned blue with a blackish discoloration at the center. Her temperature is 38°C (100.4°F), pulse is 83/min, and blood pressure is 128/84 mm Hg. Physical examination shows a bluish plaque with a central area of necrosis surrounded by a rim of erythema at the dorsum of the right hand. Which of the following animals is most likely responsible for the patient's symptoms?\nOptions: A: Diamondback rattlesnake\tB: Brown recluse spider\tC: Striped bark scorpion\tD: Deer tick\t\nThe right answer is B: Brown recluse spider.", + "output": "The patient's symptoms are indicative of a brown recluse spider bite, which is found in Texas and known to cause such reactions. Bites from this spider often lead to local tissue necrosis characterized by a blue discoloration with a black center, termed eschar, surrounded by erythema—consistent with her clinical presentation. Systemic symptoms such as fever may also occur. Diamondback rattlesnake bites typically cause prominent swelling, severe pain immediately, and systemic signs of envenomation. Striped bark scorpions can cause pain and paresthesia but rarely cause severe local tissue necrosis. Deer ticks are associated with Lyme disease, not acute necrotic lesions." + }, + { + "input": "Question: A 55-year-old construction worker is struck by a high-voltage power line while on duty. According to bystanders, the cable struck him on his right arm, after which the man slumped to the ground with his arms and legs shaking. In the emergency department, the man is alert but in severe pain localizing bilaterally in the flanks as well as in his right arm where contact with the power line occurred. A 6 cm burn is present on the right arm. His vitals are within normal limits except for mild tachypnea at 21/min. Which of the following diagnostic tests should be conducted first to evaluate this patient?\nOptions: A: FAST ultrasound assessment\tB: Urinalysis\tC: X-ray of the arm and shoulder\tD: EEG\t\nThe right answer is B: Urinalysis.", + "output": "The patient's presentation suggests electrical injury with potential deep tissue damage. The severe flank pain and tachypnea may indicate acute kidney injury (AKI) due to myoglobinuria from muscle breakdown (rhabdomyolysis), which can result from the electric current passing through the body. A urinalysis (option B) should be performed first to assess for myoglobinuria by checking for the presence of blood without red cells on microscopy, which is a sign of myoglobin presence. Early identification of AKI is crucial for initiating appropriate fluid resuscitation and preventing further renal damage. Options A, C, and D are important but secondary to the immediate concern of potential AKI. FAST ultrasound wouldn't show rhabdomyolysis, and an X-ray, while important for assessing skeletal injury, doesn't prioritize the acute potential renal failure. An EEG isn't indicated in the absence of ongoing seizure activity or unexplained altered mental status post-electrocution." + }, + { + "input": "Question: A 71-year-old woman comes to the physician for evaluation of a pigmented lesion on her left foot. She has had decreased appetite and a 6-kg (13-lb) weight loss in the past 2 months. Physical examination shows that on the left medial ankle there is a 2-cm reddish-black nodule with a verrucous surface. A full-thickness excisional biopsy of the lesion shows S100-protein-positive epithelioid cells. A subsequent CT scan of the chest, abdomen, and pelvis shows metastases to the liver; a diagnosis of metastatic melanoma is made. Chemotherapy with aldesleukin is initiated. Which of the following is the mechanism of action for this drug?\nOptions: A: Stimulating release of acute-phase reactants\tB: Stimulating antigen-processing cells\tC: Activating natural killer cells and cytotoxic T cells\tD: Supporting growth of bone marrow stem cells\t\nThe right answer is C: Activating natural killer cells and cytotoxic T cells.", + "output": "Aldesleukin is a form of interleukin-2 (IL-2), a cytokine that plays a critical role in the immune system. Its primary mechanism of action is option C, activating natural killer (NK) cells and cytotoxic T lymphocytes (CTLs). These cells are pivotal in the body's anti-tumor response. NK cells can directly kill tumor cells without prior sensitization, while CTLs recognize and destroy cells presenting specific antigens, such as those found on tumor cells. Stimulating these immune components can lead to the regression of cancerous lesions and is a commonly used strategy in immunotherapy against cancers like melanoma. Other options don't accurately describe the functionality of aldesleukin." + }, + { + "input": "Question: A 24-year-old woman delivers a baby at term. The Apgar counts are 9 and 10 at 1 and 5 minutes, respectively. Thirty minutes after delivery, the mother’s blood pressure drops to 80/60 mm Hg and her pulse is 124/min. On physical examination, her distal extremities are clammy and cold, and the uterus is boggy. Aggressive fluid resuscitation is initiated. After administering 4 units of IV fluids and 4 units of whole blood, her vitals stabilize. The next morning, she says she is unable to lactate when she tries to breastfeed her child. Which of the following is the most likely cause of the failure of lactation in this patient?\nOptions: A: Pituitary tumor\tB: Remains of placenta in uterus\tC: Pituitary infarction\tD: Pituitary hemorrhage\t\nThe right answer is C: Pituitary infarction.", + "output": "The scenario described suggests Sheehan's syndrome, a condition where there's postpartum pituitary infarction due to severe blood loss and hypotensive shock. The patient's acute drop in blood pressure and significant blood loss necessitated the transfusion, which aligns with the consequences of Sheehan's syndrome leading to anterior pituitary failure. A boggy uterus also indicates uterine atony, contributing to the hemorrhage. Pituitary infarction would impair the release of prolactin, which is necessary for lactation, hence failing to lactate is symptomatic of anterior pituitary damage and not due to a tumor, retained placental fragments, or hemorrhage, in this clinical setting. Therefore, the most likely cause of the failure of lactation in this patient is option C, pituitary infarction." + }, + { + "input": "Question: A 64-year-old man with multiple sclerosis comes to the physician because of a 4-month history of urinary incontinence. The urge to urinate occurs suddenly, and he often is unable to make it to the bathroom before leaking urine. He has no incontinence with coughing or laughing. Digital rectal examination shows no abnormalities. Ultrasonography shows a normal post-void residual volume. Which of the following is the most appropriate pharmacotherapy for this patient's incontinence?\nOptions: A: Midodrine\tB: Rivastigmine\tC: Mirtazapine\tD: Tolterodine\t\nThe right answer is D: Tolterodine.", + "output": "The most appropriate pharmacotherapy for this patient's symptoms is tolterodine (D). This patient is experiencing urge incontinence, a common urinary symptom in patients with multiple sclerosis (MS). Urge incontinence is typically caused by detrusor overactivity, which is an involuntary contraction of the bladder muscle. Tolterodine is an antimuscarinic agent which helps to relax the detrusor muscle, thereby reducing the urgency and frequency of urination. Midodrine (A) is used for orthostatic hypotension, rivastigmine (B) for dementia, particularly in Parkinson's or Alzheimer's disease, and mirtazapine (C) is an antidepressant, none of which are appropriate for treating urinary incontinence. The absence of incontinence with coughing or laughing and a normal post-void residual volume further supports the diagnosis of urge incontinence as opposed to stress incontinence or overflow incontinence, respectively." + }, + { + "input": "Question: A 42-year-old woman with a history of depression and headaches presents to the emergency room with severe, pulsating pain around the crown of her head beginning 12 hours ago. She last took ibuprofen 2 hours ago but did not feel any improvement in the pain. She has been nauseous and unable to get out of bed and is currently laying in the dark with the lights off. Her depression has improved, and she has stopped taking her sertraline 2 months ago. Other than ibuprofen as needed, she is not currently taking any additional medications. Which of the following would be the best treatment for her acute symptoms?\nOptions: A: No treatment necessary\tB: 100% oxygen\tC: Propranolol\tD: Sumatriptan\t\nThe right answer is D: Sumatriptan.", + "output": "The best treatment for the patient's acute symptoms is D. Sumatriptan. This presentation is classic for a migraine headache, which includes severe pain, pulsating quality, nausea, and sensitivity to light (photophobia). Sumatriptan is a triptan medication that is often used for the acute treatment of migraine attacks. It works by stimulating 5-HT (serotonin) receptors to cause vasoconstriction of cranial blood vessels, which helps alleviate the headache and associated symptoms. Options A and C are not appropriate for acute treatment; no treatment is not an option for severe pain, and propranolol is a prophylactic, not acute, treatment. B, inhaling 100% oxygen, is used for cluster headaches, not migraines. Given the patient's symptoms and the lack of relief from ibuprofen, Sumatriptan is the advised clinical choice." + }, + { + "input": "Question: A 17-year-old high school student was in shop class when he accidentally sawed off his pointer finger while making a bird house. He fainted when he realized his finger had been cut off. The teacher immediately transported the patient to the emergency department and he arrived within 20 minutes. He has a past medical history of asthma, and his only medication is an albuterol inhaler. The patient's current blood pressure is 122/78 mmHg. Analgesics are administered. The teacher states that he left the amputated finger in the classroom, but that the principal would be transporting it to the hospital. Which of the following is the correct method of transporting the amputated finger?\nOptions: A: Submerge the finger in ice water\tB: Wrap the finger in moist gauze\tC: Wrap finger in moist gauze, put in a plastic bag, and place on ice\tD: Wrap the finger in moist gauze and submerge in ice water\t\nThe right answer is C: Wrap finger in moist gauze, put in a plastic bag, and place on ice.", + "output": "The correct method of transporting an amputated finger is option C: wrap the finger in moist gauze, put it in a plastic bag, and place it on ice. This approach helps preserve the viability of the amputated part. Moist gauze prevents the tissue from drying out, while the plastic bag provides a sterile environment and a barrier against direct contact with ice, which could cause frostbite and further tissue damage. Placing the bag on ice cools the tissue, which reduces metabolism and extends the time window for successful reattachment. Direct submersion in water or direct contact with ice is to be avoided." + }, + { + "input": "Question: A 24-year-old woman comes to the physician for a routine gynecological examination and to refill her birth control pills. Her last pap smear at age 22 showed no intraepithelial lesions nor malignancy of the cervical cells. Menses occur at regular 28-day intervals and last 4 days. Her last menstrual period was 2 weeks ago. She is sexually active with one male partner. Her father died of colon cancer at the age of 75 years and her mother was diagnosed with breast cancer at the age of 40 years. She mentions that she is planning a backpacking trip through Europe in 2 months. Her routine immunizations are all up-to-date. Her temperature is 36.9°C (98.5°F) and her blood pressure is 94/58 mm Hg. Pelvic and speculum examination show no abnormalities. Which of the following is the best recommendation for the patient at this time?\nOptions: A: Neisseria gonorrhoeae and Chlamydia trachomatis screening\tB: Beta-hCG urine test\tC: Repeat Pap smear\tD: No further testing is necessary\t\nThe right answer is A: Neisseria gonorrhoeae and Chlamydia trachomatis screening.", + "output": "The choice of Neisseria gonorrhoeae and Chlamydia trachomatis screening (Option A) is the best recommendation given that the patient is sexually active with a male partner and is heading on a backpacking trip, which may increase the risk of new sexual encounters and thus sexually transmitted infections (STIs). The US Preventive Services Task Force (USPSTF) recommends routine screening for these infections in sexually active women age 24 years and younger and in those who are at increased risk for infection. Although her Pap smear history is unremarkable, a Pap smear at this visit is not indicated as current guidelines recommend screening every 3 years starting at age 21. Options B (pregnancy test) and D (no further testing) are less appropriate given her sexual activity status and potential risk for STIs. Therefore, STI screening is a prudent preventive health measure." + }, + { + "input": "Question: A 27-year-old man presents to the emergency department after a motor vehicle accident. The patient has a past medical history of asthma and cocaine use. His current medications include albuterol and ibuprofen. His temperature is 99.5°F (37.5°C), blood pressure is 70/35 mmHg, pulse is 150/min, respirations are 19/min, and oxygen saturation is 98% on room air. On initial trauma evaluation, his airway is intact and he is breathing spontaneously. The patient's dorsalis pedis and radial pulses are palpable bilaterally. His Glasgow Coma Scale is 9. Head to toe examination reveals instability of the pelvis when downwards force is applied. The patient is started on IV fluids and norepinephrine, and a pelvic binder is placed. The patient's vitals are measured again and his temperature is 99.5°F (37.5°C), blood pressure is 100/55 mmHg, pulse is 150/min, respirations are 17/min, and oxygen saturation is 98% on room air. A blood transfusion is started, and the patient is given hydromorphone. The patient suddenly becomes short of breath with notable bilateral wheezing with poor air movement. His blood pressure is 60/35 mmHg and pulse is 160/min. The patient is treated appropriately, intubated, stabilized, and transferred to the surgical intensive care unit. Which of the following is most likely to be found in this patient's history?\nOptions: A: Hemolytic anemia and ataxia\tB: Gum disease and poor wound healing\tC: Irregularly irregular heart rhythm\tD: Anti-A, B, or O antibodies in the serum\t\nThe right answer is A: Hemolytic anemia and ataxia.", + "output": "The correct answer, Hemolytic anemia and ataxia, is consistent with the patient’s sudden onset of shortness of breath and hypotension following a blood transfusion, suggesting a possible transfusion reaction due to a mismatched blood type. The presence of anti-A, B, or O antibodies in the serum could also explain this reaction, but the patient’s clinical presentation more strongly suggests an acute hemolytic transfusion reaction possibly related to an underlying condition like anemia, rather than immediate antibody-mediated hemolysis (which typically presents differently). Ataxia, while not directly related to the acute scenario, can be part of the history of an individual with underlying hemolytic anemia, and could have been a preexisting condition. The other options (B and C) do not directly account for the sudden hemodynamic changes seen post-transfusion." + }, + { + "input": "Question: An academic obstetrician is conducting a retrospective cohort study that evaluates the risk of placenta accreta at all statewide medical centers. Per chart review he finds that a prior cesarian birth is associated with a statistically significant increased risk of placenta accreta. The relative risk associated with this finding is 1.23. The associated p-value is 0.03. Which of the following statements is the best interpretation of the reported association in the context of the study?\nOptions: A: The 99% confidence interval includes the null hypothesis.\tB: The chance of bias in favor of the alternative hypothesis is 3%.\tC: The p-value represents the likelihood of seeing an increased risk of placenta accreta in women with prior cesarian birth, assuming the alternative hypothesis is true.\tD: The p-value represents the likelihood that the alternative hypothesis is false.\t\nThe right answer is A: The 99% confidence interval includes the null hypothesis..", + "output": "Option A is the best interpretation of the reported association because a p-value of 0.03 indicates that there is a 3% probability of observing at least as extreme a result as the one obtained in this study due to random chance alone, under the assumption that the null hypothesis is true. It does not, however, directly address the confidence interval. But given that conventionally a p-value less than 0.05 is considered statistically significant, it suggests the confidence interval likely does not include the null (no difference or risk of one). Yet, a 99% confidence interval is much stricter and could potentially include the null, indicating that more extreme values are accounted for and there is still a possibility that the null hypothesis isn't rejected at the 99% confidence level. Hence, the best interpretation, given the options, is A, which captures the uncertainty regarding whether the 99% confidence interval includes the null hypothesis." + }, + { + "input": "Question: A 55-year-old man presents with complaints of exertional dyspnea and dry cough. He reports using extra pillows at night to sleep and two nighttime awakenings to catch his breath in the last month. The patient lives in a homeless shelter and has not seen a doctor in the last 20 years. He has been drinking 5-8 drinks of alcohol per day for the last 20 years. Family history is non-significant, and the patient does not take any medications. His temperature is 37.1°C (98.9°F), blood pressure is 135/115 mm Hg, pulse is 85/min, and respiratory rate is 24/min. Physical examination reveals 2+ bilateral pitting edema. Chest auscultation reveals bibasilar crackles. Which of the following is associated with this patient’s condition?\nOptions: A: Decreased glomerular filtration rate (GFR)\tB: Decreased extracellular fluid (ECF) volume\tC: Increased renal blood flow\tD: Increased renal tubular sodium reabsorption\t\nThe right answer is D: Increased renal tubular sodium reabsorption.", + "output": "This patient's symptoms, including exertional dyspnea, dry cough, orthopnea (needing extra pillows to sleep), paroxysmal nocturnal dyspnea (nighttime awakenings to catch breath), as well as physical findings of hypertension, tachypnea, and bilateral pitting edema, suggest congestive heart failure (CHF). The findings of bibasilar crackles on chest auscultation support the diagnosis of CHF leading to pulmonary edema. In CHF, the heart is unable to pump effectively, leading to fluid backup and increased venous pressure. This scenario prompts the kidneys to retain sodium and water in an attempt to increase circulating blood volume and improve cardiac output. Therefore, the increased renal tubular sodium reabsorption (option D) is a compensatory mechanism associated with CHF. Decreased GFR (option A) could also be a result but is not directly associated with the symptoms. Options B and C are unlikely given the signs of fluid overload, not decreased ECF or improved renal blood flow." + }, + { + "input": "Question: A 22-year-old man presents to the medical clinic after getting bit in the hand by a stray dog 2 days ago. The patient has no past medical history and takes no prescription medications. He does not drink alcohol or smoke cigarettes. The patient’s vitals are currently within normal limits. On examination, the physician notices 2 clean puncture wounds with localized cellulitis. Capillary refill is 2 seconds. Sensorimotor evaluation of the hand is without deficits. Which of the following is the most appropriate treatment choice for this patient?\nOptions: A: Amoxicillin\tB: Amoxicillin-clavulanate\tC: Clindamycin\tD: Doxycycline\t\nThe right answer is B: Amoxicillin-clavulanate.", + "output": "The most appropriate treatment choice for this patient is B. Amoxicillin-clavulanate. The choice is made because dog bites can introduce a mixed flora of bacteria into the wound, including aerobic and anaerobic species. Amoxicillin alone may not be sufficient, as it covers mainly gram-positive organisms; however, amoxicillin-clavulanate (co-amoxiclav) provides a broader spectrum of coverage, including enhanced efficacy against the gram-positive, gram-negative, and anaerobic bacteria typically found in bite wounds. This combination reduces the risk of wound infection, covers potential resistant strains of bacteria and addresses the associated cellulitis indicated by localized inflammation. Options A, C, and D are less broad-spectrum and, therefore, less appropriate in this setting." + }, + { + "input": "Question: A 38-year-old woman presents to the office for a routine examination. She has no complaints and offers very little information voluntarily. She answers each question with soft, short sentences and avoids eye contact. She appears timid, anxious and admits that this is one of the very few times she has left the house in the last several years. Medical history is significant for essential hypertension. She takes hydrochlorothiazide and a daily vitamin. She has worked from home as a web graphic designer for 20 years. Questions about her social life reveal that she is very shy with few friends, and she often makes excuses to avoid parties and social gatherings. Despite this, she expresses a strong desire to be in a relationship. Today, her blood pressure is 125/85 mm Hg, heart rate is 95/min, respiratory rate is 18/min, and temperature is 37.0°C (98.6°F). On physical examination, her heart has a regular rhythm and her lungs are clear to auscultation bilaterally. Which of the following is most consistent with her behavior?\nOptions: A: Schizoid personality disorder\tB: Avoidant personality disorder\tC: Antisocial personality disorder\tD: Agoraphobia\t\nThe right answer is B: Avoidant personality disorder.", + "output": "The woman's characteristics of timidity, anxiety, limited social interactions, and the tendency to avoid social situations are indicative of Avoidant Personality Disorder (Option B). Individuals with this disorder are typically hypersensitive to rejection, have feelings of inadequacy, and are socially inhibited, which aligns with her behavior of short, quiet responses, poor eye contact, and her shyness with few friends. Unlike Agoraphobia, where fear is focused on being in places where escape might be difficult, her avoidance seems to stem more from social insecurities, which is why agoraphobia (Option D) is less consistent. Schizoid Personality Disorder (Option A) is characterized by detachment from social relationships and a limited range of emotional expression, but there is no evidence she is detached from social relationships; she expresses a desire for a relationship. Lastly, Antisocial Personality Disorder (Option C) involves a disregard for others' rights and a propensity for manipulation and deceit, which does not apply here. Therefore, her behavior is most consistent with Avoidant Personality Disorder." + }, + { + "input": "Question: A 27-year-old G3P1010 makes an appointment with her gynecologist for evaluation of profuse bleeding, which began two days ago. She is 13 weeks pregnant based on the date of her last menstrual period. She is dizzy and weak, and is unable to move around the house and perform her daily tasks. Initially, she had only light spotting, but later on in the day the bleeding increased. Thus far, she has used six sanitary pads. The blood pressure is 90/60 mm Hg, the temperature is 37.8°C (100°F), the pulse is 125/min, and the respiratory rate is 14/min. A saline infusion is started, and blood and urine specimens are sent for analysis. On pelvic examination, the vagina contains blood and the cervical os is open. She also complains of cervical motion and adnexal tenderness on examination. An ultrasound shows an intrauterine gestational sac low in the uterine cavity. No fetal movement or cardiac activity are observed. She is advised to rest for a couple of hours, after which the scan is repeated. Slight downward migration of the gestational sac is observed with no change in the fetal status. Which of the following is the next step in management?\nOptions: A: Antibiotics\tB: Magnesium sulfate\tC: Methotrexate\tD: Dilation and curettage\t\nThe right answer is D: Dilation and curettage.", + "output": "The patient presents with symptoms and signs suggestive of an inevitable miscarriage at 13 weeks gestation, including profuse bleeding, an open cervical os, and an ultrasound demonstrating a low-lying gestational sac with no fetal movement or cardiac activity. The increasing bleeding, hemodynamic instability indicated by low blood pressure and high pulse rate, and lack of fetal viability necessitate prompt intervention to prevent further blood loss and potential complications.Antibiotics (Option A) are not indicated as there is no current evidence of infection. Magnesium sulfate (Option B) is used for seizure prophylaxis in pre-eclampsia and eclampsia, which is not the issue here. Methotrexate (Option C) is used for ectopic pregnancies and not indicated for an intrauterine pregnancy that is passing.Dilation and curettage (Option D) is the next step in management to evacuate the uterine contents, which will help control the bleeding, prevent infection, and manage the incomplete miscarriage. Therefore, due to the lack of fetal viability, an open cervical os, and the patient's symptomatic presentation with increased bleeding and hemodynamic instability, D&C is the appropriate treatment." + }, + { + "input": "Question: A 61-year-old woman presents to an outpatient clinic with fatigue and mild neck pain for a month. She adds that she is having difficulty swallowing. She denies trouble with her breathing but endorses 5–10 lb of unintentional weight loss over the past few months. On physical exam, the patient’s fingernails appear flat and mild conjunctival pallor is noted. An upper barium esophagram shows an upper esophageal web. A complete blood count reveals:\nHb% 10 gm/dL\nTotal count (WBC) 11,000 /mm3\nDifferential count:\nNeutrophile 70%\nLymphocytes 25%\nMonocytes 5%\nESR 10 mm/hr\nWhat is the most likely diagnosis?\nOptions: A: Plummer-Vinson syndrome\tB: Esophageal squamous cell carcinoma\tC: Zenker’s diverticulum\tD: Achalasia\t\nThe right answer is A: Plummer-Vinson syndrome.", + "output": "The most likely diagnosis is A. Plummer-Vinson syndrome. This condition is characterized by the triad of dysphagia, iron-deficiency anemia, and esophageal webs. The patient's presentation of fatigue and difficulty swallowing, along with the finding of an esophageal web on her barium esophagram, is consistent with this syndrome. Additionally, the conjunctival pallor and low hemoglobin indicate anemia, which is common in Plummer-Vinson syndrome. The mild neck pain can be associated with the difficulty in swallowing due to the esophageal web. Other options are less likely given the clinical presentation and lab findings." + }, + { + "input": "Question: A 64-year-old male presents to his primary care physician with a complaint of bilateral knee pain that has been present for the past several years but has worsened recently. He reports pain with climbing stairs and with extended walks of greater than 100 yards. The pain worsens with activity throughout the day and is alleviated by periods of rest. He states that he has minimal morning stiffness, lasting approximately 5-10 minutes after waking up most days. Physical examination reveals tenderness to palpation of the bony structures on the medial aspect of the bilateral knees as well as crepitus and a decreased range of motion, limited at the extremes of flexion and extension. Both knee joints are cool to touch and exhibit bony enlargement upon palpation of the medial joint line. Which of the following studies would be indicated for further work-up of this patient's presenting condition?\nOptions: A: Complete blood count (CBC)\tB: Erythrocyte sedimentation rate (ESR)\tC: MRI of the knee\tD: No further work-up needed\t\nThe right answer is D: No further work-up needed.", + "output": "The patient's presentation of bilateral knee pain, with an increase in pain during activities like climbing stairs and walking, suggests a gradual, mechanical pattern typical of osteoarthritis (OA). Other symptoms including a brief duration of morning stiffness, tenderness at the medial aspects, crepitus, and bony enlargement also support this diagnosis. The lack of systemic symptoms such as prolonged morning stiffness, fever, or warmth and erythema over joints, which might suggest inflammatory conditions, negates the immediate need for blood tests like CBC or ESR. MRI is not warranted unless there is suspicion of other joint pathology or if the diagnosis is uncertain after radiographs, which is not the case here. OA is a clinical diagnosis, and advanced imaging is not necessary unless the management would change based on these findings. The condition typically requires conservative management initially, not urgent further work-up. Therefore, option D is correct because the information currently presented by the patient aligns with OA, which does not require additional immediate studies." + }, + { + "input": "Question: An investigator is studying cellular regeneration of epithelial cells. She has obtained a tissue sample from a normal thyroid gland for histopathologic examination. It shows follicles lined by a single layer of cube-like cells with large central nuclei. Which of the following parts of the female reproductive tract is also lined by this type of epithelium?\nOptions: A: Ovaries\tB: Vagina\tC: Fallopian tubes\tD: Vulva\t\nThe right answer is A: Ovaries.", + "output": "The correct answer is A, Ovaries. The thyroid gland and the ovaries are similar in that they are both lined with a single layer of cuboidal epithelium. In the thyroid gland, cuboidal cells line the follicles and engage in the production of thyroid hormones. Similarly, in the ovaries, the surface epithelium is composed of a simple cuboidal or low columnar epithelium, which is involved in the encapsulation of the ovary and lacks the direct involvement in gamete transport seen in structures like the fallopian tubes (lined with ciliated columnar epithelium), vital for their function. The vagina, option B, is lined with stratified squamous epithelium, while the vulva, option D, has skin with stratified squamous epithelium as well, making these options incorrect." + }, + { + "input": "Question: A 65-year-old male presents to his primary care physician for stiffness in his arm. He states that he has been having trouble combing his hair and reaching objects that are high on the shelf. The patient has a past medical history of diabetes mellitus type II, obesity, and hypertension. His current medications include metformin, insulin, lisinopril, and hydrochlorothiazide. The patient admits to leading a sedentary life in which he tends to stay home and watch television. He does not engage in any physical or strenuous activity. On physical exam the patient has decreased passive and active range of motion of his shoulder. Strength of the patient's upper extremity is 4/5. Which of the following is the most likely diagnosis?\nOptions: A: Rotator cuff impingement\tB: Adhesive capsulitis\tC: Glenohumeral osteoarthritis\tD: Subacromial bursitis\t\nThe right answer is B: Adhesive capsulitis.", + "output": "Adhesive capsulitis, also known as frozen shoulder, is the most likely diagnosis given the patient's history of prolonged immobility and a sedentary lifestyle, as well as the clinical presentation of decreased passive and active range of motion in the shoulder. This condition is common in diabetes patients, as this patient’s history indicates. Rotator cuff impingement and subacromial bursitis would also present with shoulder pain and decreased range of motion, but these conditions typically involve specific painful arc motions. Glenohumeral osteoarthritis could be a consideration due to the patient's age, but adhesive capsulitis is more consistent with a sudden loss of both active and passive range of motion." + }, + { + "input": "Question: A 52-year-old man presents to the clinic with a headache and dizziness for the past month. The patient reports that about 4 weeks ago he developed a “nagging” headache. He thought he had a cold, so he tried over the counter ibuprofen that provided only mild and temporary relief. Recently, he also started feeling dizzy and nauseous, which only resolves when he steps outside to shovel the snow. The patient’s medical history is significant for hypertension, type II diabetes mellitus, and asthma. His medications include amlodipine, metformin, glimepiride, and a fluticasone/salmeterol inhaler. The patient has been living at his upstate cabin for the past 5 weeks but denies other recent travel. He denies smoking tobacco or illicit drug use. The patient’s temperature is 99°F (37.2°C), blood pressure is 130/78 mmHg, pulse is 72/min, and respirations are 16/min with an oxygen saturation of 98% on room air. The patient is currently asymptomatic while sitting in the office, and physical examination is unremarkable. Labs are drawn, as shown below:\n\nHemoglobin: 20 g/dL\nHematocrit: 60%\nLeukocyte count: 10,050 cells/mm^3 with normal differential\nPlatelet count: 350,000/mm^3\n\nWhich of the following underlies the patient’s most likely diagnosis?\nOptions: A: Medication side effect\tB: Myeloproliferative neoplasm\tC: Neuronal hyperexcitability\tD: Toxic exposure\t\nThe right answer is D: Toxic exposure.", + "output": "The patient's symptoms of headache, dizziness, and nausea which resolve upon leaving an enclosed space, such as his cabin, suggest the possibility of toxic exposure, most likely carbon monoxide (CO) poisoning. This suspicion is supported by the improvements experienced when stepping outside to shovel snow, indicating that symptoms abate when he is exposed to fresh air, and potentially away from a CO source. The elevated hemoglobin and hematocrit could be explained by chronic exposure to decreased oxygen levels, prompting the body to produce more red blood cells in an effort to improve oxygen delivery. Chronic exposure to CO, which has a high affinity for hemoglobin, would reduce the oxygen-carrying capacity of blood, inducing a compensatory erythrocytosis. This scenario is consistent with CO poisoning due to improper heating systems or poor ventilation in the cabin, rather than being a medication side effect, a myeloproliferative neoplasm, or neuronal hyperexcitability, none of which characterize the context of symptomatic relief on exiting the enclosed space." + }, + { + "input": "Question: A 29-year-old woman presents to her primary care doctor with a lesion on her left labia. She first noticed the lesion 3 days ago. The patient describes the lesion as painful and swollen. She denies vaginal discharge. Her past medical history is notable for mild intermittent asthma, gout, and obesity. She uses an albuterol inhaler as needed and takes allopurinol. She has had 5 sexual partners in the past year and uses the pull-out method for contraception. She has a 10-pack-year smoking history and drinks 10-12 alcoholic beverages per week. On exam, she has an ulcerated, tender, and purulent ulcer on the left labia majora. The patient has mild unilateral painful inguinal lymphadenopathy. This patient's condition is most likely caused by which of the following pathogens?\nOptions: A: Klebsiella granulomatis\tB: Herpes simplex virus type 2\tC: Haemophilus ducreyi\tD: Treponema pallidum\t\nThe right answer is C: Haemophilus ducreyi.", + "output": "The patient presents with an ulcerated, tender, and purulent ulcer on the left labia majora, accompanied by mild unilateral painful inguinal lymphadenopathy. These symptoms are highly suggestive of a sexually transmitted infection (STI). Option B, Herpes simplex virus type 2 (HSV-2), typically presents with painful vesicular lesions that can ulcerate, and while this could account for some of the clinical findings, HSV-2 more commonly results in bilateral lymphadenopathy and recurrences, which are not mentioned here.Option C, Haemophilus ducreyi, is the causative agent of chancroid. Chancroid is characterized by painful, purulent genital ulcers and painful regional lymphadenopathy, closely matching the patient's presentation.Option D, Treponema pallidum, causes syphilis, which typically presents with a painless ulcer and non-tender lymphadenopathy, which does not match the presentation.Thus, the most likely pathogen based on her clinical presentation is Haemophilus ducreyi, making option C the correct answer." + }, + { + "input": "Question: A 28-year-old man reports to his physician that he is having second thoughts about his long-term compatibility with his wife ever since they relocated to a new city. He admits that he has noticed himself flirting with some of the female co-workers at his new workplace. When he goes on a dinner date with his wife after work one evening, he accuses her of giving their waiter flirtatious looks and asks her whether she has been cheating on him or trying to spend time with other men. The man’s actions are most consistent with which of the following psychological defense mechanisms?\nOptions: A: Displacement\tB: Dissociation\tC: Projection\tD: Repression\t\nThe right answer is C: Projection.", + "output": "The man's actions align with the defense mechanism known as projection. Projection is a psychological defense mechanism in which an individual attributes their own unacceptable thoughts, feelings, or motives onto another person. In this scenario, the man is experiencing feelings of attraction toward his co-workers; rather than confronting these feelings, he is projecting them onto his wife by accusing her of infidelity. This accusation could serve to externalize his inner conflict and guilt over his flirtatious behavior, shifting the focus onto his wife and away from his own conduct." + }, + { + "input": "Question: A 25-year-old, G2P2L2 woman presents with a complaint of missed menstrual cycle for 4 weeks. Her previous menstrual cycles were regular, occurring every 28–35 days consistently. The patient does not have any other complaints currently. Her past medical history reveals two uncomplicated and normal vaginal deliveries at term. She has a 2-year-old boy and a 6-month-old girl. The patient and her partner use the withdrawal method for contraception. The urine pregnancy test result is positive. The patient returns to the clinic after 1 week expressing her desire to discontinue with the pregnancy and inquires about the possibility of an elective abortion. Which of the following procedures is the most appropriate for elective termination of pregnancy in this patient?\nOptions: A: Dilatation and evacuation (D&E)\tB: Oxytocin-induced labor\tC: Manual uterine aspiration\tD: Oral mifepristone + misoprostol\t\nThe right answer is D: Oral mifepristone + misoprostol.", + "output": "The most appropriate option for elective termination of pregnancy in the mentioned case is D, Oral mifepristone + misoprostol. This pharmaceutical method is typically suitable for early pregnancies, which the patient is likely experiencing at 4 weeks gestation. Mifepristone is an antiprogestin that thins the uterine lining and dilates the cervix, while misoprostol induces uterine contractions to expel the pregnancy tissue. This combination is generally effective and carries fewer risks than surgical methods for early termination. Moreover, it can be administered in an outpatient setting, providing the patient privacy and convenience. The other options, such as D&E and oxytocin-induced labor, are more invasive, involve more risk, and are typically reserved for later gestational ages or when medical management is contraindicated or has failed. Manual uterine aspiration could be an alternative but is less common than medical management in early pregnancy." + }, + { + "input": "Question: A 72-year-old man presents to his primary care physician with a 6-month history of shortness of breath. He says that he used to enjoy playing golf with his friends but now he cannot walk for long enough to play. The shortness of breath is now starting to complicate his ability to get around in daily life. His past medical history is significant for diabetes, hypertension, and early stage prostate cancer that was removed 10 years ago without recurrence. He has smoked 1 pack per day for the past 55 years and drinks about 6 drinks per week. On presentation he is found to be breathing with pursed lips. Physical exam reveals decreased breath sounds on auscultation, increased chest diameter, and hyperresonance to percussion. Which of the following findings would most likely be seen in this patient?\nOptions: A: Decreased residual volume and decreased 1 second forced expiratory volume\tB: Decreased residual volume and increased 1 second forced expiratory volume\tC: Increased residual volume and decreased 1 second forced expiratory volume\tD: Normal residual volume and decreased 1 second forced expiratory volume\t\nThe right answer is C: Increased residual volume and decreased 1 second forced expiratory volume.", + "output": "The patient's long history of smoking, presentation with pursed-lip breathing, physical exam findings of decreased breath sounds, increased chest diameter, and hyperresonance are all indicative of chronic obstructive pulmonary disease (COPD). In COPD, airflow is obstructed, predominantly due to emphysema and/or chronic bronchitis. A characteristic finding in emphysema is hyperinflation of the lungs, causing an increased residual volume (the amount of air remaining in the lungs after a maximal exhalation). The obstruction of the airways also leads to impaired expiration, reflected by a decreased 1 second forced expiratory volume (FEV1), which measures the amount of air a person can forcefully exhale in one second. Hence, the most likely finding in this patient would be option C: increased residual volume and decreased 1 second forced expiratory volume. This aligns with the typical spirometry findings in a patient with COPD." + }, + { + "input": "Question: A researcher hypothesizes that low birth weight is related to obesity later in life. He conducts a study with a 95% confidence interval with a p-value of 0.049 to disprove his null hypothesis. He rejects his null hypothesis and concludes that low birth weight is associated with obesity. Which of the following statements best associates with his study?\nOptions: A: Confidence interval should have been 90%.\tB: A type 2 error is not possible in this case.\tC: A type 2 error has been made.\tD: A type 1 error has been made.\t\nThe right answer is B: A type 2 error is not possible in this case..", + "output": "A Type 2 error, also known as a false negative, occurs when a researcher fails to reject a null hypothesis that is actually false. However, in this scenario, the researcher has rejected the null hypothesis based on a p-value of 0.049, which is just below the conventional threshold of 0.05 for statistical significance. Therefore, it suggests the researcher found enough evidence to conclude that there is a statistically significant association between low birth weight and obesity later in life, within a 95% confidence interval. As the null hypothesis is rejected, the possibility of a Type 2 error (failing to detect an effect that is there) is eliminated in this context. Hence, option B is correct; a Type 2 error is not possible in this case since the null hypothesis has been rejected. The confidence level chosen and the fact that the null hypothesis was rejected precludes the possibility of a Type 2 error in this study." + }, + { + "input": "Question: A 3-week-old newborn is brought to the physician by his parents because of poor feeding, irritability, and frequent vomiting over the past week. The vomitus is greenish in color and smells strange. His parents have tried to feed him every 4 hours, but the patient often spits up or refuses to eat. The patient was born at term and had his first bowel movement at 50 hours of life. He has since had one bowel movement daily. He is at the 50th percentile for length, 10th percentile for weight, and 40th percentile for head circumference. He does not appear to be in acute distress. His temperature is 36.9°C (98.4°F), pulse is 140/min, respirations are 40/min, and blood pressure is 90/60 mm Hg. Physical examination shows that the patient has small, low-set ears, a broad and flat nasal bridge, and a large space between the first and second toes bilaterally. The abdomen is distended. When the finger is removed following a rectal exam, there is an explosive release of stool from the patient's rectum. An x-ray of the abdomen shows a section of dilated colon followed by a segment of colon without stool or air. Which of the following is most likely to confirm the diagnosis?\nOptions: A: CT scan of the abdomen\tB: Transabdominal ultrasonography\tC: Anorectal manometry\tD: Rectal suction biopsy\t\nThe right answer is D: Rectal suction biopsy.", + "output": "The clinical presentation of poor feeding, irritability, vomiting, delayed first bowel movement, and a notably distended abdomen with an explosive release of stool upon rectal exam are highly suggestive of Hirschsprung's disease. The description of the x-ray showing a dilated segment of the colon preceding an aganglionic segment further supports this suspicion. Hirschsprung's disease is caused by the absence of ganglion cells in the distal bowel, causing functional obstruction. The definitive diagnosis is established through a rectal suction biopsy (Option D), which would demonstrate the absence of ganglion cells in the submucosal and myenteric plexuses. A CT scan (Option A) and ultrasonography (Option B) may show features of bowel obstruction but are not definitive for diagnosing Hirschsprung's disease, whereas anorectal manometry (Option C) can be suggestive but is not confirmatory, especially in neonates where it can be technically difficult to perform and interpret." + }, + { + "input": "Question: A 47-year-old woman is brought to the emergency department by paramedics. She was found unconscious on a park bench by a bystander. Her history is unobtainable. Vitals include a pulse of 64/min, a respiratory rate of 7/min, and a blood pressure of 110/70 mm Hg. On examination, the patient is unresponsive to voice and touch but shows a flexor response to pain. Her breathing is shallow. The pupils are constricted, and a response to light cannot be determined. An arterial blood gas sample reveals:\npH 7.26\nPco2 70 mm Hg\nHCO3- 26 mEq/L\nWhich of the following could explain this patient's presentation?\nOptions: A: Diabetic ketoacidosis\tB: Diuretic overdose\tC: Hyperventilation\tD: Opioid overdose\t\nThe right answer is D: Opioid overdose.", + "output": "The patient's symptoms include unconsciousness, shallow breathing, constricted pupils, and a flexor response to pain—all of which are consistent with opioid overdose. The respiratory rate is low, indicating respiratory depression, a common effect of opioids. The ABG results support this: her pH is low due to respiratory acidosis (indicated by the high Pco2), while the HCO3- is in the normal range, reflecting a primary respiratory issue rather than a metabolic one. Diabetic ketoacidosis typically presents with high glucose levels and metabolic acidosis (low pH and HCO3-). Diuretic overdose might lead to electrolyte imbalances but would be unlikely to cause such severe respiratory depression or constricted pupils. Hyperventilation would likely cause a low Pco2 and an elevated pH due to respiratory alkalosis, the exact opposite of what's observed. Therefore, D. Opioid overdose is the most likely explanation for this presentation." + }, + { + "input": "Question: A 25-year-old Hispanic male presents with heat intolerance and recent weight loss. Serum analysis shows increased levels of T4 and T3, as well as the presence of thyroglobulin-stimulating immunoglobulins. The patient is found to be tachycardic and has marked edema and waxy discoloration in his legs. Which of the following would be consistent with this patient's disease?\nOptions: A: Sympathetic underactivity\tB: Anti-thyroglobin antibodies\tC: Exophthalmos\tD: Increased TSH release\t\nThe right answer is C: Exophthalmos.", + "output": "The patient's symptoms and serum analysis results suggest hyperthyroidism, likely due to Graves' disease. The increased levels of T4 and T3, along with thyroglobulin-stimulating immunoglobulins, indicative of thyroid-stimulating immunoglobulins (TSI), confirm this diagnosis. Graves' disease is an autoimmune disorder that often causes hyperthyroidism by stimulating the thyroid gland to overproduce thyroid hormones. Exophthalmos, or protrusion of the eyeballs, is a classic sign of Graves' disease, which is consistent with the patient's disease profile. The other options are inconsistent: sympathetic underactivity and increased TSH release oppose the expected findings in hyperthyroidism, while anti-thyroglobulin antibodies are not typically the driving factor in Graves' disease." + }, + { + "input": "Question: A 70-year-old man presents with cough and progressively worsening shortness of breath for the last 6 months. He feels short of breath even without exertion and states he ‘got winded’ while walking to his bedroom last night. He describes his cough as non-productive and says he can identify no recognizable triggers. No significant past medical history. The patient is a retired welder and has been living with his son since his wife passed away 5 years ago. He denies any smoking history or current alcohol or drug use. His vital signs include: pulse rate 72/min, respiratory rate 15/min, blood pressure 134/80 mm Hg, and temperature 36.8°C (98.0°F). On physical examination, digital clubbing is noted. Bilateral basilar fine crackles on noted on pulmonary auscultation. Expiratory flow rates are measured and found to be high when corrected for lung volume. A chest X-ray is performed and shown in the image below. Which of the following most likely accounts for the increased expiratory flow rate in this patient?\nOptions: A: Anatomical dead space\tB: Lung compliance\tC: Radial traction on airway wall\tD: Small airway wall thickness\t\nThe right answer is C: Radial traction on airway wall.", + "output": "The increased expiratory flow rate in this patient is likely due to radial traction on the airway walls, represented by option C. In patients with interstitial lung diseases, such as idiopathic pulmonary fibrosis which could be suggested by the patient's symptoms and history (prolonged exposure to metal fumes as a welder, clubbing, fine crackles, and the chronic nature of his respiratory issues), the fibrotic process increases the stiffness of the lung parenchyma. This fibrosis enhances the pull or traction on the bronchioles during expiration, keeping them open and preventing premature collapse, which accounts for the high expiratory flow rates in relation to lung volume. The other options (A, B, and D) would typically lead to decrements in flow rates or would be irrelevant in the context of high expiratory flow rates." + }, + { + "input": "Question: A 45-year-old man presents to his primary care provider after feeling sad and depressed for the past several months. His wife divorced him about 6 months ago and took the kids and moved out of state. Since then he has had difficulty waking up in the morning and getting to work on time. He takes some comfort in food and has gained about 9.1 kg (20 lb). He eats out several times a week and sometimes consumes 2 whole meals in one sitting. Other than this new obsession with eating large quantities of food and excess sleeping he no longer enjoys doing things he liked doing in the past. Besides eating and sleeping he is always excited to see his children and looks forward to their visits. He says it's the one thing keeping him going. Additionally, he does not feel like he has the same drive or focuses at work. He reports that he has no interest in hurting himself or others. His blood pressure is 119/81 mm Hg, pulse rate is 85/min, respiratory rate is 12/min, and the temperature is 36.8°C (98.2°F). His physical exam is normal. What other symptom is most likely present in this patient?\nOptions: A: Leaden paralysis\tB: Depressive symptoms that are worse in the morning\tC: Stereotypies\tD: Mania\t\nThe right answer is A: Leaden paralysis.", + "output": "The patient's presentation of significant weight gain, hypersomnia (excess sleeping), and anhedonia (loss of interest in previously enjoyed activities) suggests a major depressive disorder, possibly with atypical features. Atypical depression can include mood reactivity – where the person can feel improved mood in response to positive events (as seen with the patient's excitement about seeing his children) – and leaden paralysis, which is a heavy, leaden feeling in the arms or legs that is unique to atypical depression. Given the patient's symptoms of increased appetite, weight gain, and oversleeping, leaden paralysis (A) is another likely symptom that fits the profile of atypical depression. The other options list symptoms that are not as consistent with the patient's overall presentation: Depressive symptoms tend to be worse in the morning for typical, not atypical, depressive episodes (B); stereotypies (C) are repetitive, purposeless movements associated with disorders such as autism and are not indicated here; and mania (D) is characterized by elevated mood and energy, which the patient does not exhibit." + }, + { + "input": "Question: A 35-year-old man comes to the emergency department with acute shortness of breath that developed after a 10-hour international flight. His pulse is 124/min and pulse oximetry on room air shows an oxygen saturation of 90%. He weighs 50-kg (110-lb). A diagnosis of pulmonary embolism is suspected and intravenous heparin is initiated. If the volume of distribution of heparin is equivalent to 60 mL/kg and the target peak plasma concentration is 0.5 units/mL, which of the following is the most appropriate loading dose for this patient?\nOptions: A: 3,000 units\tB: 1,500 units\tC: 6,000 units\tD: Cannot be calculated, as bioavailability is not known\t\nThe right answer is B: 1,500 units.", + "output": "Given the volume of distribution (Vd) for heparin is 60 mL/kg, and the target peak plasma concentration is 0.5 units/mL, we calculate the loading dose (LD) using the formula: LD = Vd * target plasma concentration * body weight. The patient weighs 50 kg, giving us the following calculation: LD = 60 mL/kg * 0.5 units/mL * 50 kg. This equates to 1,500 units (60*0.5*50 = 1,500). Therefore, option B (1,500 units) is the most appropriate loading dose. Bioavailability does not affect the calculation since heparin is given intravenously and hence has 100% bioavailability." + }, + { + "input": "Question: A 3466-g (7-lb, 10-oz) female newborn is delivered at 38 weeks' gestation to a 32-year-old woman, gravida 2, para 1. Apgar scores are 7 and 8, at 1 and 5 minutes, respectively. The mother has a history of schizophrenia and was treated with lithium until 5 weeks' gestation, when she found out that she was pregnant. The mother was treated for alcohol abuse disorder 2 years ago. She has been sober for 14 months now. Examination shows that the child has micrognathia, a broad nasal bridge, a short philtrum, and small, low-set ears. Examination of the mouth shows a cleft palate. A grade 3/6 systolic ejection murmur can be heard over the left sternal border. Echocardiography shows a single overriding great vessel arising from the heart. Further evaluation is most likely to show which of the following findings?\nOptions: A: Low levels of parathyroid hormone in serum\tB: Double bubble sign on abdominal x-ray\tC: Chromosome 5p deletion on FISH studies\tD: Bilateral cataracts on ocular examination\t\nThe right answer is A: Low levels of parathyroid hormone in serum.", + "output": "The newborn presents with physical features such as micrognathia, broad nasal bridge, short philtrum, small low-set ears, cleft palate, and a conotruncal cardiac defect (single overriding great vessel), which are indicative of DiGeorge syndrome (22q11.2 deletion syndrome). DiGeorge syndrome can present with conotruncal cardiac anomalies, abnormal facial features, and thymic hypoplasia. A common endocrine abnormality in this condition is hypoparathyroidism, which leads to hypocalcemia due to low levels of parathyroid hormone (PTH) in serum. Hence, Option A describing low levels of parathyroid hormone in serum is the most likely additional finding in this patient's evaluation, fitting the clinical picture of DiGeorge syndrome. The other options are less consistent with the presented features and the suspected diagnosis." + }, + { + "input": "Question: A 10-year-old boy is brought to his pediatrician after discovering a painless mass in the left testicle. Results from tumor markers and a biopsy are as follows:\nAFP 350 ng/mL (normal value < 10 ng/mL)\nhCG 0.4 IU/L (normal value < 0.5 IU/L)\nBiopsy: Presence of glomeruli-like structures with a central capillary within a mesodermal core, lined by flattened layers of germ cells.\nWhat is the most likely diagnosis in this patient?\nOptions: A: Classic seminoma\tB: Choriocarcinoma\tC: Teratoma\tD: Yolk sac tumor\t\nThe right answer is D: Yolk sac tumor.", + "output": "The most likely diagnosis for the patient is D. Yolk sac tumor, also known as endodermal sinus tumor. This diagnosis is supported by the elevated alpha-fetoprotein (AFP) level of 350 ng/mL, which is significantly higher than the normal value. Yolk sac tumors are known to secrete AFP, and such a high level is commonly associated with these tumors in children. The human chorionic gonadotropin (hCG) level is within the normal range, which is not typically elevated in yolk sac tumors and helps rule out choriocarcinoma, as hCG would be expected to be high in this type of tumor. Additionally, the biopsy findings describe structures that resemble glomeruli with a central capillary, which is characteristic of Schiller-Duval bodies unique to yolk sac tumors. Classic seminoma and teratoma do not commonly present with elevated AFP or these histological features in children." + }, + { + "input": "Question: An 80-year-old man presents with severe pain of the right lower extremity. He says that symptoms had an acute onset while he was in the ICU for a COPD exacerbation due to a recent upper respiratory infection, which was treated with nebulized albuterol and ipratropium bromide, continuous oxygen, IV methylprednisolone, and levofloxacin. He says he has been hospitalized 3 other times this year for COPD exacerbations. The medical history is significant for a left main coronary artery myocardial infarction after a coronary artery bypass graft (CABG) 2 months ago, mild cognitive impairment, and type 2 diabetes mellitus. Current medications are metformin, aspirin, clopidogrel, metoprolol, atorvastatin, galantamine, and enalapril. He reports a 40-pack-year smoking history but quit 10 years ago. His temperature is 39.0°C (102.5°F); blood pressure is 100/75 mm Hg; pulse is 122/min; respiratory rate is 20/min, and oxygen saturation is 88% on room air. The physical examination is significant for the cutaneous finding shown in the picture. The involved area is severely tender to palpation and there are warmth and crepitus. A foul-smelling discharge is also noted. Pain seems out of proportion to the exam findings. Which of the following is the next best step in management?\nOptions: A: Immediate debridement\tB: Immediate debridement and empiric IV antibiotics\tC: Biopsy of wound\tD: Contrast CT of the chest and abdomen\t\nThe right answer is B: Immediate debridement and empiric IV antibiotics.", + "output": "The presentation suggests a necrotizing soft tissue infection (NSTI), a severe, life-threatening condition requiring prompt treatment to reduce morbidity and mortality. The acute onset of severe pain, systemic symptoms of fever, tachycardia, hypotension, and the local signs of tenderness, warmth, crepitus along with foul-smelling discharge are highly indicative. Immediate debridement is crucial to remove necrotic tissue and decrease bacterial load. Empiric IV antibiotics are also essential to address the infectious component aggressively, as NSTIs are typically polymicrobial. Early antibiotic therapy, together with surgical intervention, is the gold standard for treating NSTIs, as delays can significantly impact the outcome. Hence, option B is the next best step." + }, + { + "input": "Question: A 3-year-old boy is brought to his pediatrician for evaluation because his parents have been concerned by a number of troubling signs. Specifically, they have noticed that he has been exhibiting aggressive behavior as well as hyperactivity. In addition, he has had intellectual disability and is slow to hit developmental milestones. Physical exam reveals coarse facies, short stature, and joint stiffness. Eye exam reveals no abnormalities of the cornea or lens. Based on these findings, the patient is referred to a geneticist for further evaluation. Which of the following substances would most likely accumulate in the cells of this patient?\nOptions: A: Cerebroside sulfate\tB: Galactocerebroside\tC: Heparan sulfate\tD: Sphingomyelin\t\nThe right answer is C: Heparan sulfate.", + "output": "The clinical presentation suggests a lysosomal storage disease, given the combination of developmental delay, coarse features, short stature, joint stiffness, and aggressive behavior. The absence of eye abnormalities, such as corneal or lenticular opacities, helps to narrow down the options. The accumulation of heparan sulfate is characteristic of mucopolysaccharidosis (MPS), specifically MPS III, or Sanfilippo syndrome. Patients with this condition often show the symptoms described without the eye issues that occur in other types of MPS, thus making option C, heparan sulfate, the most likely substance to accumulate in this patient's cells." + }, + { + "input": "Question: A 4-year-old boy presents to the pediatrician’s office. His mother complains that he has been sleeping poorly, and she has noticed that he often scratches his perianal area. A scotch tape test is found to be positive and mebendazole is prescribed. Into which of the following subsets have the naïve T cells (TH0) developed due to the infection in this patient?\nOptions: A: TH1 cells\tB: TH2 cells\tC: TH17\tD: T0 cells\t\nThe right answer is B: TH2 cells.", + "output": "The patient's presentation is indicative of a parasitic infection, likely enterobiasis (pinworm infection), given the positive scotch tape test and perianal itching. The TH2 subset of helper T cells is involved in the immune response to extracellular parasites and is responsible for orchestrating the production of IgE and the activation of eosinophils, both important in combating parasitic infections. Mebendazole is an antihelminthic medication, which further supports the presence of a helminthic infection. Consequently, the naïve T cells (TH0) have likely differentiated into TH2 cells in response to this type of infection, coordinating an appropriate immune response to the pathogen." + }, + { + "input": "Question: A 40-year-old female with a past medical history of high cholesterol, high blood pressure, hyperthyroidism, and asthma presents to the primary care clinic today. She has tried several different statins, all of which have resulted in bothersome side effects. Her current medications include hydrochlorothiazide, levothyroxine, albuterol, oral contraceptives, and a multivitamin. Her physical examination is unremarkable. Her blood pressure is 116/82 mm Hg and her heart rate is 82/min. You decide to initiate colesevelam (Welchol). Of the following, which is a concern with the initiation of this medication?\nOptions: A: Colesevelam can cause cognitive impairment.\tB: Colesevelam can increase the risk of cholelithiasis.\tC: Timing of the dosing of colesevelam should be separated from this patient’s other medications.\tD: Colesevelam can significantly decrease your HDL.\t\nThe right answer is C: Timing of the dosing of colesevelam should be separated from this patient’s other medications..", + "output": "The correct answer is C, which pertains to the timing of the dosing of colesevelam being separated from the patient's other medications. The rationale behind this choice is that colesevelam is known to bind to bile acids as well as other drugs, potentially decreasing their absorption and effectiveness. Given that this patient is on multiple medications, including levothyroxine and oral contraceptives, it is crucial to ensure these drugs are taken at different times to avoid interactions. Levothyroxine, in particular, is known to bind to colesevelam, which can reduce thyroid hormone levels if not timed correctly, potentially leading to hypothyroidism or suboptimal control of the patient's existing hyperthyroidism." + }, + { + "input": "Question: A 63-year-old man presents to his primary care physician for follow-up. He reports a slow and steady weight gain of 6 pounds over the past 6 months, despite attempts to control his diet and increase his level of exercise. His medications include pravastatin, lisinopril, and hydrochlorothiazide. On exam, his vital signs are stable. He is obese (BMI 32), and his waist circumference is 43 inches. His physician is concerned about an abnormal fasting blood glucose and dyslipidemia. On further work-up with oral glucose tolerance test, the patient is diagnosed with diabetes. Which of the following associations is consistent with this patient’s most likely form of diabetes?\nOptions: A: Strong HLA class II gene makeup\tB: Pancreatic islet cell amyloid deposition\tC: Pancreatic islet cell leukocyte infiltration\tD: Auto-antibodies against pancreatic islet cell antigens\t\nThe right answer is B: Pancreatic islet cell amyloid deposition.", + "output": "The patient's age (63 years old), steady weight gain, obesity (BMI 32), and metabolic issues like dyslipidemia and now diabetes align with a diagnosis of type 2 diabetes mellitus (T2DM). T2DM is characterized by insulin resistance and is typically not immune-mediated, distinguishing it from type 1 diabetes mellitus which often features auto-antibodies and leukocyte infiltration (options C and D). The strong HLA class II gene makeup (option A) is more commonly associated with type 1 diabetes. Option B, pancreatic islet cell amyloid deposition, is a hallmark of T2DM, where amyloid deposits within the islets contribute to β-cell dysfunction. Therefore, option B is the most likely associated with the patient's form of diabetes." + }, + { + "input": "Question: A 10-year-old boy is brought to the emergency department because he has not been able to walk since waking up that morning. His mother said that when he tried to get out of bed that he was unable to stand without support. He also complained of prickling in his hands and feet. Three weeks ago, he had a fever, dry cough, and a sore throat. The fever and sore throat subsided a week after they began, but the cough is persisting. He has no history of serious illness and takes no medication. His vital signs are within normal limits. The lungs are clear to auscultation. There is severe weakness in both lower extremities and mild sensory loss of the hands and feet. Knee and ankle jerk reflexes are absent. The remainder of the examination shows no abnormalities. Laboratory studies show:\nHemoglobin 12.0 g/dL\nLeukocyte count 6000/mm3\nSegmented neutrophils 64%\nEosinophils 2%\nLymphocytes 26%\nMonocytes 7%\nPlatelet count 160,000/mm3\nErythrocyte sedimentation rate 27 mm/h\nSerum\nNa+ 138 mEq/L\nCl- 101 mEq/L\nK+ 4.9 mEq/L\nHCO3- 26 mEq/L\nCa2+ 9.7 mg/dL\nCreatine kinase 93 U/L\nA lumbar puncture is performed; cerebrospinal fluid analysis shows a leukocyte count of 2/mm3, a glucose concentration of 60 mg/dL, and a protein concentration of 91 mg/dL. Which of the following is the most appropriate next step in treatment?\"\nOptions: A: Intravenous immunoglobulin (IVIG)\tB: Prednisone\tC: Botulism antitoxin\tD: Gabapentin\t\nThe right answer is A: Intravenous immunoglobulin (IVIG).", + "output": "The patient's symptoms and medical history are suggestive of Guillain-Barré syndrome (GBS), a post-infectious autoimmune disorder where the body's immune response to an infection mistakenly targets peripheral nerves, leading to muscle weakness and sensory symptoms. Common triggers include respiratory or gastrointestinal infections, such as those the boy experienced. The absence of reflexes (areflexia), severe weakness in the lower extremities, and cerebrospinal fluid with elevated protein but normal white cell count fit the classic GBS profile.The most appropriate next step in treatment for GBS is Intravenous immunoglobulin (IVIG) (Option A). IVIG helps by reducing the autoimmune response, thus slowing or halting nerve damage. Corticosteroids like Prednisone (Option B) are not effective for GBS. Botulism antitoxin (Option C) is irrelevant as botulism presents differently, and Gabapentin (Option D) is primarily used for neuropathic pain rather than as a treatment for the cause of GBS." + }, + { + "input": "Question: A 2-day-old infant male is brought to the emergency department with a 5-hour history of vomiting and irritability. He was born at home and had unreliable prenatal care because his parents have been experiencing housing instability. About a day after birth, he started to become lethargic and was feeding poorly. His parents thought that he might just be tired so they put him to bed; however, by the next morning he started to vomit and act extremely irritable. On presentation, he is found to have nuchal rigidity and bulging fontanelles. Based on this presentation, appropriate cultures are obtained and he is placed on empiric antibiotics. After all cultures grow out the same gram-positive bacteria, the patient is narrowed to penicillin G. Which of the following is a characteristic of the most likely organism that is causing this patient's symptoms?\nOptions: A: Enlarges hemolytic area of Staphylococcus aureus\tB: Produces lipooligosaccharide rather than lipopolysaccharide\tC: Produces toxin that increases cAMP levels\tD: Tumbling motility in broth at 22 degrees\t\nThe right answer is A: Enlarges hemolytic area of Staphylococcus aureus.", + "output": "The most likely organism causing this infant's symptoms is Group B Streptococcus (GBS), also known as Streptococcus agalactiae. GBS is a common cause of meningitis in neonates and typically presents with symptoms such as irritability, vomiting, lethargy, and signs of meningeal irritation including nuchal rigidity and bulging fontanelles, as seen in this patient. The characteristic of GBS that is relevant here is its ability to enlarge the hemolytic area of Staphylococcus aureus on blood agar plates through the production of the CAMP factor. This CAMP factor is a diffusible extracellular protein that interacts synergistically with the beta-hemolysin produced by Staphylococcus aureus, enhancing the area of hemolysis. This is known as the CAMP test and is used for the identification of Group B Streptococcus. Thus, the correct answer is option A: Enlarges hemolytic area of Staphylococcus aureus. Options B, C, and D describe characteristics of other bacteria and are not related to GBS." + }, + { + "input": "Question: A 76-year-old woman comes to the physician for a routine examination. She has a history of hypertension well controlled with diet and exercise. She has no history of smoking or illicit drug use. She walks one mile daily. Her pulse is 68/min, respirations are 16/min, and blood pressure is 119/76 mm Hg. Physical examination shows no abnormalities. When compared to a healthy 20-year-old woman, which of the following changes in respiratory function is most likely present in this patient?\nOptions: A: Increased lung compliance\tB: Decreased residual volume\tC: Increased chest wall compliance\tD: Decreased A-a gradient\t\nThe right answer is A: Increased lung compliance.", + "output": "As individuals age, structural changes in the lungs and chest wall occur. In an elderly patient, such as the 76-year-old woman in the scenario, respiratory function changes include decreased elasticity of the lung tissues and chest wall stiffness. This leads to decreased chest wall compliance, not increased, ruling out option C. Smoking, which she does not have a history of, could affect the A-a gradient, but this is not the primary concern here. Instead, age-related changes typically lead to an increased residual volume (due to loss of elastic recoil and air trapping), making option B incorrect. Therefore, the most likely change is increased lung compliance (option A) due to decreased elastic recoil of the lung tissues with aging." + }, + { + "input": "Question: A 32-year-old man jumps off a 3 story building during a suicide attempt. Paramedics arrive quickly, and he is taken to the hospital where several stable lumbar fractures and a shattered right calcaneus are found. He undergoes surgery and his calcaneus is repaired with 11 screws. Which of the following statements best describes the long term post-surgery outcome in this patient?\nOptions: A: He will not be able to plantar flex his foot.\tB: He will not be able to point his toe.\tC: He will have pain with inversion and eversion of his foot.\tD: He will have pain while dorsiflexing his foot.\t\nThe right answer is C: He will have pain with inversion and eversion of his foot..", + "output": "The most likely long-term outcome following a shattered calcaneus repair with screws is pain with inversion and eversion of the foot, which corresponds to option C. The calcaneus, or heel bone, is integral to the subtalar joint which allows for inversion and eversion movements. Post-surgical pain in these movements can be attributed to several factors such as the development of post-traumatic arthritis, changes in the biomechanics of the foot, or possible hardware irritation. Options A and B are incorrect as the muscles responsible for plantar flexion and toe pointing (gastrocnemius, soleus, and flexor hallucis longus) are not primarily affected by calcaneal repair. Option D is less likely than C because dorsiflexion does not involve the calcaneus as directly as inversion and eversion movements do." + }, + { + "input": "Question: A 25-year-old woman comes to the physician because of periodic painful double vision for 3 days. Her symptoms occur only when looking sideways. She has myopia and has been wearing corrective lenses for 10 years. Ten days ago, she lost her balance and fell off her bike, for which she went to a hospital. A CT scan of the head at that time showed no abnormalities and she was released without further treatment. Her only medication is an oral contraceptive. Her vital signs are within normal limits. The pupils are equal and reactive to light. Her best corrected visual acuity is 20/40 in each eye. She has an adduction deficit in the right eye and nystagmus in the left eye when looking left; she has an adduction deficit in the left eye and nystagmus in the right eye when looking right. Fundoscopy reveals bilateral disc hyperemia. Which of the following is the most likely cause of this patient's findings?\nOptions: A: Demyelination of the medial longitudinal fasciculus\tB: Compressive tumor of the pons\tC: Oxidative damage due to thiamine deficiency\tD: Infarction of the brainstem\t\nThe right answer is A: Demyelination of the medial longitudinal fasciculus.", + "output": "The presenting symptoms are indicative of internuclear ophthalmoplegia (INO), a disorder of conjugate lateral gaze. INO is characterized by impaired adduction on the side of the lesion (medial longitudinal fasciculus) when attempting to look laterally and nystagmus of the abducting eye. The bilateral presentation, also known as bilateral INO, and the recent history of trauma are not typical for a compressive tumor or oxidative damage due to thiamine deficiency. Brainstem infarction would likely present with additional brainstem findings and acute onset. Hence, demyelination, commonly seen in conditions like multiple sclerosis, is the most plausible cause given the patient's age and symptomatology." + }, + { + "input": "Question: A 68-year-old woman presents with an intermittent throbbing headache with an onset approximately 3 weeks ago. The pain is bilateral, “band-like,” and extends to the neck. The patient rates the 4 points out of 10 on a visual scale. The headache is not accompanied by nausea, vomiting, photophobia, or phonophobia, and is not aggravated by movements, but gets worse when the patient coughs or laughs. The headache has persisted throughout the day and is usually worse in the morning an hour after she has breakfast and taken her medications. The patient was diagnosed with stage 1 arterial hypertension 2 years ago and stable angina pectoris 3 weeks ago. Currently, she takes aspirin (81 mg once daily), rosuvastatin (10 mg once daily), lisinopril (20 mg once daily), and isosorbide mononitrate (60 mg once daily). She has been menopausal for the last 15 years. In addition to her chief complaint, she also notes that she had a runny nose with a temperature elevation approximately 3 weeks ago, and her headache was linked to that acute illness. The vital signs are as follows: blood pressure 135/80 mm Hg, heart rate 76/min, respiratory rate 14/min, and temperature 36.5℃ (97.7℉). The physical examination reveals S2 accentuation that is best heard in the second intercostal space at the right sternal border. Palpation of the paranasal sinuses is painless. The neurologic exam shows no focal symptoms, no pain on palpation of the trigeminal points, and no meningeal symptoms. What is the most likely mechanism underlying the patient’s headaches?\nOptions: A: Meningeal irritation by blood products originating from a chronic subdural hematoma\tB: Dissection of an intracerebral vessel\tC: Meningeal irritation by a growing intracranial neoplasm\tD: Dilation of the intracerebral arterioles\t\nThe right answer is D: Dilation of the intracerebral arterioles.", + "output": "The patient's intermittent throbbing headache with characteristics like being bilateral, \"band-like,\" and without any accompanying symptoms such as nausea, vomiting, photophobia, or phonophobia, suggests a tension-type headache, which is primarily caused by the dilation of intracerebral arterioles (Option D). The fact that her headache worsens with coughing or laughing also supports a vascular cause, as these actions can transiently increase intracranial pressure and thereby affect the arterioles. Although she was recently diagnosed with stable angina and has hypertension, there is no indication of neurological deficits or signs that would suggest a subdural hematoma (Option A), vessel dissection (Option B), or a growing neoplasm (Option C). Her vital signs and physical examination do not suggest meningitis or intracranial pressure elevation. Therefore, the most likely mechanism is vascular, involving the dilation of intracerebral arterioles." + }, + { + "input": "Question: A 62-year-old woman with small-cell lung cancer comes to the physician because of a painful, burning red rash on her face and hands that developed 30 minutes after going outside to do garden work. She wore a long-sleeved shirt and was exposed to direct sunlight for about 10 minutes. The patient is light-skinned and has a history of occasional sunburns when she does not apply sunscreen. Current medications include demeclocycline for malignancy-associated hyponatremia and amoxicillin for sinusitis. Physical examination shows prominent erythema and a papular eruption on her forehead, cheeks, neck, and the dorsum of both hands. Which of the following is the most likely cause of this patient's symptoms?\nOptions: A: Degranulation of presensitized mast cells\tB: Formation of drug-induced autoantibodies\tC: Accumulation of porphobilinogen\tD: Cell injury due to drug metabolites\t\nThe right answer is D: Cell injury due to drug metabolites.", + "output": "The patient's symptoms of a painful, burning red rash on the areas exposed to sunlight are indicative of a photosensitive reaction. Demeclocycline, a medication she is taking for her hyponatremia, is a tetracycline antibiotic known to cause photosensitivity. Photosensitivity is a skin reaction associated with exposure to ultraviolet (UV) radiation and the presence of certain chemicals in the body that react to UV light. In this case, demeclocycline's metabolites absorb UV light, leading to cell injury and the visible skin manifestations. The onset of the rash soon after sun exposure and its localization to sun-exposed areas supports this rationale. Other options such as mast cell degranulation or autoantibody formation would not cause such a rapid, UV light-dependent reaction, while porphobilinogen accumulation is related to porphyria, which does not fit the described symptoms. Therefore, the most likely cause is cell injury due to drug metabolites (Option D)." + }, + { + "input": "Question: A 25-year-old female is brought to the emergency department by her family. They found her somnolent next to her computer and scattered pill bottles over two hours ago. The family cannot recall the types of pill bottles that they found and did not bring them to the ED. The patient's past medical history is notable for anxiety, obesity, hypertension, and polycystic ovarian syndrome. She is currently not on any medications other than an exercise program with which she is non-compliant. A physical exam is performed and is within normal limits. The patient begins communicating with the physician and states that she did take acetaminophen but it was only a few pills. Her temperature is 99.5°F (37.5°C), pulse is 82/min, blood pressure is 125/85 mmHg, respirations are 11/min, and oxygen saturation is 97% on room air. Which of the following is the next best step in management?\nOptions: A: N-acetylcysteine\tB: Charcoal\tC: IV fluids\tD: Syrup of ipecac\t\nThe right answer is A: N-acetylcysteine.", + "output": "Considering the scenario where a young woman is found somnolent with scattered pill bottles, suggesting a possible overdose, and her claim of ingesting acetaminophen, the next best step in management is A, N-acetylcysteine (NAC). Acetaminophen overdose can lead to severe hepatotoxicity; NAC is the antidote and acts to replenish glutathione stores, mitigating liver damage. The presence of pills and somnolence warrants prompt treatment regardless of the quantity reported by the patient, since the patient's reliability is uncertain. The other options, such as activated charcoal (B), are most beneficial within an hour of ingestion, and the time frame exceeded this. IV fluids (C) support hydration but do not counteract the overdose. Syrup of ipecac (D) is not recommended due to potential complications and lack of efficacy in poison management." + }, + { + "input": "Question: A 33-year-old woman at 17-weeks gestation comes to the physician for evaluation of a painless mass in the right breast for the last 3 weeks that has not increased in size. Physical examination shows dense breasts and a 2.5-cm rubbery, mobile mass that is nontender in the upper outer quadrant of the right breast. A biopsy of the mass shows rounded contour with overgrowth of fibrous and glandular tissue. Which of the following is the most likely diagnosis?\nOptions: A: Fibroadenoma\tB: Normal pregnancy-related changes\tC: Fat necrosis\tD: Phyllodes tumor\t\nThe right answer is A: Fibroadenoma.", + "output": "The most likely diagnosis is A. Fibroadenoma. This benign tumor is consistent with the clinical findings: a well-circumscribed, rubbery, mobile, nontender mass in a young patient. Fibroadenomas are commonly found in women in their 20s and 30s and often do not change rapidly in size. The biopsy showing an overgrowth of fibrous and glandular tissue aligns with the pathology of a fibroadenoma. Pregnancy-related changes usually involve generalized breast changes rather than a solitary discrete mass. Fat necrosis typically presents with a history of trauma and potentially skin changes, and a Phyllodes tumor tends to grow more quickly and is less common than fibroadenoma." + }, + { + "input": "Question: A 30-year-old woman presents to an outpatient clinic for a scheduled gynecologic check-up. The patient appears well and has no specific complaints. She is current with her Pap smears, all of which have been normal. Her menstrual periods are regular and usually last 3–4 days. She denies any recent travel or a history of trauma. The patient has not been sexually active for the past 2 months. She smokes 1 pack of cigarettes per day and drinks alcohol occasionally, but has never used illicit drugs. The pelvic examination reveals a small, firm, non-tender, immobile cystic mass at the edge of the exocervical os with a yellow hue. The patient is otherwise asymptomatic and has no other complaints. The remainder of the physical examination is unremarkable. Which of the following is the next best step in management?\nOptions: A: No further intervention\tB: Colposcopy and biopsy\tC: Administration of oral contraceptives\tD: Administration of antibiotics\t\nThe right answer is A: No further intervention.", + "output": "The key information in this scenario is the discovery of a small, firm, non-tender, immobile cystic mass with a yellow hue at the edge of the exocervical os. Given the mass's characteristics and the patient's asymptomatic status—and considering her regular, normal Pap smears—it is highly suggestive of a Nabothian cyst. Nabothian cysts are benign, common, and usually do not require intervention unless they are symptomatic or cause clinical concern due to size or appearance. Colposcopy and biopsy (Option B) are typically reserved for suspicious lesions that indicate potential malignancy, which this presentation does not suggest. Neither oral contraceptives (Option C) nor antibiotics (Option D) would be indicated, as there’s no evident hormonal or infectious condition present. Therefore, \"No further intervention\" (Option A) is the best choice to manage a simple Nabothian cyst in an asymptomatic patient." + }, + { + "input": "Question: An investigator is studying the growth pattern of a particular bacterial strain that caused a respiratory disease outbreak in children in rural Venezuela. The bacteria are found to be able to grow in a specialized culture that contains sheep blood, cystine, and potassium tellurite; the colonies are black and have a shining surface. The isolated bacteria are most likely which of the following?\nOptions: A: Legionella pneumophila\tB: Haemophilus influenzae\tC: Bordetella pertussis\tD: Corynebacterium diphtheriae\t\nThe right answer is D: Corynebacterium diphtheriae.", + "output": "The rationale for selecting option D, Corynebacterium diphtheriae, as the most likely bacteria responsible for the described condition lies in the uniqueness of the growth requirements and colony characteristics. Corynebacterium diphtheriae is known to grow on media containing cystine and potassium tellurite, which inhibits the growth of other bacteria while allowing diphtheriae to thrive. The black colonies with a shining surface are distinctive for this bacterium when grown on potassium tellurite media, as it reduces tellurite to tellurium, leading to the formation of black colonies. The other bacteria listed do not exhibit these specific growth patterns and colony morphologies in the presence of these media components, thus leading to the conclusion that the isolated bacteria are Corynebacterium diphtheriae." + }, + { + "input": "Question: A 4-year-old boy with acute lymphoblastic leukemia is admitted to the hospital to undergo allogeneic bone marrow transplantation. Two weeks after the conditioning regimen is started, he develops a temperature of 38.5°C (101.3°F). Laboratory studies show:\nHemoglobin 8 g/dL\nLeukocyte count 1400/mm3\nSegmented neutrophils 20%\nEosinophils 0.5%\nLymphocytes 87%\nMonocytes 1%\nPlatelet count 110,000/mm3\nWhich of the following is the most appropriate pharmacotherapy for this patient?\"\nOptions: A: Transforming growth factor-β\tB: Erythropoietin\tC: Interleukin-5\tD: Granulocyte-macrophage colony-stimulating factor\t\nThe right answer is D: Granulocyte-macrophage colony-stimulating factor.", + "output": "The most appropriate pharmacotherapy for this patient is D. Granulocyte-macrophage colony-stimulating factor (GM-CSF). The patient's laboratory studies indicate neutropenia (segmented neutrophils at 20% of a low leukocyte count), which is a common complication after bone marrow transplantation and the conditioning regimen. Neutropenia puts the patient at high risk for infections, as evidenced by the fever. GM-CSF is a hematopoietic growth factor that stimulates the proliferation and differentiation of neutrophil precursors, which can help reduce the duration of neutropenia and its associated risk of infection. The other options, such as erythropoietin or IL-5, are not directly relevant to the current primary concern, which is the patient's neutropenia and its consequent risk of infection. Transforming growth factor-β is not typically used in this context, as it can actually inhibit immune cell proliferation." + }, + { + "input": "Question: A 49-year-old man comes to the physician because of a 1-week history of diarrhea and abdominal bloating. His stools are bulky, foul-smelling, and difficult to flush. Over the past 6 months, he has had recurrent dull epigastric pain that is triggered by meals and lasts for a few days. He drinks 6 to 8 beers daily. Abdominal examination shows mild epigastric tenderness with no rebound or guarding. A CT scan of the abdomen is shown. The structure indicated by the arrows is most likely lined by which of the following?\nOptions: A: Simple ductal epithelium\tB: Granulation tissue\tC: Pyogenic membrane\tD: Columnar mucinous epithelium\t\nThe right answer is B: Granulation tissue.", + "output": "The presentation of bulky, foul-smelling stools that are difficult to flush alongside a history of epigastric pain triggered by meals suggests malabsorption, possibly due to chronic pancreatitis, which is consistent with the patient's significant alcohol intake. Chronic pancreatitis can lead to the formation of cystic structures, often referred to as pseudocysts, which are typically lined by granulation tissue, not a specialized epithelium. Granulation tissue forms as part of the healing process during inflammation and is composed of new connective tissue and tiny blood vessels. Thus, the structure indicated by the arrows on the CT scan is most likely lined by granulation tissue, making option B the correct answer. Options A, C, and D are less likely because ductal epithelium lines ducts, pyogenic membranes are associated with abscesses, and columnar mucinous epithelium lines areas such as the gut, but not pseudocysts." + }, + { + "input": "Question: A 15-year-old girl is brought to the physician by her mother because of worsening grades over the past year. Since she started high school one year ago, her academic performance has decreased. She also has had difficulty finding friends at the new school. She is afraid that her classmates will make fun of her and think that she is “stupid.” One month ago, when she had to give a presentation, she could not stop wondering how her classmates were going to react if she said something wrong. During the presentation, her heart started racing and she became flushed. Since this event, she avoids saying anything in class. She spends her breaks in the restroom because she is worried that nobody will talk to her. Physical and neurologic examinations show no abnormalities. On mental status examination, the girl avoids eye contact and appears uncomfortable and anxious. Which of the following is the most appropriate pharmacotherapy for this patient's condition?\nOptions: A: Clomipramine\tB: Phenelzine\tC: Fluoxetine\tD: Propranolol\t\nThe right answer is C: Fluoxetine.", + "output": "The most appropriate pharmacotherapy for this patient's condition is option C, Fluoxetine. The patient displays signs of social anxiety disorder, characterized by intense fear of social situations, concern about being embarrassed, and avoidance behaviors which are impairing her functioning at school. Fluoxetine is a selective serotonin reuptake inhibitor (SSRI) that is first-line treatment for anxiety disorders in adolescents due to its efficacy, safety profile, and relatively mild side-effects. Clomipramine, an older tricyclic antidepressant with more side effects, is not the first choice. Phenelzine, an MAOI, has dietary restrictions and more potential side effects. Propranolol, a beta-blocker, can help with performance anxiety symptoms but does not address the underlying anxiety disorder. Thus, fluoxetine is the most suitable option." + }, + { + "input": "Question: A 67-year-old man is brought to the hospital by his relatives. He complains of early satiety, abdominal discomfort after food intake, and abdominal bloating. These symptoms have been present since the patient was 52, but they were milder at that time so he paid little attention to them since. Tingling and decreased sensation in both lower extremities began a few months ago. His relatives also note that he has become excessively somnolent and forgetful. The patient’s medical history is significant for acute hepatitis B 20 years ago. He smokes a pack of cigarettes per day and drinks alcohol occasionally. His weight is 61 kg (134.5 lb), height is 181 cm (5 ft 11 in), and BMI is 18.6 kg/m2. His vital signs include: blood pressure 110/80 mm Hg, heart rate 89/min, respiratory rate 13/min, and temperature 36.1°C (96.9°F). The patient’s skin is pale, thin, dry, and hypoelastic. Lymph nodes and the thyroid gland are not enlarged. Lung and heart examinations are within normal limits for his age. The patient’s tongue is bright red with atrophic papillae. The abdomen is distended and tender to palpation in the epigastric area. Neurological examination shows symmetrical bilateral distal hypoesthesia for all sensations and decreased muscle strength in both upper and lower extremities. On a mini-mental status examination, the patient scores 25. The patient’s blood test results are as follows:\nErythrocytes 2.8 x 109/mm3\nHb 8.6 g/dL\nHt 37%\nMean corpuscular hemoglobin 49.9 pg/cell (3.1 fmol/cell)\nMean corpuscular volume 142 µm3 (142 fL)\nReticulocyte count 0.3%\nTotal leukocyte count 3,070/mm3\nNeutrophils 54%\nLymphocyte 32%\nEosinophil 3%\nMonocyte 11%\nBasophil 0%\nPlatelet count 195,000/mm3\nHbA1C 4.3%\nFasting plasma glucose 4.6 mmol/l (82.9 mg/dL)\nAspartate aminotransferase 22 U/L\nAlanine aminotransferase 19 U/L\nAmylase 32 U/L\nTotal bilirubin 32 µmol/L (1.87 mg/dL)\nConjugated bilirubin 8 µmol/L (0.4677 mg/L)\nWhich of the following options best explains the patient’s neurological and psychiatric symptoms?\nOptions: A: Chronic bilirubin toxicity\tB: Chronic acetaldehyde influence\tC: Impaired methylation of myelin phospholipids\tD: Influence of pseudo-neurotransmitters\t\nThe right answer is C: Impaired methylation of myelin phospholipids.", + "output": "The patient presents with early satiety, abdominal discomfort, memory deficits, excessive somnolence, neurological symptoms, and megaloblastic anemia evidenced by low erythrocytes, low Hb, high mean corpuscular volume (MCV), and high mean corpuscular hemoglobin (MCH). This clinical presentation suggests vitamin B12 deficiency, which impairs DNA synthesis in rapidly dividing cells, like those in the bone marrow, leading to anemia. Vitamin B12 deficiency also causes demyelination of the nervous system due to impaired methylation of myelin phospholipids, explaining the neurological and psychiatric symptoms. Chronic bilirubin toxicity, acetaldehyde influence, and pseudo-neurotransmitters do not typically present with a high MCV or the described combination of neurological and psychiatric symptoms, so these options are less likely. Hence, option C, \"Impaired methylation of myelin phospholipids,\" is the best explanation for this patient's complex presentation." + }, + { + "input": "Question: A 53-year-old male presents to his primary care physician complaining of fatigue and joint and muscle pain. He reports a 6-month history of slowly progressive tiredness associated with dull achy pain in his knees, hips, and shoulders. His past medical history is notable for hypertension, diabetes, and stable angina. He takes lisinopril, metformin, glyburide, and aspirin. He has a history of intravenous drug use and was homeless in his early 30’s. He drinks alcohol socially and has a 30 pack-year smoking history. His temperature is 98.6°F (37°C), blood pressure is 130/85 mmHg, pulse is 95/min, and respirations are 18/min. Physical examination reveals a generally well-appearing male without evidence of ascites, peripheral edema, or jaundice. Results from laboratory tests are below:\n\nHepatitis A IgM: Negative\nHepatitis A IgG: Negative\nHepatitis B surface antigen: Negative\nHepatitis B E antigen: Negative\nHepatitis B core antigen: Negative\nHepatitis B surface IgG: Positive\nHepatitis B E IgG: Negative\nHepatitis B core IgG: Negative\nHepatitis C IgG: Positive\nHCV RNA: 100,000,000 IU/ml\n\nThis patient should be started on which of the following medications?\nOptions: A: Sofosbuvir and simeprevir\tB: Interferon-gamma\tC: Ribavirin\tD: Lamivudine\t\nThe right answer is A: Sofosbuvir and simeprevir.", + "output": "Option A, Sofosbuvir and simeprevir, is the appropriate choice for this patient who presents with HCV RNA indicating active Hepatitis C infection. His symptoms of fatigue and joint and muscle pain may be related to chronic Hepatitis C. The history of intravenous drug use presents a risk factor for HCV infection. Sofosbuvir and simeprevir are direct-acting antivirals that are effective for treating Hepatitis C. Interferon-gamma (option B) is not used in the treatment of HCV, and Ribavirin (option C) is rarely used alone due to its side effects and lower efficacy as monotherapy. Lamivudine (option D) is used for Hepatitis B, not C." + }, + { + "input": "Question: A 62-year-old woman with a history of hypertension and type 2 diabetes mellitus comes to the physician for a routine health maintenance examination. She has smoked 1 pack of cigarettes daily for the last 15 years. Current medications include glyburide and amlodipine. The physician prescribes a medication that decreases the production of mevalonate. Which of the following changes to the serum is most likely to develop as an adverse effect of the prescribed drug?\nOptions: A: Increased creatine kinase concentration\tB: Decreased glucose concentration\tC: Increased triglyceride concentration\tD: Increased bradykinin concentration\t\nThe right answer is A: Increased creatine kinase concentration.", + "output": "The physician most likely prescribed a statin, which is a class of drug that inhibits HMG-CoA reductase, thereby decreasing the production of mevalonate, a precursor to cholesterol. Statins are well known to be used in patients with cardiovascular risk factors such as hypertension and type 2 diabetes mellitus, and they can also provide benefit in patients who smoke by reducing cardiovascular events. One well-documented adverse effect of statins is myopathy, which can range from mild muscle pain to severe rhabdomyolysis. Creatine kinase (CK) is an enzyme released into the serum during muscle breakdown, so an increased serum concentration of CK is a potential adverse effect of statins indicating muscle injury. This is why option A, increased creatine kinase concentration, is the most likely serum change to develop as an adverse effect of the drug. Options B, C, and D are less likely to be direct adverse effects of statins." + }, + { + "input": "Question: A previously healthy, 24-year-old man comes to the physician because of a 6-week history of loose, nonbloody stools. He also reports abdominal pain, intermittent nausea, and fever. He has not had vomiting, tenesmus, or rectal pain. His vital signs are within normal limits. Abdominal examination reveals tenderness of the right lower abdomen without rebound tenderness. Rectal exam is unremarkable. Laboratory studies show a leukocyte count of 14,800/mm3 and an erythrocyte sedimentation rate of 51 mm/h. Test of the stool for occult blood and stool studies for infection are negative. A CT scan of the abdomen shows mural thickening and surrounding fat stranding of discrete regions of the terminal ileum and transverse colon. A colonoscopy is performed and biopsy specimens of the affected areas of the colon are taken. Which of the following findings is most specific for this patient's condition?\nOptions: A: Neutrophilic inflammation of the crypts\tB: Neutrophil-rich pseudomembranes\tC: Formation of noncaseating granulomas\tD: Presence of pseudopolyps\t\nThe right answer is C: Formation of noncaseating granulomas.", + "output": "The patient's symptoms and diagnostic tests suggest an inflammatory bowel disease, specifically Crohn's disease. The normal vital signs with chronic symptoms, the leukocytosis, elevated erythrocyte sedimentation rate (ESR), and imaging revealing terminal ileum and transverse colon involvement all fit with Crohn's disease. Option C, the formation of noncaseating granulomas, is most specific for Crohn's disease. Unlike the other options that could be seen in various inflammatory or infectious conditions, noncaseating granulomas are a hallmark of Crohn's pathology and are not found in conditions such as ulcerative colitis or infectious colitis, which could present with similar clinical symptoms. Neutrophilic inflammation of the crypts (A) can occur in any inflammatory condition of the colon, neutrophil-rich pseudomembranes (B) are associated with Clostridioides difficile infection, and presence of pseudopolyps (D) can occur in any chronic inflammatory condition of the colon." + }, + { + "input": "Question: A 15-year-old boy is brought to the emergency department because of severe abdominal pain and vomiting for 8 hours. He has had around 3–4 episodes of greenish colored vomit during this period. He has major depressive disorder with psychosis. His mother has Graves' disease. Current medications include sertraline and haloperidol. He appears uncomfortable. His temperature is 37.3°C (99.1°F), pulse is 87/min, and blood pressure is 118/72 mm Hg. He is diagnosed with acute appendicitis and is taken to the operating room for an emergency laparoscopic appendectomy. The appendix is gangrenous and perforated with purulent fluid in the pelvis. Just after the appendiceal base is ligated, the patient shows muscle rigidity and profuse diaphoresis. His temperature is 39.1°C (102.4°F), pulse is 130/min, and blood pressure is 146/70 mm Hg. The pupils are equal and reactive to light. The end-tidal CO2 is 85 mm Hg. Which of the following is the most appropriate treatment for this patient's condition?\nOptions: A: Dantrolene\tB: Bromocriptine\tC: Cyproheptadine\tD: Surgical embolectomy\t\nThe right answer is A: Dantrolene.", + "output": "The patient's symptoms suggest malignant hyperthermia (MH), a life-threatening condition triggered by certain drugs used during anesthesia, often in genetically predisposed individuals. The clinical presentation includes a rapid increase in body temperature, muscle rigidity, a rapid pulse, and elevated end-tidal CO2. The use of sertraline and haloperidol is not directly linked to MH. However, haloperidol may predispose to neuroleptic malignant syndrome, which shares features with MH but develops over days to weeks, not acutely. The family history of Graves' disease is noncontributory in this context. Dantrolene is the drug of choice for MH because it acts directly on the ryanodine receptors in the muscle cells to halt the excessive calcium release that contributes to muscle rigidity and hypermetabolism. Bromocriptine and cyproheptadine are treatments for neuroleptic malignant syndrome and serotonin syndrome, respectively, and surgical embolectomy is for arterial emboli, not relevant here." + }, + { + "input": "Question: A 45-year-old man comes to the physician for a routine health maintenance examination. He was diagnosed with HIV 15 years ago. He was taking triple antiretroviral therapy but stopped a few months ago because he was feeling well. He lives in Wyoming. Vital signs are within normal limits. Cardiopulmonary examination shows no abnormalities. His CD4+ T-lymphocyte count is 47/mm3 (N ≥ 500). The patient currently refuses to restart antiretroviral therapy. Which of the following medication regimens is most appropriate at this time?\nOptions: A: Azithromycin and itraconazole\tB: Azithromycin and amphotericin B\tC: Trimethoprim, sulfamethoxazole, azithromycin\tD: Dapsone, pyrimethamine, itraconazole, azithromycin\t\nThe right answer is C: Trimethoprim, sulfamethoxazole, azithromycin.", + "output": "The patient's CD4+ T-lymphocyte count of 47/mm3 indicates severe immunosuppression and warrants prophylaxis against opportunistic infections. The best choice among the options is C. Trimethoprim-sulfamethoxazole (TMP-SMX) is used for Pneumocystis jirovecii pneumonia (PJP) prophylaxis, and at the dose for PJP prophylaxis, it also provides coverage against toxoplasmosis. Azithromycin is added for prophylaxis against Mycobacterium avium complex (MAC), another common opportunistic infection in patients with advanced HIV who have a CD4+ count less than 50/mm3. The other options lack the broad coverage necessary for this patient or address different clinical concerns not indicated by the patient's current status." + }, + { + "input": "Question: A 24-year-old woman comes to the physician because of persistent fatigue for the past 4 months. She has no history of major medical illness. Her temperature is 36°C (96.8°F), pulse is 121/min, and blood pressure is 120/78 mm Hg. Physical examination shows pale conjunctivae. A peripheral blood smear is shown. Which of the following is the most likely cause of this patient's symptoms?\nOptions: A: Hereditary spherocytosis\tB: Vitamin B12 deficiency\tC: Iron deficiency\tD: β-thalassemia minor\t\nThe right answer is C: Iron deficiency.", + "output": "The patient's symptoms of persistent fatigue and the physical examination finding of pale conjunctivae suggest anemia. The high pulse rate could be compensatory tachycardia due to anemia. Among the given options, iron deficiency (C) is the most common cause of anemia, especially in young women who might experience heavy menstrual bleeding. Beta-thalassemia minor (D) is less likely in the absence of relevant family history or ethnic background. Hereditary spherocytosis (A) and vitamin B12 deficiency (B) are less common, and B12 deficiency typically presents with neurological symptoms alongside anemia. Without the peripheral smear image, we deduce that iron deficiency anemia is the most probable diagnosis based on the clinical scenario provided." + }, + { + "input": "Question: A 40-year old man presents to the clinic with vague upper abdominal pain that has been worsening for the last several days. He says that he often gets similar pain but less severe and that it worsens with meals. The pain sometimes radiates to his back. He recently lost 15 kg (33.6 lb) of weight secondary to his lack of appetite since his last visit 2 months ago. He admits to drinking alcohol almost every night since the age of 17. His temperature is 37.0° C (98.6°F), respirations are 15/min, pulse is 67/min, and blood pressure is 122/98 mm Hg. Physical examination reveals moderate epigastric tenderness. An abdominal CT scan reveals calcifications as shown by the white arrows. Which of the following organs is affected in this patient?\nOptions: A: Pancreas\tB: Jejunum\tC: Duodenum\tD: Gall bladder\t\nThe right answer is A: Pancreas.", + "output": "The patient's symptoms of worsening upper abdominal pain, weight loss, pain that aggravates with meals, and pain radiating to the back, along with a history of chronic alcohol use, are classic for chronic pancreatitis. Chronic pancreatitis often presents with calcifications in the pancreas, which can be seen on imaging such as a CT scan. The presence of calcifications in the abdomen on the CT scan is indicative of long-standing inflammation of the pancreas, hence linking the findings to option A, the Pancreas. Other options (Jejunum, Duodenum, Gallbladder) are less likely to present with this constellation of symptoms and calcifications and they do not have a direct association with chronic alcohol consumption." + }, + { + "input": "Question: A 36-year-old man presents to his physician for a regular checkup, and he expresses an intent to quit smoking. He is a financial director with a 15-pack-year history of smoking who tried quitting several times. He tried different nicotine replacement products, none of which helped him. The patient has gastrointestinal reflux disease and takes omeprazole. The medical history is significant for a head trauma after which he had two seizure episodes. He does not have a history of any other disorders. Currently, he is not on any antiepileptic medication. He heard from his friend about an antidepressant that efficiently helps with smoking cessation. He would like to try this medication, and asks his physician for an advice. Which of the following statements regarding the use of the mentioned medication is correct?\nOptions: A: Omeprazole interferes with bupropion metabolism and should be taken one hour prior to omeprazole consumption.\tB: Bupropion is only effective when used together with nicotine replacement products.\tC: Bupropion lowers the seizure threshold and it should not be taken by patients with a history of seizure disorder.\tD: The physician may prescribe bupropion for this patient only after a full cardiac workup is performed.\t\nThe right answer is C: Bupropion lowers the seizure threshold and it should not be taken by patients with a history of seizure disorder..", + "output": "Bupropion is an antidepressant that is also indicated for smoking cessation. However, one important side effect is its potential to lower the seizure threshold. This means it can make seizures more likely in predisposed individuals. Given that the patient has a history of head trauma followed by seizure episodes, this risk is particularly relevant. Bupropion should be used with caution or avoided in individuals with a history of seizures. None of the other options are correct or relevant to the specific concerns raised. Omeprazole does not have a significant interaction with bupropion metabolism, bupropion can be effective without nicotine replacement products, and a full cardiac workup is not routinely required before prescribing bupropion unless there are other underlying cardiovascular concerns. Therefore, recommendation C is the correct statement, advising against the use of bupropion in a patient with a seizure history." + }, + { + "input": "Question: A 47-year-old female presents to her primary physician for follow up after an ED visit for nephrolithiasis 1 week prior. CT scan in the ED revealed a 4-mm stone occluding the right ureter. She was able to pass the stone without assistance over the course of 5 hours. Pathology report on the stone states it was composed of calcium oxalate. She says that the pain of passing the stone was excruciating and asks how she can prevent kidney stones in the future. Which of the following interventions is most likely to prevent renal calculi formation in the future for this patient?\nOptions: A: Decreasing calcium intake\tB: Decreasing sodium intake\tC: Increasing consumption of nuts and seeds\tD: Urine acidification with cranberry juice\t\nThe right answer is B: Decreasing sodium intake.", + "output": "Decreasing sodium intake is the most effective intervention to prevent renal calculi formation for this patient. Excessive sodium in the diet leads to increased calcium in the urine, which can combine with oxalate to form stones. By reducing sodium intake, urinary calcium levels may decrease, lowering the risk of stone formation. Calcium intake should not be decreased as dietary calcium can bind with oxalate in the intestines, reducing the oxalate absorption and subsequent stone formation. Increasing consumption of nuts and seeds, which are high in oxalate, may actually increase kidney stone risk. Urine acidification is not typically used for calcium oxalate stones." + }, + { + "input": "Question: A 30-year-old Japanese woman is brought to the emergency department after fainting at work. She says she was only unconscious for a moment and recovered quickly. She reports increasing fatigue and joint pain for the past 4 months. Her blood pressure is 90/60 mm Hg, and her temperature is 36.6°C (98.0°F). On physical examination, the patient is fully conscious. Her radial pulse is absent at her right wrist and 1+ at her left wrist. Laboratory findings are significant for the following:\nHemoglobin: 10.9 g/dL\nHematocrit: 34.7%\nLeukocyte count: 5,500/mm3\nNeutrophils: 65%\nLymphocytes: 30%\nMonocytes: 5%\nMean corpuscular volume: 78.2 μm3\nPlatelet count: 190,000/mm3\nErythrocyte sedimentation rate: 56 mm/h\nWhat complications is this patient at risk for?\nOptions: A: Myocardial infarction\tB: Tongue infarction\tC: Palpable purpura\tD: Polyneuropathy\t\nThe right answer is A: Myocardial infarction.", + "output": "The patient's absent radial pulse on the right wrist, coupled with lower blood pressure, suggest decreased perfusion which could be indicative of a form of vasculitis, such as Takayasu arteritis, which predominantly affects women of Asian descent in their 20s and 30s. Takayasu arteritis may lead to stenosis or occlusion of the aorta and its branches. Fatigue, joint pain, and the high erythrocyte sedimentation rate (ESR) further support an inflammatory process. With Takayasu arteritis, complications such as myocardial infarction are possible due to impaired blood flow to the coronary arteries. The other options – tongue infarction, palpable purpura, and polyneuropathy – are more characteristic of other conditions and not directly linked to the vascular changes suggested by the symptoms and demographic in the given case. Therefore, option (A) Myocardial infarction is the complication this patient is most at risk for based on the given clinical presentation." + }, + { + "input": "Question: A 45-year-old woman presents to the physician with a history of headache, palpitations, and episodic diaphoresis. She was diagnosed with hypertension in the past and is now regularly taking antihypertensive medication (amlodipine 10 mg once daily). She has had a diffuse thyroid swelling for the past 4 months. She complains of pain in the long bones of her arms and legs for the past 3 months. She is slightly built, appears thin, and is anxious and apprehensive. Her skin is moist and warm. Her weight is 45 kg (99.2 lb) and height is 146 cm (4 ft 7 in); BMI is 21.12 kg/m2. Her pulse is 116/min, the blood pressure is 196/102 mm Hg, the respiratory rate is 29/min, and the temperature is 37.2°C (99.0°F). Diffuse and firm thyromegaly is present. Her upper extremities exhibit fine tremors during an anxiety attack. The laboratory test results reveal elevated 24-hour urinary VMA and metanephrines. Serum epinephrine, calcitonin, and parathormone levels are also elevated. Hypercalcemia and hyperglycemia are also noted. Which of the following is the most likely diagnosis?\nOptions: A: Von Hippel-Lindau syndrome\tB: Multiple endocrine neoplasia (MEN) 2A\tC: Multiple endocrine neoplasia (MEN) 2B\tD: Neurofibromatosis\t\nThe right answer is B: Multiple endocrine neoplasia (MEN) 2A.", + "output": "The patient presents with a combination of symptoms that are characteristic of Multiple Endocrine Neoplasia (MEN) type 2A: hypertension with elevated urine vanillylmandelic acid (VMA) and metanephrines suggesting a pheochromocytoma; hyperparathyroidism indicated by elevated parathormone levels and hypercalcemia; and thyroid pathology inferable from diffuse thyromegaly along with elevated calcitonin, which is indicative of medullary thyroid carcinoma. The presentation is not as consistent with MEN 2B, which is usually associated with distinctive mucosal neuromas and a marfanoid habitus, or with Von Hippel-Lindau syndrome, which is typically associated with hemangioblastomas and renal cell carcinoma, nor neurofibromatosis, which typically presents with café-au-lait spots and neurofibromas. Therefore, MEN 2A is the most likely diagnosis given the constellation of endocrine abnormalities more commonly associated with that syndrome." + }, + { + "input": "Question: A 45-year-old woman presents with sudden-onset dyspnea. Her past medical history is significant for varicose veins in her lower extremities, well-controlled diabetes mellitus type 2, and a 25 pack-year smoking history. The patient currently takes metformin and pioglitazone. Family history is significant for her sister who died at the age of 35 because of a pulmonary embolism. Her temperature is 37.8°C (100.0°F), the pulse is 111/min, the respirations are 18/min, and the blood pressure is 130/60 mm Hg. On physical examination, the patient is pale and diaphoretic. Cardiac exam is significant for an S3 gallop. There is erythema and swelling over multiple varicose veins in her left leg. Ultrasound of her left leg is positive for a deep vein thrombosis. The patient is admitted, and anticoagulation is started with a vitamin K inhibitor. However, despite these interventions, she dies shortly after admission. Which of the following was the most likely etiology of this patient’s condition?\nOptions: A: Distal peripheral microembolism\tB: Necrotizing fasciitis\tC: Protein S deficiency\tD: Waldenström macroglobulinemia\t\nThe right answer is C: Protein S deficiency.", + "output": "Given the patient's sudden-onset dyspnea, varicose veins, well-controlled diabetes, smoking history, and especially her family history of pulmonary embolism, the most likely etiology appears related to a hypercoagulable state. Protein S deficiency is a genetic condition leading to hypercoagulability, and can predispose individuals to thrombotic events such as deep vein thrombosis (DVT) and pulmonary embolism (PE), which aligns with the patient's history and clinical presentation. Her sister's death from PE also suggests a possible inherited thrombophilia. The option of Protein S deficiency (C) postulates a genetic factor that likely contributed to both this patient's and her sister's conditions, coherent with the clinical picture of familial thrombotic events." + }, + { + "input": "Question: A 37-year-old woman with an HIV infection comes to the physician for a follow-up examination. Six months ago, combined antiretroviral therapy consisting of dolutegravir, tenofovir, and emtricitabine was initiated. Laboratory studies show a decrease in the CD4 count and an increase in the viral load despite ongoing treatment. The patient is switched to a new treatment regimen, including a drug that acts by preventing viral DNA synthesis without undergoing intracellular phosphorylation. Which of the following is the most likely drug?\nOptions: A: Efavirenz\tB: Ritonavir\tC: Raltegravir\tD: Lamivudine\n\"\t\nThe right answer is A: Efavirenz.", + "output": "Efavirenz is the most likely drug in the given scenario because it functions as a non-nucleoside reverse transcriptase inhibitor (NNRTI). NNRTIs bind directly to the reverse transcriptase enzyme, inhibiting the RNA- to DNA transcription that is necessary for HIV replication. This action occurs without the requirement of intracellular phosphorylation, a step necessary for the activation of nucleoside reverse transcriptase inhibitors (NRTIs) like lamivudine and for the actions of tenofovir and emtricitabine. Ritonavir acts as a protease inhibitor, while raltegravir is an integrase inhibitor, neither of which fit the described mechanism of action." + }, + { + "input": "Question: A 14-year-old girl presents with sudden drooping of the right side of her face with drooling and excessive tearing. The patient’s mother says that the patient was recently in northern Maine and spent most of her time during the trip outdoors. Physical examination reveals a slight asymmetry of the facial muscles with an inability to whistle or close the right eye. A circular red rash with central clearing is present on the trunk. There is also decreased taste sensation. Which of the following most likely transmitted the organism responsible for this patient’s illness?\nOptions: A: Ixodes scapularis\tB: Tsetse fly\tC: Culicidae\tD: Anopheles\t\nThe right answer is A: Ixodes scapularis.", + "output": "The clinical presentation of a sudden-onset facial droop (Bell's palsy), drooling, excessive tearing, and a circular rash with central clearing (erythema migrans) strongly suggests Lyme disease, which is endemic in northern states like Maine. Additionally, the decreased taste sensation can also be associated with nervous system involvement in Lyme disease. Ixodes scapularis, the black-legged tick (also known as the deer tick), is the vector known to transmit Borrelia burgdorferi, the causative agent of Lyme disease, in North America. The other listed options, such as tsetse fly, Culicidae (mosquito family), and Anopheles (a genus of mosquito), are responsible for the transmission of other diseases and are not typically involved in the transmission of the organism causing the symptoms described here." + }, + { + "input": "Question: A 60-year-old woman presents to the outpatient clinic complaining of severe vulvar itching that has been gradually worsening for the past year. She mentions that she has tried over-the-counter lubricants, but lubricants do not seem to provide substantial relief. Her medical history is remarkable for diet-controlled type 2 diabetes mellitus and hypertension treated with hydrochlorothiazide. Menopause occurred at 52 years of age. She has been sexually active with her husband until recently and reports severe pain during vaginal intercourse. The physical examination reveals dry, thin, white plaque-like lesions in the vulvar area with atrophy of the labia minora. The clitoris appears retracted. The perianal skin appears pale and wrinkled. Which of the following is the most likely diagnosis?\nOptions: A: Squamous cell hyperplasia\tB: Lichen sclerosus\tC: Squamous cell carcinoma of the vulva\tD: Lichen planus\t\nThe right answer is B: Lichen sclerosus.", + "output": "The most likely diagnosis is B. Lichen sclerosus, given the patient's presentation of severe vulvar itching, white plaque-like lesions, atrophy of the labia minora, retraction of the clitoris, and pale, wrinkled perianal skin. Squamous cell hyperplasia tends to cause thickened rather than thinned skin, and while squamous cell carcinoma could present with plaques, they are typically more irregular and can be ulcerative or erosive. Lichen planus also causes vulvar itching and white changes but is usually associated with a lacy or net-like pattern of white lesions rather than the atrophy and retraction seen in this case." + }, + { + "input": "Question: A 50-year-old woman with Systemic Lupus Erythematosus (SLE) presents to your clinic with complaints of left eye pain and reduced vision. She notes that she was in her usual state of health until 3 days prior to presentation when she noticed some mild tenderness around her left eye, unrelieved by any pain relievers. This morning when she woke up she was unable to see out of her left eye and made an appointment to come see you. Other than SLE, her medical history is unremarkable. She has had no additional symptoms other than pain and vision loss. Her vital signs are all within normal limits. On exam, she has no vision in her left eye, but 20/30 visual acuity in her right eye. When you shine a light in her left eye there is no response. What response would you expect when you shine a light in her right eye?\nOptions: A: Bilateral miosis\tB: Miosis of the right eye only\tC: Miosis of the left eye only\tD: Mydriasis of the left eye\t\nThe right answer is A: Bilateral miosis.", + "output": "The sudden vision loss and lack of pupillary response in the left eye suggest optic nerve involvement or severe retinal pathology. When light is shone into one eye, both pupils should constrict due to the direct and consensual pupillary light reflex. Despite the left eye's inability to see, the neural pathways for the pupillary reflex may still be intact. If so, when the light is shone into the non-responding left eye, the right eye should still constrict in response; this is the consensual reflex. Conversely, if light is shone into the right eye and the pathways are intact, both eyes should exhibit miosis due to the pupillary reflex (the direct response in the right eye and the consensual response in the left). Hence, shining light into either eye should, under normal circumstances, produce bilateral miosis if the reflex pathways are intact, which is why the correct answer is A, Bilateral miosis." + }, + { + "input": "Question: A 24-year-old woman comes to the emergency department with her boyfriend 2 hours after an episode of loss of consciousness. She was seated and was being tattooed on her right upper flank when she became diaphoretic, pale, and passed out. According to her boyfriend, the patient slipped to the floor and her arms and legs moved in a jerky fashion for a few seconds. She regained consciousness within half a minute and was alert and fully oriented immediately. She has no history of serious illness. She works as an event manager and has had more stress than usual due to a recent concert tour. She appears well. Her temperature is 37°C (98.4°F), pulse is 68/min, respirations are 16/min, and blood pressure is 120/72 mm Hg. Her cardiopulmonary examination shows no abnormalities. Neurologic examination shows no focal findings. An ECG shows an incomplete right bundle branch block. Which of the following is the most likely diagnosis?\nOptions: A: Vasovagal syncope\tB: Orthostatic syncope\tC: Cardiac arrhythmia\tD: Cardiac structural abnormality\n\"\t\nThe right answer is A: Vasovagal syncope.", + "output": "The most likely diagnosis is A. Vasovagal syncope. The patient's syncope occurred in a situation that could trigger a vasovagal response—while being tattooed, which is a painful and stressful event. Her symptoms (diaphoresis, pallor) followed by a brief loss of consciousness with quick and full recovery are characteristic of vasovagal syncope. The involuntary movements described by her boyfriend could suggest a convulsive syncope rather than a primary seizure. Her normal vital signs and lack of any cardiac history or abnormal findings on the cardiopulmonary examination make cardiac etiologies, such as arrhythmias or structural abnormalities, less likely. The incomplete right bundle branch block on the ECG can be a normal variant and is not necessarily indicative of an arrhythmogenic cause of her symptoms, especially in the absence of any history of heart disease. The stress from her job could also contribute to a vasovagal episode." + }, + { + "input": "Question: A 62-year-old man is brought to the physician by his wife because of difficulties walking for the past year. He has had multiple falls over the past 4 months. Over the past 6 months, he has had progressive urinary urgency and now wears an adult diaper as is not always able to control the urge to void. His appetite has also decreased in this time period. He used to drive, but his wife convinced him to stop after she noticed that he was becoming increasingly inattentive while driving. He has type 2 diabetes mellitus and hypertension. Current medications include ramipril and metformin. His temperature is 37.1C (98.8F), pulse is 90/min and blood pressure is 132/88 mm Hg. Examination shows a broad-based gait with slow and short steps. Neurologic examination shows no focal findings. On mental status examination, he is oriented to person and place but not to time. Attention and concentration are impaired. He recalls only one of three objects after 5 minutes. Which of the following is the most appropriate therapy for this patient's symptoms?\nOptions: A: Ventricular shunting\tB: Acetazolamide therapy\tC: Temporal lobectomy\tD: Donepezil therapy\t\nThe right answer is A: Ventricular shunting.", + "output": "The patient's symptoms suggest normal pressure hydrocephalus (NPH), which is characterized by the triad of gait difficulties, urinary incontinence, and cognitive impairment. His presentation with a broad-based gait, urinary urgency, inattention while driving, and impaired memory and concentration align with this diagnosis. NPH is caused by the accumulation of cerebrospinal fluid in the brain's ventricles, leading to increased pressure and the subsequent symptoms. The most definitive treatment for NPH is ventricular shunting, specifically a ventriculoperitoneal (VP) shunt, to divert excess fluid from the brain to the abdomen, relieving the symptoms by reducing intracranial pressure. Therefore, option A (Ventricular shunting) is the most appropriate therapy for this patient's symptoms. Options B (Acetazolamide therapy), C (Temporal lobectomy), and D (Donepezil therapy) are not typical treatments for NPH. Donepezil is used for Alzheimer's disease, which does not fully explain his symptom profile." + } + ] +}